Módulo I: Disciplinar, Didáctico Y Evaluación De Los Aprendizajes Del área De Matemática _______________________________

  • Uploaded by: Eleazar MP
  • 0
  • 0
  • February 2021
  • PDF

This document was uploaded by user and they confirmed that they have the permission to share it. If you are author or own the copyright of this book, please report to us by using this DMCA report form. Report DMCA


Overview

Download & View Módulo I: Disciplinar, Didáctico Y Evaluación De Los Aprendizajes Del área De Matemática _______________________________ as PDF for free.

More details

  • Words: 44,393
  • Pages: 144
Loading documents preview...
Taller de Actualización Docente del Área de Matemática UGEL CASMA

MÓDULO I DISCIPLINAR, DIDÁCTICO Y EVALUACIÓN DE LOS APRENDIZAJES DEL ÁREA DE MATEMÁTICA _______________________________ BLOQUE TEMÁTICO 1 Resolvemos situaciones de cantidad

2018

MÓDULO PEDAGÓGICO PARA EDUCACIÓN SECUNDARIA DE ÁREA DE MATEMÁTICA

2

ÍNDICE PRESENTACIÓN.......................................................................................................................................7 INTRODUCCIÓN..............................................................................................................................9 RUTA FORMATIVA GENERAL............................................................................................................10 COMPETENCIAS GENERALES............................................................................................................11 COMPETENCIA ESPECÍFICA...............................................................................................................11 PRODUCTO DEL BLOQUE TEMÁTICO................................................................................................11 RUTA FORMATIVA ESPECÍFICA.........................................................................................................11 UNIDAD I..........................................................................................................................................12 ESQUEMA DE CONTENIDOS.............................................................................................................12 1. Reflexión desde La Práctica..........................................................................................................13 2. Reflexión Teórica..........................................................................................................................13 2.1. La educación matemática en el Perú........................................................................................14 2.1.1 El enfoque lógico-conjuntista...........................................................................................16 2.1.2 El enfoque centrado en la resolución de problemas........................................................18 2.1.3 El planteamiento y la resolución de situaciones problemáticas como práctica social......18 2.2. Del sistema de aprendizaje sensorial al sistema del aprendizaje conceptual...........................20 2.2.1. El pensamiento matemático y el enfoque centrado en la resolución de problemas........22 2.2.2. La interculturalidad y el enfoque centrado en la resolución de problemas......................24 2.2.3. Desarrollo de actitudes en el enfoque centrado en resolución de problemas.................25 2.1. Sobre la naturaleza y el concepto de competencia matemática..............................................26 2.3.1. Competencias y capacidades matemáticas......................................................................26 Actuar matemáticamente.....................................................................................................27 Pensar matemáticamente....................................................................................................27 2.3.2. La acción y el pensamiento matemático...........................................................................27 2.1. Necesidad de plantearnos un modelo formativo: la pedagogía problémica............................28 2.4.1. Rasgos de la pedagogía problémica..................................................................................29 2.4.2. Funciones de la pedagogía problémica.............................................................................29 2.4.3. Objetivos de la pedagogía problémica..............................................................................29 2.4.4. ¿Qué es una situación problemática?...............................................................................30 2.4.5. ¿Qué es resolver una situación problemática planteada?................................................30 2.4.6. La pedagogía problémica y su didáctica...........................................................................30 2.4.7. El juego y la pedagogía problémica..................................................................................31 2.4.8. Los materiales concretos en la pedagogía problémica.....................................................32 2.1. La etnomatemática y el enfoque centrado en resolución de problemas..................................32 3. Herramientas para la nueva práctica............................................................................................33 3.1 Actividad de reflexión individual y/o grupal.............................................................................33 3.2 Actividades a distancia.............................................................................................................33 3.3 Actividades metacognitivas......................................................................................................34 4. GLOSARIO.................................................................................................................................34 5. REFERENCIAS............................................................................................................................34 ESTRUCTURA DEL BLOQUE TEMÁTICO 1..........................................................................................36 PRESENTACIÓN.................................................................................................................................37 ESQUEMA DE LOS CONTENIDOS......................................................................................................37 1. Reflexión desde la práctica...........................................................................................................38 2. Reflexión teórica...........................................................................................................................38 2.1 ¿Cómo se relaciona la competencia con el enfoque del área?.................................................38 2.1.1 Historia de la mano y la cabeza: resolución de problemas como práctica social..............39 2.1.2 Historia del sistema numérico en el Perú.........................................................................40 2.2 ¿Cómo desarrollamos capacidades en la competencia de situaciones de cantidad?...............41 2.2.1 Capacidad 1. ¿Qué significa “matematizar situaciones” en esta competencia?......................42

3

2.2.2 Capacidad 2. ¿Qué significa “comunica y representa ideas matemáticas” en esta competencia?...................................................................................................................................42 2.2.3 Capacidad 3. ¿Qué significa “elabora y usa estrategias”?.......................................................43 2.2.4 Capacidad 4. ¿Qué significa “razona y argumenta generando ideas matemáticas”?..............44 2.3 ¿Cuáles son las orientaciones didácticas que emplearías para el desarrollo de la competencia?.......................................................................................................................................45 2.4 ¿Cómo debemos evaluar la competencia?...............................................................................46 2.5 ¿Cuáles son los estándares e indicadores nacionales de evaluación?......................................47 2.6 ¿Cuáles son los recursos y materiales que permiten el desarrollo de la competencia?...........50 3. Herramientas para la nueva práctica............................................................................................51 3.1 Actividad de reflexión individual/grupal...................................................................................51 3.2 Actividades a distancia.............................................................................................................51 3.3 Actividades metacognitivas......................................................................................................51 3.4 Autoevaluación.........................................................................................................................52 4. Glosario........................................................................................................................................52 5. Texto complementario.................................................................................................................52 6. Referencias...................................................................................................................................53 UNIDAD II.............................................................................................................................................54 PRESENTACIÓN.................................................................................................................................54 ESQUEMA DE LOS CONTENIDOS......................................................................................................54 1. Reflexión desde la práctica...........................................................................................................55 2. Reflexión teórica...........................................................................................................................56 2.1. Fundamento teórico..................................................................................................................56 2.1.1 ¿Cómo surgieron los números enteros?...............................................................................57 2.1.2 ¿Cómo surgieron los números racionales?...........................................................................58 2.1.3 Conceptos y procedimientos.......................................................................................................58 2.2 ¿Cuáles son las orientaciones didácticas que empleas para el desarrollo de los aprendizajes? 62 2.2.1 Producción de papas en Coyllurqi........................................................................................62 2.2.2 Previniéndonos contra las bajas temperaturas....................................................................66 2.2.3 El terreno de don César........................................................................................................69 3. Herramientas para la nueva práctica............................................................................................72 3.1. Actividades de reflexión individual...........................................................................................72 3.2. Actividades a distancia.............................................................................................................73 3.3. Actividades de metacognición..................................................................................................73 3.4. Actividad para reflexionar........................................................................................................73 3.5. Autoevaluación.........................................................................................................................73 4. Glosario....................................................................................................................................73 5. Texto complementario.............................................................................................................74 6. Referencias...............................................................................................................................74 Anexo...............................................................................................................................................75 UNIDAD III............................................................................................................................................78 PRESENTACIÓN DE LA UNIDAD.........................................................................................................78 ESQUEMA DEL LOS CONTENIDOS.....................................................................................................78 1. Reflexión desde la práctica...........................................................................................................80 2. Reflexión teórica...........................................................................................................................81 2.1. Fundamento teórico.................................................................................................................81 2.1.1. ¿Cómo surgió la proporcionalidad?......................................................................................81 2.2. ¿Cuáles son las orientaciones didácticas que emplearías para el desarrollo de los aprendizajes?.......................................................................................................................................83 2.2.1. La producción del biohuerto.................................................................................................84 2.2.2. ¿A más trabajadores, menos tiempo?..................................................................................87 2.2.3. Cuidemos nuestras carreteras..............................................................................................89 3. Herramientas para la nueva práctica............................................................................................92

4

3.1. Actividades de reflexión individual...........................................................................................92 3.2. Actividades a distancia.............................................................................................................92 3.3. Actividades metacognitivas......................................................................................................92 3.4. Reflexión...................................................................................................................................93 3.5. Autoevaluación.........................................................................................................................93 4. Glosario....................................................................................................................................93 5. Texto complementario.............................................................................................................93 6. Referencias...............................................................................................................................94 Anexo...................................................................................................................................................95 UNIDAD IV........................................................................................................................................97 PRESENTACIÓN DE LA UNIDAD.........................................................................................................97 ESTRUCTURA DE LOS CONTENIDOS..................................................................................................97 1. Reflexión desde la práctica...........................................................................................................98 2. Reflexión teórica...........................................................................................................................99 2.1. Fundamento teórico.................................................................................................................99 2.1.1. ¿Cómo surgió la teoría de los números?..............................................................................99 2.2. ¿Cuáles son las orientaciones didácticas que emplearías para el desarrollo de los aprendizajes?.....................................................................................................................................105 2.2.1. Una carta muy triste...........................................................................................................105 2.2.2. Los vendedores de frutas...................................................................................................108 3. Herramientas para la nueva práctica..........................................................................................114 3.1. Actividades de reflexión individual.........................................................................................114 3.2. Actividades a distancia...........................................................................................................114 3.3. Actividades metacognitivas....................................................................................................114 4. Glosario..................................................................................................................................115 5. Texto complementario...........................................................................................................115 6. Referencias.............................................................................................................................116 UNIDAD V.......................................................................................................................................119 PRESENTACIÓN DE LA UNIDAD.......................................................................................................119 CONTENIDOS DE LA UNIDAD..........................................................................................................119 1. Reflexión desde la práctica.........................................................................................................120 2. Reflexión teórica.........................................................................................................................121 2.1. Fundamento teórico...............................................................................................................121 2.1.1. ¿Cómo surgió el interés a través de la historia?.................................................................121 2.1.2. Conceptos y procedimientos..............................................................................................122 2.2. ¿Cuáles son las orientaciones didácticas que emplearías para el desarrollo de los aprendizajes?.....................................................................................................................................123 2.2.1. Buscando la mejor opción..................................................................................................123 2.2.2. Financiando el techado del aula.........................................................................................128 3. Herramientas para la nueva práctica..........................................................................................132 3.1. Actividades de reflexión individual.........................................................................................132 3.2. Actividades a distancia...........................................................................................................132 3.3. Actividades metacognitivas....................................................................................................132 3.4. Autoevaluación.......................................................................................................................133 4. Glosario..................................................................................................................................133 5. Texto complementario...........................................................................................................133 6. Referencias.............................................................................................................................134 Anexo.................................................................................................................................................135

5

PRESENTACIÓN Estimado docente: El presente módulo Disciplinar, Didáctico y Evaluación de los aprendizajes: “, un material educativo basado en el enfoque intercultural y por competencias, que parte de diversas situaciones problemáticas del área, de tal forma que te apoye en tu trabajo pedagógico de aula, para el desarrollo de las competencias del área y la mejora en los aprendizajes de los estudiantes. El presente módulo está dirigido a docentes del área de Matemática de Educación Secundaria, que quienes están comprometidos con la mejora continua de su práctica pedagógica. El módulo brinda los conocimientos disciplinares, las orientaciones didácticas y las técnicas e instrumentos de evaluación pertinentes y necesarios para que los participantes fortalezcan sus competencias y desempeño docente, a partir del análisis de su práctica pedagógica. De este modo, se promuevan procesos de autoformación que permitan su desarrollo profesional para atender con pertinencia las necesidades de los estudiantes y generar la mejora y logros de sus aprendizajes. La ruta metodológica para el desarrollo del presente módulo se enmarca dentro de los enfoques intercultural y por competencias, los que privilegian los espacios de autorreflexión del docente para garantizar procesos de mejora de la práctica pedagógica. En este sentido, se ha organizado el desarrollo del módulo en cuatro bloques temáticos, relacionados con las cuatro competencias del área de Matemática. Estos bloques temáticos se desarrollan a través de tres fases metodológicas: Reflexión desde la práctica. Esta primera parte del módulo se caracteriza por basarse en las experiencias más cercanas de los docentes participantes, con el fin de captar sus intereses y propiciar la activación de los saberes previos para garantizar la construcción progresiva de los aprendizajes. A partir de la problematización y cuestionamientos de hechos concretos sobre su práctica pedagógica, se da inicio al tratamiento de cada temática. Reflexión teórica. Esta segunda parte del módulo presenta los fundamentos teóricos que permiten, al docente participante, confrontar sus saberes previos con la información procedente de fuentes confiables. Esta fase metodológica se caracteriza por plantear actividades que demandan la activación de habilidades cognitivas de mayor complejidad para reconstruir sus conocimientos a la luz del manejo de teorías explícitas y de experiencias de interaprendizaje, que permitan la articulación de la teoría con la práctica pedagógica desde un aspecto disciplinar, didáctico y de evaluación. Herramientas para la nueva práctica. Esta tercera parte del módulo se orienta al desarrollo de la autorreflexión del docente desde una perspectiva orientada a la transformación y al mejoramiento de su práctica. Es el momento en que, luego de haber analizado información y reflexionado sobre su actuar, el docente participante aplica los nuevos saberes en su quehacer pedagógico, reconociendo la importancia de la autorreflexión para el mejoramiento de su práctica.

6

El bloque temático tiene como objetivo: reflexionar sobre la práctica docente, a partir del enfoque intercultural, consolidando las bases teóricas desde la reflexión teórica; asimismo, construir herramientas para la nueva práctica, lo que permitirá desarrollar y fortalecer las competencias profesionales a través de una acertada intervención pedagógica, tomando como base el enfoque de resolución de problemas, para dar un sentido a la matemática funcional y real, útil para la vida. De esta manera, estamos contribuyendo a un proceso intercultural, que invita al docente a elaborar su planificación curricular contextualizada a las necesidades de los estudiantes, a partir de las propias reflexiones de su quehacer pedagógico, que serán el soporte para generar nuevas prácticas pedagógicas que repercutan en actividades significativas para el aprendizaje de los estudiantes.

7

INTRODUCCIÓN En nuestro país, la educación matemática parece haber evolucionado en los últimos años, por lo menos a un nivel formal, a saber: se han venido asignando metas amplias y diferentes a las antiguas, se han anunciado reiteradas veces pasar de la pedagogía por objetivos conductuales a la pedagogía por competencias, se han introducido contenidos novedosos, se ha organizado y reorganizado sucesivas veces el currículo, se han anunciado en diversas ocasiones nuevos modelos de intervención pedagógica. Sin embargo, la actividad cotidiana en el aula, la manera como se transmite el conocimiento matemático al estudiante no ha mejorado significativamente. Y ello, a pesar de los enormes esfuerzos realizados por el Estado peruano en los últimos tiempos, en los procesos de capacitación docente en todos los niveles. Las ideas de Dienes, escritas hace más de cuatro décadas, siguen, entonces, todavía vigentes: Actualmente, son muy pocos los profesores de matemáticas, cualquiera sea el nivel en que trabajan, que se encuentren honestamente satisfechos del modo como transcurre su enseñanza. Dienes, Z.P. (1970). La construcción de las matemáticas. Barcelona: Vicens-Vives. p. 5. Es evidente que la situación actual de la educación matemática en nuestro país, así como en cualquier otro, se debe a la confluencia de múltiples factores. Sin embargo, pensamos que en el Perú uno de esos factores más gravitantes tienen que ver con el enfoque dominante del aprendizaje de la matemática, instalado en nuestro sistema educativo, el cual, en términos generales, configura la concepción dominante entre los docentes y los padres de familia acerca de qué es la educación matemática escolar y en qué consiste el quehacer matemático en el aula. Si visitáramos diversas aulas de las II.EE. de nuestro país, observando detenidamente lo que sucede al interior de las mismas , qué materiales disponen y cómo los utilizan , los cuadernos de los estudiantes, los roles tanto del docente como del estudiante , los padres de familia del aula , el Comité de aula, en suma, observando las variables que determinan la dinámica interna del aula y de las cuales depende la calidad educativa, es posible determinar rasgos comunes que caracterizan el enfoque determinante de aprendizaje en el aula. A pesar del muy amplio abanico de estilos de aprendizaje y prácticas docentes, es posible caracterizar al enfoque dominante de aprendizaje de la matemática en nuestro país.

8

COMPETENCIAS GENERALES Planifica la enseñanza de forma colegiada, garantizando la coherencia entre los aprendizajes que quiere lograr en sus estudiantes, el proceso pedagógico, el uso de los recursos disponibles y la evaluación, en una programación curricular en permanente revisión. Crea un clima propicio para el aprendizaje, la convivencia democrática y la vivencia de la diversidad en todas sus expresiones, con miras a formar ciudadanos críticos e interculturales. Conduce el proceso de enseñanza con dominio de los contenidos disciplinares y el uso de estrategias y recursos pertinentes, para que todos los estudiantes aprendan de manera reflexiva y crítica lo que concierne a la solución de problemas relacionados con sus experiencias, intereses y contextos culturales. Evalúa permanentemente el aprendizaje con los objetivos institucionales previstos, para tomar decisiones y retroalimentar a sus estudiantes y a la comunidad educativa, teniendo en cuenta las diferencias individuales y los contextos culturales. Reflexiona sobre su práctica y experiencia institucional y desarrolla procesos de aprendizaje continuo de modo individual y colectivo, para construir y afirmar su identidad y responsabilidad profesional.

COMPETENCIA ESPECÍFICA Aplica los conocimientos disciplinares actualizados y pertinentes, y de la didáctica del área, demostrando dominio en el desarrollo de cantidades, a través de la resolución de problemas en sus diversos niveles de complejidad, trabajando de manera creativa, reflexiva y colaborativa, tomando en cuenta el contexto donde labora.

PRODUCTO DEL BLOQUE TEMÁTICO Planificación de una sesión de aprendizaje utilizando las orientaciones didácticas, recursos e instrumentos de evaluación para favorecer el desarrollo de la competencia. Un video de una sesión planificada y ejecutada.

RUTA FORMATIVA GENERAL El módulo Dominio disciplinar, didáctico y de evaluación de aprendizajes en el área de Matemática: busca desarrollar en el participante el dominio teórico y el manejo de estrategias, recursos y herramientas didácticas, así como la aplicación de diversas técnicas e instrumentos de evaluación que le permitan el desarrollo de las competencias propias del área, a partir de un enfoque crítico, reflexivo e intercultural.

RUTA FORMATIVA ESPECÍFICA El bloque temático 1 “Resolvemos situaciones de cantidad en un contexto intercultural” está organizado en 5 unidades didácticas. Cada una ellas contiene tres partes: 1) reflexión desde la práctica; 2) reflexión teórica; y 3) herramientas para la nueva práctica, culminando con las orientaciones didácticas para el uso de materiales, recursos, técnicas e instrumentos de evaluación, que le permitan al docente mejorar su desempeño.

9

MÓDULO MÓDULO III III DISCIPLINAR, DISCIPLINAR, DIDÁCTICO DIDÁCTICO YY EVALUACIÓN EVALUACIÓN DE DE LOS LOS APRENDIZAJES APRENDIZAJES DEL DEL ÁREA ÁREA DE DE MATEMÁTICA MATEMÁTICA

A continuación, se presenta la ruta metodológica del módulo que se desarrollará a través de cuatro bloques temáticos:

BLOQUE TEMÁTICO 1: RESOLVEMOS SITUACIONES DE CANTIDAD EN UN CONTEXTO INTERCULTURAL La educación matemática peruana, enfoques y la necesidad de un nuevo modelo formativo Aplica los conocimientos disciplinares actualizados y pertinentes, y de la didáctica del área, demostrando dominio en el desarrollo de cantidades, a través de la resolución de problemas en sus diversos niveles de complejidad, trabajando de manera creativa, reflexiva y colaborativa, tomando en cuenta el contexto donde labora.

BLOQUE TEMÁTICO 2: RESOLVEMOS SITUACIONES DE REGULARIDAD, EQUIVALENCIA Y CAMBIO EN UN CONTEXTO INTERCULTURAL Aplica los conocimientos disciplinares actualizados y pertinentes, y de la didáctica del área, demostrando dominio en el desarrollo de regularidad, equivalencia y cambio, a través de la resolución de problemas en sus diversos niveles de complejidad, trabajando de manera creativa, reflexiva y colaborativa, tomando en cuenta el contexto donde labora.

BLOQUE TEMÁTICO 3: RESOLVEMOS SITUACIONES DE FORMA, MOVIMIENTO Y LOCALIZACIÓN EN UN CONTEXTO INTERCULTURAL Aplica los conocimientos disciplinares actualizados y pertinentes, y de la didáctica del área, demostrando dominio en el desarrollo de forma, movimiento y localización, a través de la resolución de problemas en sus diversos niveles de complejidad, trabajando de manera creativa, reflexiva y colaborativa, tomando en cuenta el contexto donde labora.

BLOQUE TEMÁTICO 4: RESOLVEMOS SITUACIONES DE GESTIÓN DE DATOS E INCERTIDUMBRE EN UN CONTEXTO INTERCULTURAL Aplica los conocimientos disciplinares actualizados y pertinentes, y de la didáctica del área, demostrando dominio en el desarrollo de gestión de datos e incertidumbre, a través de la resolución de problemas en sus diversos niveles de complejidad, trabajando de manera creativa, reflexiva y colaborativa, tomando en cuenta el contexto donde labora.

10

UNIDAD I

La educación matemática peruana, enfoques y la necesidad de un nuevo modelo formativo ESQUEMA DE CONTENIDOS

11

Reflexión desde la práctica

ENFOQUES ENFOQUES YY LA LA NECESIDAD NECESIDAD DE DE UN UN NUEVO NUEVO MODELO MODELO FORMATIVO FORMATIVO

Reflexión teórica

La educación matemática en el Perú

Del sistema de aprendizaje sensorial al sistema de aprendizaje conceptual

La pedagogía problémica y la educación matemática

Necesidad de plantearnos un modelo formativo: la pedagogía problémica

La etnomatemática y el enfoque centrado en resolución de problemas

Herramientas para la nueva práctica

Actividades de reflexión individual/equipo, a distancia, de metacognición y evaluación

Glosario, texto complementario, referencias.

1. Reflexión desde La Práctica Analicemos el texto iconográfico:

¡Muchachos…! En la última evaluación solo han aprobado 3 de 26.

12

Volviste a sacar 18 en Educación Física… pero qué vamos a hacer con Matemática, no sabes ni una.

Pero no entendí la clase, profesor, ¿imatacha rimakunkipas?

Enséñeme con los Tito, hoy no saldrás recursos de mi pueblo, A partir de la observación deresuelve la historieta, responde las siguientes preguntas: al recreo, yo amo la naturaleza. 10 ejercicios.

a) ¿El profesor emplea el enfoque intercultural en la clase? b) ¿Qué enfoque matemático utiliza el profesor? c) ¿Crees que el uso de la lengua materna del estudiante influye en el desempeño escolar?

2. Reflexión Teórica Con la intención de presentar una reflexión clara desde la práctica, haremos primero un breve recorrido histórico para ver cuál ha sido y cómo ha ido evolucionando el enfoque dominante de la matemática escolar en el Perú. A continuación, trataremos de precisar nuestras ideas sobre la educación matemática que nuestro país necesita.

2.1. La educación matemática en el Perú Comenzaremos a analizar, aunque sea muy genérica y someramente, lo que nuestra experiencia personal desde la práctica nos puede permitir, es decir, desde la educación matemática de épocas recientes. La matemática escolar de la década de 1960 se centraba en operaciones aritméticas, algebraicas, geométricas y trigonométricas, así como en la resolución de ejercicios y problemas para la aplicación de los conocimientos matemáticos previamente aprendidos. Un detalle importante que debemos considerar es que más que la expresión “enseñanza de la matemática” se solía emplear “enseñanza de cálculo”. La práctica docente perseguía, esencialmente, que cada estudiante llegara a apropiarse de las rutinas, procedimientos y artificios matemáticos, que le serían útiles para aplicarlos en la resolución de problemas tipo. El centro de atención del desarrollo curricular eran, pues, los algoritmos de lápiz y papel, y los modelos de resolución de ejercicios. La matemática escolar se convertía así en un conjunto de técnicas calculatorias, de modelos de resolución correspondientes a cada una de las operaciones aritméticas, algebraicas, geométricas y trigonométricas, más sus variantes, y de conceptos y términos específicos. Su enseñanza era frontal y con frecuencia interpretada como adiestramiento y trasmisión de conocimientos, pues con la explicación del instructor y el ejercicio insistente y reiterado del estudiante, se pretendía que este se apropiara de los contenidos matemáticos establecidos. Esta matemática denominada “matemática pura” estuvo posicionada en nuestro país hasta la década de 1970, fecha en la que ingresan a nuestro sistema educativo cambios, innovaciones y nuevas interpretaciones, que ya se habían estado fraguando en el medio internacional. En efecto, las iniciativas para el cambio en la enseñanza de la matemática se venían cristalizando desde aquella década en Occidente y Oriente, provenientes de tres ámbitos: el propiamente matemático, el

13

psicopedagógico y el político. Los grupos más progresistas del ámbito matemático aconsejaban que los avances y cambios producidos a lo largo del siglo en la matemática tuvieran un reflejo en la enseñanza. La necesidad de una renovación en los currículos que modificara los contenidos tradicionales adaptándolos al sentir de la época e introdujera otros nuevos se fue extendiendo entre investigadores y profesionales de la educación. La enseñanza de la matemática, se creía, debiera responder al espíritu y necesidades de la época e incluir las teorías modernas. En suma, se aconsejaba posicionar la “matemática moderna” en lugar de la matemática pura. En términos de enfoque, se pretendía reemplazar al enfoque lógico por el conjuntista o estructuralista. La matemática moderna (conceptos y términos conjuntistas, estructuras matemáticas, etc.), por ser considerada básica y unificadora, debería ser introducida en las escuelas. Las nociones de estructura matemática, elementos de la matemática combinatoria, el trabajo en bases de numeración distintas, nuevos campos numéricos y un nuevo enfoque de la geometría iban engrosando, poco a poco, los currículos de esta área. Con la renovación de los contenidos se pretendía no solo incluir conocimientos matemáticos modernos sino, además, y esto es trascendental, inculcar un nuevo modo de pensar que fomentara el razonamiento lógico y habilitara a los futuros ciudadanos para el trabajo científico y tecnológico. Al conjunto de estos nuevos contenidos matemáticos se le denominó, con un fin de marketing, como “matemática moderna” o “matemática estructuralista”. Por otra parte, estas innovaciones venían arropadas por interpretaciones psicopedagógicas del aprendizaje que daban sustento a la enseñanza activa. Un rol especial jugaron en este proceso de innovación psicopedagógica Piaget y sus colaboradores: la Escuela de Ginebra. La fundamentación psicológica a las diferentes pedagogías, es decir, el psicologismo, fue ganando cada vez más terreno en el ámbito educativo. De este modo, se pensaba que la enseñanza activa de la nueva matemática, de la matemática moderna, ayudaría a los estudiantes a pensar con rigor, a desarrollar el razonamiento lógico, a generalizar, etc. Por tanto, la enseñanza de la matemática moderna ahora tenía una nueva meta, a saber: contribuir a la formación integral de los estudiantes desarrollando el razonamiento lógico, las capacidades simbólicas y el pensamiento abstracto. De este modo, la matemática escolar, que en los años anteriores se circunscribía al aprendizaje de destrezas calculatorias y de resolución de ejercicios, ahora veía ampliada sus funciones, ya que su aprendizaje contribuía al desarrollo del pensamiento del aprendiz. Esta concepción de la educación matemática penetró de lleno a nuestro sistema educativo en todos sus componentes (materiales educativos, formación y capacitación docente, etc.) durante la reforma educativa de la década de 1970. Así nos encontramos, con el hecho de que, paulatinamente, a partir de esa década fue evolucionando la idea que se tenía en años anteriores sobre la enseñanza de la matemática. Antes de ello, se pensaba que enseñar matemáticas consistía, básicamente, en trasmitir conceptos matemáticos determinados (número, decena, recta, ángulo, etc.), propiedades matemáticas (la asociatividad, la conmutatividad, etc.), unas destrezas calculatorias, sobre todo algorítmicas, y unos patrones de resolución de ejercicios, tal y como venían indicados en los programas curriculares. Pero, a partir de esa década, va posesionándose la idea de que más que hablar de la enseñanza de las matemáticas se debería hablar del aprendizaje de las matemáticas. Con ello se pretende centrar la educación matemática en el estudiante más que en el docente. Entonces se empieza a considerar a la enseñanza y el aprendizaje de la matemática como medios excelentes para el desarrollo de ciertas capacidades matemáticas. Se trata ahora de convertir a cada estudiante en protagonista de su propio aprendizaje, proponiéndole actividades y pidiéndole participación activa en la manipulación de materiales concretos estructurados o no. Es decir, que el estudiante, en lugar de manipular solo símbolos o de enfrentarse únicamente a situaciones notacionales, como promueve la matemática pura, debería manipular también materiales concretos y/o enfrentarse a situaciones conjuntistas. En suma, estas nuevas concepciones sobre la educación matemática invitan a realizar cambios, igualmente, en los métodos de enseñanza y aprendizaje, así como en la utilización

14

de materiales. Este fenómeno interesante, a pesar de contribuir a un cambio metodológico paradigmático, no pudo desplazar a la matemática pura. Esta sobrevivió a tales avatares y se consolidó con el tiempo, formando, más bien, una unidad sinérgica con la matemática moderna. Por otro lado, hay que tener en cuenta, además, otro fenómeno interesante: el hecho de que, a medida que pasaban los años setenta y ochenta, la matemática se iba convirtiendo en el área de desarrollo humano estrella en todos los currículos. Ello debido a la creciente matematización que sufre nuestra tecnologizada sociedad y, por tanto, a la necesidad de unos conocimientos matemáticos que ayuden a interpretar el entorno. A medida que va creciendo el prestigio social de esta área, se va hablando con más fuerza de la educación matemática, expresión que contiene un significado más amplio que la enseñanza de las matemáticas. Resumiendo, de aquella idea de las matemáticas escolares como simplemente “cálculo” hemos ido derivando hacia la acepción actual, mucho más compleja, de “educación matemática”. Hoy, incluso se tiende a distinguir la matemática como disciplina científica respecto de las matemáticas escolares, toda vez que estas se han ido alejando del formalismo y del rigor deductivo propio de la disciplina matemática. Con estos precedentes llegamos a los años noventa, década en la que se va a realizar el Diagnóstico Nacional de la Educación Peruana (1993), a partir del cual, en nuestro país se comienza a implementar una serie de programas de mejoramiento de la calidad educativa. En este marco, la educación matemática sufre una serie de cambios, a saber: se han venido asignando metas amplias y diferentes a las antiguas, se han anunciado reiteradas veces pasar de la pedagogía por objetivos conductuales a la pedagogía por competencias, se han introducido contenidos novedosos (estadística, probabilidad, programación lineal, etc.), se ha organizado y reorganizado sucesivas veces el currículo, se han anunciado en diversas ocasiones nuevos modelos de intervención pedagógica, etc. Sin embargo, la actividad cotidiana en el aula, la manera como llega el conocimiento matemático al estudiante no ha cambiado significativamente. Y ello, a pesar de los enormes esfuerzos realizados por el Estado peruano. Esta situación actual de la educación matemática en las instituciones educativas de nuestro país se debe, sin duda, a la confluencia de múltiples factores, de los cuales, en nuestra opinión, uno de los más influyentes es la concepción epistemológica dominante entre los diferentes actores (profesorado, autoridades, padres de familia, etc.) sobre: (1) cuál es la matemática escolar que la educación peruana necesita (¿La matemática pura, la matemática moderna, la matemática realista o la matemática funcional?) y (2) en qué se fundamenta o se basa la construcción del conocimiento matemático (¿En la lógica, la teoría de conjuntos o ambas, a la vez, la base de la construcción del conocimiento matemático? ¿O es el planteamiento y la resolución de situaciones problemáticas la movilizadora de la construcción de los conocimientos matemáticos? Estas preguntas pueden ser sintetizadas en una sola: ¿Cuál es el enfoque que la educación matemática peruana necesita?). Gracias a influjos de muy diversa procedencia (psicología del aprendizaje, psicología del desarrollo, epistemología de la matemática, reformas educativas, etc.) se ha ido posesionando el enfoque lógico-conjuntista en nuestro sistema educativo. Y a pesar de que el año 2013, después de una serie de marchas y contramarchas que se inició el año 1996, se definió el enfoque problémico para la educación matemática de nuestro país, pareciera que todavía hoy sigue teniendo validez la expresión: así aprendí la matemática en la escuela, en el colegio y en la universidad y así seguirán aprendiendo las nuevas generaciones. Impera, pues, todavía una concepción de la educación matemática conformada a base de tradición, que es coherente con la perspectiva epistemológica desarrollada en los pasajes precedentes. La matemática pura y la matemática moderna han tomado cuerpo en el aula y unidas forman un todo global que se apoya en una tercera idea firmemente arraigada en la creencia: toda enseñanza produce aprendizaje.

15

2.1.1 El enfoque lógico-conjuntista Para el enfoque lógico-conjuntista, el objetivo prioritario de la enseñanza y aprendizaje de la matemática es el desarrollo del razonamiento “lógico”, entendiendo por tal el razonamiento ordenado y certero. Este enfoque se gestó en nuestro país en el marco de la reforma educativa de la década de 1970. Varios aspectos confluyeron en su gestación:  El inicio de reformas en la enseñanza y aprendizaje de la matemática que introdujeron la teoría de conjuntos en la escuela básica.  La moda de la matemática moderna que trajo consigo el estructuralismo y el olvido de contenidos tradicionales.  La influencia creciente de la Escuela de Ginebra o piagetiana. A este respecto hay que destacar la notable influencia del INIDE (Instituto Nacional de Investigación y Desarrollo de la Educación) hasta bien entrada la década de 1990. Por diferentes razones, el enfoque lógico-conjuntista en la educación básica resultó ser un fracaso; pero el paso del tiempo fue muy piadoso con él, ya que sus huellas siguen siendo significativas en nuestro sistema educativo, a pesar de los cambios cíclicos emprendidos. No entraremos aquí a valorar este hecho en toda su magnitud para no complicar la elucidación de sus repercusiones conceptuales y metodológicas en nuestro sistema educativo. En adelante, sin embargo, utilizaremos situaciones muy elementales y del dominio común, de suerte que queden reducidas al mínimo las circunstancias que velan la esencia de la cuestión. Solo nos detendremos, entonces, en cuatro puntos: qué es el número natural, qué son las operaciones aditivas (suma y resta), cuándo y cómo iniciarlas y, finalmente, cuándo y cómo iniciar el planteamiento y resolución de situaciones problemáticas aditivas. Desde el punto de vista del enfoque lógico-conjuntista, el número natural es una construcción que resulta del dominio de la ordenación de pequeñas cantidades y de la inclusión jerárquica de unas en otras. Cuando la síntesis entre ambos aspectos se produce, permanece invariable el valor numérico. Es lo que llaman invariancia o conservación de la cantidad: se conserva el valor del número con independencia de la disposición física y, además, se usa el número como atributo en la comparación de cantidades. El número, es pues, desde este enfoque, lo que decía Piaget: la síntesis entre la seriación y la inclusión jerárquica de clases. El número es construido por los seres humanos gracias a la reflexión sobre acciones de ordenación e inclusión, y a la coordinación e interiorización de esas acciones. En su planificación, los docentes lógico-conjuntistas suelen seguir el siguiente itinerario: Primero: La matemática es una actividad principalmente cualitativa y se enfatizan actividades que corresponden a los siguientes indicadores:  Forma colecciones de objetos, tomando en cuenta características comunes, y expresa por qué los agrupó.  Compara colecciones de objetos, usando la correspondencia uno a uno, y expresa dónde hay “más que”, “menos que” y “tantos como”.  Expresa si “muchos”, “pocos”, “uno” o “ninguno” de los objetos de una colección tienen una característica señalada. Segundo: Se inicia con la matemática cuantitativa y se enfatizan actividades que corresponden a los siguientes indicadores:

16

 Construye operativamente el número (primeros números naturales), en el que enfatiza la cardinalidad (nombre de un conjunto – colección de objetos).  Asocia una cantidad de hasta 5 objetos con el símbolo del número que le corresponde.  Señala la posición de un objeto en una fila, usando los ordinales primero, segundo, tercero, cuarto y quinto. Tercero: Se inicia con las actividades aditivas y se enfatizan actividades que corresponden a los siguientes indicadores:  Compone y recompone números sobre cantidades concretas pequeñas.  Agrega o quita una cantidad en colecciones de hasta 5 objetos, usando material concreto y el conteo.  Realiza operaciones de adición y sustracción con números pequeños. Cuarto: Resuelve problemas tipo en los que requiere agregar o quitar una cantidad en colecciones de hasta 5 objetos, usando material concreto y operaciones previamente aprendidas. ¿Cuándo debemos iniciar la enseñanza de la adicion y sustracción como operaciones aritméticas, es decir, trabajando con los códigos notacionales adecuados? Cuando ya hayan construido la noción de número. Y ¿cómo sabemos que ya tienen adquirido el concepto de número? Pues con pruebas como las famosas pruebas piagetianas de la conservación de la cantidad. ¿Cuándo y cómo iniciar con el planteamiento y resolución de situaciones problemáticas de cantidad? Cuando ya hayan aprendido a sumar y restar. Y ¿cómo sabemos que ya han aprendido a resolver ejercicios de adición y sustracción? Pues con pruebas como las planas de ejercicios que se les aplica a los estudiantes. Así pues, desde la perspectiva lógico-positivista, el concepto de número se concibe como cardinalidad de una colección, la sustracción y adición se construyen sobre la base de las operaciones conjuntistas de unión y diferencia de conjuntos, y luego se resuelven problemas de adición y sustracción como actividades de aplicación de operaciones aritméticas previamente aprendidas. Frente a la posición lógico-positivista que prioriza el razonamiento lógico, en la perspectiva problémica se da protagonismo al planteamiento y resolución de situaciones problemáticas. A continuación, desarrollamos el enfoque problémico.

2.1.2 El enfoque centrado en la resolución de problemas. El enfoque problémico coloca, como objetivo prioritario de la enseñanza y aprendizaje de la matemática, el planteamiento y la resolución de situaciones problemáticas. Mientras que para los lógico-conjuntistas la actividad matemática está orientada, sobre todo, al razonamiento lógico mediante la acción sobre objetos y colecciones, situaciones y símbolos, los problémicos sostienen que el razonamiento también se desarrolla con el planteamiento y resolución de situaciones problemáticas. Por lo tanto, centran su atención en ello, siendo clave el uso de estrategias informales como punto de partida no solamente para el planteamiento y la resolución de problemas, sino también para el aprendizaje de estrategias escolarizadas. En el marco de lo dicho, para el enfoque problémico, la estrategia prioritaria es la estrategia heurística. El enfoque problémico y las prácticas de su órbita consideran innecesarias las fundamentaciones de índole piagetiana, y una pérdida de tiempo el trabajo con la teoría de conjuntos. A decir verdad, siempre estuvieron en contra de ellas. Ya Morris Kline (1976) se adelantaba proclamando el fracaso de la matemática moderna.

17

El número, desde esta perspectiva, no se construye al modo lógico-operatorio que interpreta la epistemología genética, sino que, como afirma la epistemología historicista: el número se engendra por la actividad del planteamiento y la resolución de situaciones problémicas de cantidad. El planteamiento y la resolución de situaciones problemáticas sirven de contexto no solo para construir conceptos matemáticos, sino también para descubrir procedimientos y estrategias matemáticas. El objetivo prioritario de la enseñanza y aprendizaje de la matemática en el marco del enfoque problémico es el desarrollo de la competencia matemática “Resolución de situaciones problemáticas”. Desde esta perspectiva, el razonamiento no es más que una de las herramientas de la competencia matemática señalada. Por consiguiente, la focalización ya no está centrada en el desarrollo del pensamiento lógico, sino en la resolución de situaciones problemáticas.

2.1.3 El planteamiento y la resolución de situaciones problemáticas como práctica social. Cabe aquí la parábola de cómo, durante mucho tiempo, nuestras manos fueron enseñando a nuestra cabeza. Con el tiempo, nuestras manos fueron adiestrándose y nuestra cabeza, despejándose. Nuestra habilidad manual desarrollaba nuestra inteligencia; pero según nuestra cabeza se aclaraba, más frecuentemente dirigía el trabajo de nuestras manos. Nuestras manos no podían levantar un pesado bloque de piedra y nuestra cabeza aconseja colocar una palanca. Esta solo nos puede ayudar a levantar la piedra un tanto, ¿pero cómo subirla a lo alto? De nuevo interviene nuestra cabeza: crea el plano inclinado, nos recomienda colocar troncos redondos bajo la piedra, ¡hacer rodar es más fácil que arrastrar! Sin embargo, la construcción de un plano inclinado para elevar pesos es faena laboriosa y compleja, y nuestra cabeza encuentra otra vez una solución más simple: inventa la polea. Haciendo pasar la cuerda por ella el peso sube mejor; y si, además, el peso cuelga de una segunda polea móvil, nuestras dos manos pueden levantar un objeto que cuatro manos movían con dificultad. Pero esto nos pareció poco. Entre manos y el peso colocamos tres, cinco poleas, siete poleas, etc. Cuantas más son las poleas, más fuertes nos hacemos. Así, ahora levantamos, sin gran trabajo, pesos cuyo manejo era exclusividad de los gigantes. Nuestra cabeza ayuda a nuestras manos, pero estas tampoco le dan reposo a aquella. Le plantean muchas otras tareas. No es fácil hacer subir el agua del río para que reguemos los campos, y nuestra cabeza crea el pozo con la “cigüeña”, con lo que nosotros hacemos subir el cubo del río. Pero un cubo es poco; hace falta más agua. Nuestras manos ya no se dan abasto, y aparece el torno. Una manivela sujeta a un rodillo, nuestra mano hace girar la manivela, el rodillo da vueltas enroscando una cuerda, esta arrastra un cubo. ¡Asombrosos descubrimientos! Durante miles de años ayudarían a nuestras manos en su trabajo. Pero crece la demanda de agua, y aumenta el trabajo. La necesidad es la mejor de las maestras. Nuestra cabeza piensa: ‘‘¿No se podría hacer eso mismo, pero sin las manos?’’. Recuerda a los cuadrúpedos servidores de los humanos, acostumbrados desde largo tiempo a transportar cargas sobre sí. Nuestras manos enganchan un cuadrúpedo a la palanca, el caballo da vueltas, haciendo girar una rueda dentada. Esta, tras su rotación al rodillo que arrolla la cuerda, a la que va sujeta a una tina. Nuestras manos se liberan así de un trabajo que puede realizar un animal.

18

En cambio, les espera un problema más complicado: construir los dientes de la rueda. Nuestras manos van haciendo trabajos cada vez más delicados y complejos, pero también nuestra cabeza tiene que resolver tareas más arduas. El ser humano utiliza al caballo para sacar agua del río, pero comienza a pensar si no se puede prescindir aquí del noble bruto. ¿Para qué emplear al caballo? ¡Que el propio río suba el cubo de agua y lo vierta en el surco! Nuestras manos reciben una tarea más complicada: construir y colocar en el río una rueda tal que saque ella misma el agua. El río corre por su lecho y tropieza con un obstáculo: las aletas de la rueda. El río las empuja, y eso es lo que se buscaba. La rueda gira, carga el agua y la sube, vertiéndola por último en el canalón. El río riega los campos en los que crece el trigo. El otoño llega y la cosecha debe ser recogida y el grano de las espigas, molido. Hubo tiempos en que se molía el grano en pequeños molinos de mano. Esto era suficiente para una familia campesina, pero cuando hubo que dar de comer a ejércitos enteros, cuando surgieron enormes ciudades que necesitaban inmensas cantidades de harina para las tahonas, fue necesario poner en marcha grandes molinos y túselas de piedra. Semejantes muelas no se podían mover a mano, y de nuevo nuestra cabeza busca la forma de salir del problema. Entonces vuelve a probar la manivela, recuerda otra vez al caballo y a aquel trabajador más fuerte que el caballo: el agua. El ser humano ya había dominado al río. Quita los cangilones de la rueda y deja las aletas. En su eterno andar, el río empuja las aletas, hace girar el rodillo-eje que mueve la rueda dentada; esta se engancha en otra que pone en movimiento un nuevo eje, en el que se encuentra la muela. Al principio, todo esto debió parecer un cuento, y los primeros molinos de agua significaron, probablemente, una gran fiesta: la blanca espuma del agua al estrellarse contra la rueda, la nube blanca de harina flotando sobre la muela y las mujeres alrededor, escuchando el jubiloso zumbido del molino de agua, más agradable que el triste chirrido del primitivo manual. Mas con todo su júbilo, no comprendían entonces la portentosa fuerza que habían descubierto. ¿Podían acaso suponer que el molino de agua sería el origen de centenares de máquinas que no solo molerían el trigo, sino que forjarían el hierro, machacarían el mineral, tejerían? Estas máquinas que trabajarían por el ser humano y para el ser humano, que habrían de vestirlo y alimentarlo y, más tarde, incluso trasportarlo por el aire. De esta manera, la resolución de problemas es consustancial a nuestra propia existencia como ser social. Una vez que como homo sapiens nos erigimos sobre el resto del reino animal, nuestra propia vida nos impone encontrar soluciones a los disímiles problemas que nos plantea nuestra supervivencia misma. La adaptación a un medio que cambiaba ante nuestros ojos, tanto por los cambios que objetivamente se producen en nuestro entorno (escasez de alimentos, condiciones climáticas adversas, etc.) como por nuestra propia visión que vamos teniendo de la realidad que nos rodea, nos plantea a diario situaciones problemáticas para las que no poseemos respuesta inmediata, o contradicen nuestras creencias establecidas o somos incapaces de resolverlas con los instrumentos (materiales o teóricos) para enfrentarlas. Así, a lo largo de nuestra milenaria existencia sobre el planeta Tierra, nuestra historia ha discurrido a través de la resolución de problemas cada vez más complejos, en un número cada vez mayor de ámbitos de nuestra propia vida y del medio que nos rodea. La historia de la humanidad es también, entonces, la historia de la resolución de sus problemas; precisamente, a esto se debe, como hemos visto, el avance de la ciencia y la tecnología, y de la matemática en particular. Nuestras cabezas y nuestras manos siguen unidas como en el pasado, ayudándose entre sí. Y el conocimiento consolida y sintetiza la grandeza de la capacidad humana: plantear y resolver situaciones problemáticas.

19

2.2. Del sistema de aprendizaje sensorial al sistema del aprendizaje conceptual ¿Pero cómo se “hace’’ la matemática? ¿Cómo los sabios han construido el conocimiento matemático? ¿Cómo calcularon el diámetro de la Tierra en la remota antigüedad? ¿Cómo supieron que la Tierra es esférica si hasta hace poco nadie la había visto desde afuera? Para construir el conocimiento matemático, los sabios se valen de todos sus sentidos, exactamente lo mismo que cualquiera de nosotros. Cuando contemplamos todo lo que nos rodea, observamos lo que nos interesa. Los sabios también contemplan y observan. Hay muchas cosas que conocemos tocándolas, palpándolas, valiéndonos del sentido del tacto. El oído nos permite distinguir innumerables sonidos producidos en el aire por los diferentes objetos que vibran y se mueven. Zumba un insecto. Ese ruido se forma por las múltiples vibraciones de sus delgadísimas alas. Sacudidos por el viento, los postes de telégrafo zumban con sus voces de bajo. El olfato es de gran utilidad. El gusto nos permite diferenciar lo dulce de lo salado, lo amargo de lo ácido. Los sabios también se valen del oído, el gusto y el olfato. Pero no siempre se conforman con lo que de por sí puede transmitir cada uno de esos órganos de los sentidos acerca del mundo que los rodea. ¿Y por qué? ¿Es que los sentidos nos engañan? Sí, a veces nos engañan. Cuando miramos una línea férrea, nos parece que los rieles se juntan a lo lejos cuando, en realidad, son siempre paralelos. Pero esto es fácil de comprobar. Basta con hacer una regla del ancho de los rieles y allí donde la apliquemos abarcará el espacio de un riel al otro. Cuando la Luna sale, nos parece mayor que cuando llega al centro del cielo. ¿Es así en realidad? También esto se comprueba fácilmente. Recojan ustedes la imagen de la Luna en un espejo. Esta imagen será siempre del mismo tamaño, aproximadamente el de una moneda. Resulta que no tiene mayor importancia que a veces nuestros sentidos nos engañen. Además, contamos con muchos medios de suplir los defectos de los órganos sensoriales. Con unos sentidos podemos comprobar los otros.

20

El ojo humano puede distinguir decenas de matices de cualquier color a los que ni siquiera podemos dar nombre. Tenemos el rosado o el anaranjado, pero entre ellos apreciamos decenas de sutiles tonalidades. Si en un conjunto de mil lámparas se apaga una, la vista advierte esa pequeña diferencia de iluminación. Pero no puede distinguir una tela de araña algo más fina que un pelo. La mano no percibe un peso algo menor al de un gramo. Mas las fallas de nuestros órganos de los sentidos no solo pueden corregirse; también es posible ampliarlos, reforzarlos, tornarlos más sutiles. No importa que la mano no perciba un peso pequeño. Con ayuda de una balanza diminuta, podemos pesar objetos millones de veces más livianos que el gramo. No importa que la tela de araña no se vea en cuanto se desprende del techo. El ser humano ha creado aparatos para incrementar de tal modo la vista que nos permite ver la estructura de seres infinitesimales, un millón de los cuales puede instalarse en la punta de una aguja. Este extraordinario aparato se llama microscopio electrónico. La vista no percibe todos los rayos solares; por ejemplo, los que ponen la piel morena, los ultravioletas. Pero los instrumentos registran infaliblemente su presencia e intensidad. Hay sonidos –los ultrasonidos– que no se captan con el oído; sin embargo, existen dispositivos que los registran con facilidad. Tenemos una aguja magnética que oscila en la caja de la brújula. No podemos sentir directamente la fuerza magnética, pero la aguja señala dónde están el Norte y el Sur. La corriente fluye por los conductores alimentando las lámparas. La vista y el tacto no nos permiten determinar su intensidad, dirección o magnitud. Mas el ser humano ha creado aparatos que permiten observar las propiedades del magnetismo y la acción de la electricidad. Muchos de los dispositivos con los cuales la humanidad estudia la naturaleza son complejos aparatos técnicos. Toda la potencia de la nueva tecnología está puesta hoy al servicio de la ciencia y de su aprovechamiento. En ese sentido, la nueva tecnología no es otra cosa que la prolongación de nuestros sentidos. De esta manera, no hay nada en nuestro cerebro que no haya pasado por nuestros órganos sensoriales. Nuestros sentidos son una especie de mediadores entre la naturaleza y nuestro cerebro. Este procesa la información que captamos del mundo circundante a través de los sentidos. Como resultado se obtiene un conocimiento. Por esta razón, se suele decir que el conocimiento es el producto de procesos de abstracción y generalización de regularidades que se observan en la naturaleza. No obstante, la mayoría de los conceptos matemáticos no tienen su origen en los objetos, sino en las relaciones que establecemos entre dichos objetos. Así, por ejemplo, el concepto “dos” no está presente en el par de elementos que tenemos frente a nosotros, sino en la relación entre los elementos que establecemos al entender que uno es el primero y el otro es el segundo y que el “dos” al que llegamos en el conteo resume la cantidad del conjunto de elementos de que

21

disponemos. En cambio, el concepto “hexágono” puede ser un ejemplo de abstracción “física” de un objeto real, por cuanto podemos extraerlo de un panal de abejas.

2.2.1. El pensamiento matemático y el enfoque centrado en la resolución de problemas No vamos a hablar aquí acerca del proceso histórico del origen del ser humano y su facultad de pensar, es decir, del problema de la filogénesis del pensamiento, sino de la ontogénesis, la formación del pensamiento matemático en el humano contemporáneo. El niño no nace como ser pensante, llega a serlo. Las condiciones socioculturales, históricamente formadas, dan origen a factores cada vez nuevos que determinan el desarrollo del pensamiento y hacen que cada niño se familiarice con la riqueza de las formas intelectuales elaboradas por sus pueblos. En ese sentido, el pensamiento matemático, lo mismo que cualquier otro fenómeno complejo, se puede considerar, según sus distintos aspectos y desde distintos puntos de vista, en particular como facultad de plantear y resolver situaciones problemáticas, cualesquiera que sean: trátese de problemas escolares (por ejemplo, los que se da a resolver a los estudiantes de primer grado de primaria: “A ver, José Gabriel, si tenías tres canicas y has ganado cinco canicas, ¿cuántas canicas tienes ahora en total?”), de situaciones problemáticas cotidianas o de contexto científico, o de los que se plantean a quienes diseñan máquinas o a los que desentrañan los enigmas del micro o macromundo. ¿Acaso hay modo de enumerar todas las situaciones problemáticas que se han planteado y se plantean diariamente? Sin discernir y comprender estos problemas, sin saber encontrar las vías de solución, el ser humano no puede existir ni un día y es incapaz de llevar a término no ya algo serio, sino ni siquiera lo más baladí. La psicología considera las funciones psíquicas como formas indispensables de la actividad del sujeto, como proceso en que estén planteadas y resueltas tales o cuales situaciones problemáticas. Ahora bien, en términos generales, plantear o resolver una situación problemática significa, ante todo, transformar una situación inicial, lo cual se consigue mediante determinadas operaciones que se realizan mentalmente y luego se trasladan al exterior, al objeto. La tarea estriba, precisamente entonces, en aclarar de qué modo las acciones objetivas se convierten en mentales y aparece, con ello, un nuevo proceso psicológico. En lo que sigue, para no complicar la elucidación de cómo se forman los procesos mentales, utilizaremos situaciones problemáticas sencillas y de dominio común, de modo que queden reducidas a su mínima expresión las circunstancias que velan la esencia de la cuestión. Así que volvamos al problema que le plantearon a José Gabriel, veamos cómo lo resolvió y qué contestó este niño de primer grado de Educación Primaria:

Si tenías tres canicas y has ganado cinco canicas, ¿cuántas canicas tienes en total?

http://2.bp.blogspot.com/-T2Epz5WdP40/T_n9cjWjj_I/AAAAAAAAAeE/eXFL9WZudOM/s1600/juego+de+canicas.jpg

22

La profesora pregunta a José Gabriel: “¿Cuántas canicas tienes en total?”. –¡Siete! –respondió José Gabriel. –¡Qué dices, José! –le replicó en tono de censura la profesora–. Piénsalo bien, ¿cuánto resulta si a las tres canicas que tenías les agregas las cinco canicas que has ganado? –¡Nueve! –gritó con viveza José Gabriel. –¡No, niño, no! –la profesora miró a José Gabriel, sin disimular su descontento, y movió la cabeza–. Piénsalo bien y no te precipites. ¡Piensa! El niño arrugó la frente y alzó la mirada al techo. Lo mismo solía hacer su mamá cuando decía: “¡No me distraigas, déjame pensar!”. Esperó unos diez segundos, dirigió la mirada a su profesora y dijo, indeciso: –Ocho. –Ya era hora –la profesora exhaló un suspiro de alivio. Y en tono edificante, dirigiéndose tanto a los demás estudiantes como a José Gabriel, añadió en conclusión: “¡Antes de responder una pregunta, es necesario pensar, hay que pensar bien lo que se va a responder!”.

José Gabriel se sentó en su asiento y en la clase nadie se dio cuenta de que aquel niño –y no solo él – no sabía pensar correctamente cuando se trataba de resolver problemas aditivos. Era un muchacho listo. Sabía divertirse en los juegos más diversos, con el “tangram” armaba figuras espléndidas y con el “mecano” construía extraordinarias máquinas, grúas y barcos; por síntomas imperceptibles que solo él captaba, se daba exacta cuenta de cuál era el estado de ánimo de papá y mamá en un momento dado y, en consecuencia, decidía si le era posible hacerse un poco el caprichoso o si debía cumplir sin chistar lo que le mandaban. En cambio, en la escuela… Para ejecutar tales operaciones aditivas a un nivel abstracto, es indispensable dejar el uso de objetos reales y calcular mentalmente. Al principio, ello resulta bastante difícil, la vista se desliza sin querer hacia los objetos que se adicionan. Luego aparece el hábito, de momento solo en forma de operación ejecutada en voz alta. El niño dice “uno”, y la palabra ha sustituido al palito; dice dos, y ha sustituido al segundo… La acción material se transfiere al plano del lenguaje en voz alta, donde las palabras que se pronuncian se convierten en objetos del cálculo. Ahora bien, tales palabras no se deben pronunciar como sea, al azar, sino en exacta correspondencia con el curso del proceso que reflejan o representan. Las palabras, a su vez, son representadas por otros símbolos y estos, por otros aún más abstractos, y así sucesivamente. De esta manera, nosotros, en lugar de manipular objetos, nos acostumbramos a manipular símbolos. Así llegamos a un punto de extraordinaria importancia: transferir la acción con materiales concretos a imágenes ideales, es decir, llegamos a pensar matemáticamente. En el jardín, José Gabriel aprendió a contar de uno a diez. Él sabía que dos son más que uno antes de ingresar al primer grado; también sabía que tres son más que dos. En la escuela, se dio cuenta de que es mayor aquel número para llegar al cual hay que contar más tiempo. Uno, dos, tres, cuatro, cinco: la cuenta es más larga; por lo tanto, el número es mayor. Luego José Gabriel comprendió que cuando se dice “añadir” y preguntan cuánto resulta, hay que nombrar un número algo mayor. Se imaginaba, asimismo, sin tener de ello plena conciencia, que es necesario decir un número mayor que el que figura en el problema. Los problemas aditivos aparecían para José Gabriel como un juego del tipo “¡Adivina lo que tengo en mano!”, al que a menudo jugaba con otros niños en la calle. Ahora bien, en el juego decía: “Canica, piedra, hayruro”, mientras que en la escuela pronunciaba los nombres de los números que conocía. A veces, acertaba la primera vez, y entonces procuraba grabar en la memoria la feliz combinación. “Si a tres añadimos uno, tenemos cuatro”, esta combinación la recordó enseguida. Pero, otras veces, tenía que decir dos o tres números antes de que la maestra comentara. “Bueno, ahora has pensado y has hallado la solución correcta”. Sin embargo, cada vez, tanto si daba una respuesta acertada como si respondía equivocadamente, el niño pensaba del mismo modo, es decir, siempre

23

aplicaba el mismo procedimiento mental: citaba al azar uno de los diez números que conocía, procurando que el número fuera “un poco mayor” si se trataba de sumar o “un poco menor” si se trataba de restar. José Gabriel era un alumno aplicado: no hacía travesuras en la clase, escuchaba lo que la profesora decía y copiaba cuidadosamente de la pizarra todos los ejemplos. Era pulcro al escribir, no tenía un borrón en el cuaderno, y la profesora estaba contenta con él. Todo marchó bien hasta que llegaron las primeras actividades matemáticas centradas en resolución de problemas. El niño “resolvió” el primer problema que implicaba realizar la operación aritmética: 3 + 1 = 4. El segundo le obligó a cavilar la operación que involucraba sumar: 5 + 2 = … ¿Cuál de los números hay que escribir en este caso? José Gabriel no sabía elegir por sí mismo el número adecuado. Lo único que podía hacer era citar un número, al azar; pero quien debía decidir si había acertado o no era la profesora. Por eso, era la profesora. Así se encontró el niño en una situación difícil. Pero, en aquel momento, su maestra le dijo: –Cuando resuelvas un problema, reflexiona bien, ¡recuerda cómo en situaciones semejantes nos ayudábamos con bastoncitos para encontrar la respuesta! Entonces José Gabriel se acordó: a los palitos colocados en orden sobre el pupitre, a un lado, añadían palitos de los que formaban un montón. Colocaban los palitos uno al lado del otro; por consiguiente, también había que colocar las cifras una al lado de la otra. Contaban todos los palitos juntados y así obtenían el resultado requerido. José Gabriel aplica este modelo y halla la solución del problema. De esta manera, desarrollar una amplia actividad con material concreto aparece como condición necesaria para que se forme la actividad mental. Más uno no debe quedarse en esta etapa. El paso siguiente es el de la generalización, que resulta de la abstracción de la estructura de las operaciones realizadas con materiales concretos.

2.2.2. La interculturalidad y el enfoque centrado en la resolución de problemas. Nuestro país es pluricultural y multilingüe. En consecuencia, la educación matemática, para ser pertinente a esta realidad, tiene que ser intercultural y multilingüe. Desde la perspectiva del enfoque problémico, esto se traduce en los siguientes términos: En primer lugar, debemos plantear a nuestros estudiantes situaciones problémicas con un contexto sociocultural concreto que recoja la realidad pluricultural del país. En segundo lugar, debemos generar espacios de aprendizaje y de reflexión que permitan a nuestros estudiantes desarrollar sus capacidades matemáticas utilizando las formas de comunicación, expresión, conocimiento, y formas de conocer de nuestras culturas ancestrales, en diálogo intercultural y en pie de igualdad con otras culturas. En este sentido, afirmamos que la educación matemática tiene una perspectiva intercultural. El conocimiento matemático, a lo largo de su historia, ha tenido varias formas de presentarse. Una de ellas es la matemática “estándar”, que se ha constituido como modelo de pensamiento rigurosamente formal. El mayor logro de esta matemática ha sido el de convertirse en un valioso instrumento para el desarrollo científico y tecnológico. A la par de esta matemática, existe el conocimiento matemático de grupos sociales, cuyas construcciones y formas de presentación son diferentes. Este conocimiento se denomina etnomatemático.

24

La etnomatemática se concibe, entonces, desde la constatación de que cada cultura tiene su forma de enfrentar y resolver situaciones problemáticas, que les ha permitido construir su propio conocimiento sobre cuestiones relativas a la cantidad, al orden, al espacio y forma, al cambio y relaciones, a la incertidumbre, etc. En el contexto de lo dicho, podemos encontrar en nuestras culturas andinas y amazónicas, por ejemplo, tradiciones numéricas, conocimientos geométricos, formas de organización y representación de información cuantitativa, claramente diferenciables, que deben ser incorporados en los procesos de enseñanza y aprendizaje de nuestros estudiantes. Así tenemos: la tradición quechua, de base decimal, basada en agregación y multiplicación; la tradición aimara, que deja huellas de un sistema de numeración quinaria; la tradición centro amazónica (arahuac pano), de base binaria y procedimientos duplicativos, distinguiendo claramente la paridad e imparidad de los números; la tradición haranmbut, cuya base de numeración es ternaria; entre otras.

2.2.3. Desarrollo de actitudes en el enfoque centrado en resolución de problemas. La importancia del enfoque problémico radica en que eleva el grado de la actividad mental en el aula, promueve el pensamiento creativo y contribuye al desarrollo de la personalidad de los estudiantes. En efecto, la actividad mental es aquella característica de la personalidad, que representa el esfuerzo, la perseverancia, la constancia intelectual que el estudiante debe realizar conscientemente en la resolución de una situación problémica. Con el incremento sistemático del nivel de la actividad mental en el proceso de enseñanza-aprendizaje, se fomenta el aprendizaje consciente de la matemática y se desarrolla la independencia cognoscitiva, afectiva y volitiva de los estudiantes. El uso sistemático de la metodología problémica posibilita, además, la adquisición de experiencia en la actividad creativa, con la que el estudiante posteriormente podría seguir aprendiendo, para consolidar así, gradualmente, el desarrollo de su pensamiento creativo. No menos significativa es la contribución del enfoque problémico al desarrollo de la personalidad de los estudiantes. Es comprensible que esta forma de estructurar el aprendizaje de la matemática favorece el desenvolvimiento del razonamiento y de importantes operaciones del pensamiento, así como la formación de convicciones y rasgos de conducta, que se convierten en un motor impulsor del desarrollo de la personalidad del estudiante. Desde este punto de vista, el enfoque problémico constituye una vía potente y eficaz para desarrollar actitudes positivas hacia la matemática. Este tipo de desarrollo se relaciona con verse uno mismo capaz de resolver situaciones problémicas y capaz de aprender matemática, considerándola útil y con sentido para la vida. Entendida de esta forma, la posibilidad que los estudiantes tienen para enfrentarse a situaciones problemáticas con diferentes niveles de exigencia matemática junto con la lógica del trabajo grupal que el docente construye en el aula son elementos importantes para el desarrollo de actitudes positivas hacia esta disciplina, aspiraciones que la sociedad peruana le plantea a la escuela peruana contemporánea.

25

2.3.1.Sobre la naturaleza y el concepto de competencia matemática. La competencia matemática nace desde posiciones básicamente funcionales como una alternativa a modelos formativos tradicionales y como una consecuencia de la necesidad de superar al aprendizaje memorístico de conocimientos matemáticos, hecho que conlleva a la dificultad para que estos conocimientos puedan ser aplicados a la vida real. Po lo tanto, la competencia matemática, en el ámbito de la educación matemática, identifica aquello que cualquier estudiante necesita para enfrentar una situación problemática en su vida cotidiana, es decir, la competencia matemática alude, sobre todo, a una actuación eficaz en diferentes contextos de la vida cotidiana a través de una serie de herramientas y acciones que movilizan e integran diferentes componentes del saber pensar matemáticamente. En el marco de lo dicho, vamos a entender a la competencia matemática como un saber actuar en un contexto particular, que nos permite resolver situaciones problemáticas. Un actuar pertinente a las características de la situación y a la finalidad de nuestra acción, que selecciona y moviliza una diversidad de saberes propios o de recursos del entorno, a través de procedimientos que satisfagan determinados criterios básicos. Un saber actuar: Alude a la intervención de una persona sobre una situación problemática determinada para resolverla, pudiendo tratarse de una acción que implique solo actividad matemática. En un contexto particular: Alude a una situación problemática real o simulada pero plausible, que establezca ciertas condiciones y parámetros a la acción humana, y que deben tomarse en cuenta necesariamente. Un actuar pertinente: Alude a la indispensable correspondencia de la acción con la naturaleza del contexto en el que se interviene para resolver la situación problemática. Una acción estereotipada, que se reitera en toda situación problemática, no es una acción pertinente. Que selecciona y moviliza saberes: Alude a una acción que echa mano de los conocimientos matemáticos y habilidades, y de cualquier otra capacidad matemática que le sea más necesaria para realizar la acción y resolver la situación problemática que enfrenta. Que utiliza recursos del entorno: Alude a una acción que puede hacer uso pertinente y hábil de toda clase de medios o herramientas externas, en la medida que el contexto y la finalidad de resolver la situación problemática lo justifiquen. A través de procedimientos basados en criterios: Alude a formas de proceder que necesitan exhibir determinadas características, no todas las deseables, sino aquellas consideradas esenciales para que logren validez y efectividad.

2.3.1.Competencias y capacidades matemáticas. En correspondencia a lo descrito en los pasajes precedentes y tomando en cuenta la definición de competencia matemática dada líneas arriba, se ha formulado la competencia matemática curricular en términos de un saber actuar y de un saber pensar matemáticamente, que, proyectada a cuatro situaciones, dan lugar a cuatro competencias fundamentales para los requerimientos de los procesos de enseñanza y aprendizaje de esta área en la Educación Básica, a saber:

26

Actúa y piensa matemáticamente en situaciones de regularidad, equivalencia y cambio.

Actúa y piensa matemáticamente en situaciones de cantidad.

COMPETENCIA

Actúa y piensa matemáticamente en situaciones de forma, movimiento y localización.

Actúa y piensa matemáticamente en situaciones de gestión de datos e incertidumbre.

-

Actuar matemáticamente.

La matemática es un área de desarrollo humano donde el saber hacer prevalece sobre el saber qué. Es una ciencia, como diría Miguel de Guzmán, en la que el método prevalece sobre el contenido: hay que conceder una gran importancia al estudio de cuestiones que se refieren a los procesos de planteamiento y resolución de situaciones problemáticas. En el contexto de lo dicho, actuar matemáticamente no es otra cosa que plantear y resolver diversas situaciones problemáticas, a saber: situaciones de cantidad; situaciones de regularidad, equivalencia y cambio; situaciones de forma, movimiento y localización; y situaciones de gestión de datos e incertidumbre.

-

Pensar matemáticamente.

El pensar matemáticamente es la facultad de operar con imágenes mentales, colocadas entre sí en diferentes relaciones que corresponden a las que se dan entre los objetos reales. Con esto, no son ya las cosas y los procesos reales los que se convierten en objeto de la actividad del pensamiento matemático, sino sus imágenes ideales de la conciencia. Concebida así, la facultad de operar matemáticamente implica un conjunto de procesos mentales, como matematizar, razonar y argumentar, comunicar y representar, y elaborar y usar estrategias.

2.3.2.La acción y el pensamiento matemático. Para el pensamiento ya formado, las relaciones entre el pensamiento matemático y la acción pasan a ser contrarias respecto de lo que sucede mientras el pensamiento está en proceso de formación. Cuando en el ser humano se han estructurado ya las acciones mentales, toda acción práctica, cualquiera que sea, comienza con la representación que de ella se tenga. La persona primero establece un plan de acción, aclara qué desearía obtener mediante su actividad, examina las posibles vías para alcanzar el fin propuesto y los posibles resultados a que conducirán tales o cuales acontecimientos. Primero piensa y luego actúa. La actividad humana cuando se enfrenta a diversas situaciones problemáticas, resultaría imposible sin la previa acción del pensamiento matemático. De manera que el pensamiento desarrollado no solo permite reflejar pasivamente aquella situación problemática con que el ser humano se enfrenta directamente en la realidad circundante, sino que la persona, además, prevé y enjuicia de antemano los resultados de la actividad.

27

Ahora, sin embargo, para el pensamiento aún no formado o que está en proceso de formación, las acciones reales preceden a las acciones mentales. El pensamiento necesita de las acciones para desarrollarse. Por esta razón, se debe transitar del sistema de aprendizaje sensorial hacia el sistema de aprendizaje conceptual. Así, el planteamiento y la resolución de situaciones problemáticas reales movilizan el desarrollo de las capacidades matemáticas.

A C T U A R M A T E M Á T I C A M E N T E

PENSAR MATEMÁTICAMENTE Capacidad

Plantear

Resolver

Matematiza

Razona y argumenta generando ideas matemáticas

Comunica y representa ideas matemáticas

Elabora y usa estrategias

Actuar y pensar matemáticamente en diversas situaciones implica la sinergia de dos condiciones interrelacionadas: 1. Condición necesaria: El planteamiento y la resolución de diversas situaciones problemáticas deben movilizar el desarrollo de las capacidades matemáticas fundamentales, como: matematizar situaciones, comunicar y representar ideas matemáticas, elaborar y usar estrategias para plantear y resolver problemas, razonar y argumentar generando ideas matemáticas a través de sus conclusiones y respuestas, y usar diversas herramientas matemáticas. 2. Condición suficiente: Las acciones mentales del pensamiento deben desplegar un plan de acción que nos permita plantear y resolver situaciones problemáticas con eficiencia y eficacia.

2.4.1.Necesidad de plantearnos un modelo formativo: la pedagogía problémica. Esta perspectiva de aprendizaje de la matemática, centrada en el desarrollo de competencias, nos obliga a plantearnos un modelo formativo que priorice el saber hacer sobre el saber teórico, es decir, sobre el saber por el saber, y que se oriente hacia el desarrollo de las capacidades del ser humano, o sea, hacia la formación integral de las personas. Desde esta lógica de formación integral o para la vida, el conocimiento matemático debe ser aprendido funcionalmente en diversos ámbitos cotidianos, incluido el académico. Es, precisamente, en nuestras instituciones educativas, dada su función, donde la formación matemática, focalizada en planteamiento y resolución de situaciones problemáticas, se puede convertir en un verdadero cambio paradigmático que nuestro país necesita. La propuesta pedagógica que asume el enfoque problémico como marco para el desarrollo de las competencias y capacidades matemáticas fundamentales es la pedagogía problémica, que promueve el aprendizaje helicoidal y marca una ruptura epistemológica con la tradicional manera de concebir el aprendizaje de la matemática como lineal y por acumulación. La pedagogía problémica implica pasar de un aprendizaje que, en la mayoría de los casos, se ha reducido a una memorización de conocimientos matemáticos como prerrequisitos para aprender a resolver problemas (hecho que conlleva la dificultad de que estos conocimientos puedan ser aplicables a la vida real) a poner énfasis en el desarrollo del pensamiento matemático y la construcción de conocimientos matemáticos, a partir del planteamiento y resolución de situaciones problemáticas.

28

La pedagogía problémica promueve la didáctica inversa como herramienta de intervención formativa, que consiste en partir de situaciones problémicas para construir conceptos y descubrir procedimientos y propiedades matemáticas, para actuar y pensar matemáticamente.

2.4.1.Rasgos de la pedagogía problémica. Los rasgos más importantes de la pedagogía problémica son los siguientes:  El planteamiento y la resolución de situaciones problemáticas debe impregnar íntegramente el currículo de la matemática El planteamiento y la resolución de situaciones problemáticas no son un tema específico ni tampoco una parte diferenciada del currículo de la matemática, sino que constituyen el eje vertebrador alrededor del cual se organizan la enseñanza, el aprendizaje y la evaluación de la matemática.  La matemática se enseña y se aprende planteando y resolviendo situaciones problemáticas El planteamiento y la resolución de situaciones problemáticas sirven de contexto para que los estudiantes construyan nuevos conceptos matemáticos, descubran relaciones entre entidades matemáticas y elaboren procedimientos propios de esta disciplina.  Las situaciones problemáticas deben plantearse en contextos de la vida real o en contextos científicos y tecnológicos Los estudiantes se interesan en el conocimiento matemático, le encuentran significado y utilidad, lo valoran más y mejor cuando pueden establecer relaciones de funcionalidad matemática con situaciones de la vida real o de contexto científico y tecnológico, ya que en el futuro ellos necesitarán aplicar cada vez más matemática durante su vida.  Las situaciones problemáticas deben responder a los intereses y necesidades de los estudiantes Los problemas deben ser interesantes para los estudiantes y responder a sus demandas cognitivas, a fin de involucrarlos en la búsqueda de sus soluciones.  El planteamiento y la resolución de situaciones problemáticas sirven de contexto para desarrollar capacidades matemáticas A través de la resolución de problemas, los estudiantes desarrollan sus capacidades matemáticas, tales como: la matematización, la representación, la comunicación, la utilización de expresiones simbólicas, la argumentación, el uso de herramientas matemáticas, etc.

2.4.2.Funciones de la pedagogía problémica La pedagogía problémica surge como una alternativa de solución a dificultades que nos enfrentamos en nuestro quehacer docente, por     

Dificultades de razonamiento matemático. Problemas de significatividad y funcionalidad de los conocimientos matemáticos. Problemas de aburrimiento, valoración y falta de interés por la matemática. Problemas de desarrollo del pensamiento crítico. Problemas de contextualización del pensamiento matemático.

29

2.4.3.Objetivos de la pedagogía problémica Los objetivos de la pedagogía problémica son lograr que el estudiante:  Se involucre en una situación problemática (tarea o actividad matemática) para plantearla y resolverla con iniciativa y entusiasmo.  Comunique y explique el proceso de planteamiento y resolución de situaciones problemáticas.  Razone de manera efectiva, adecuada y creativa partiendo de un conocimiento integrado, flexible y utilizable, durante todo el proceso de planteamiento y resolución de situaciones problemáticas.  Busque información y utilice los recursos que promuevan un aprendizaje significativo.  Sea capaz de evaluar su propia capacidad de resolver la situación problemática planteada.  Reconozca sus fallas en el proceso de construcción de sus conocimientos matemáticos.  Colabore de manera efectiva, como parte de un equipo que trabaja para lograr una meta común.

2.4.4.¿Qué es una situación problemática? Estuvimos frente a una situación problemática cuando no pudimos levantar un pesado bloque de piedra. La resolvimos cuando inventamos la palanca. Pero esta no podía ayudarnos a levantar ese pesado bloque de piedra un tanto más y, entonces, nuevamente nos enfrentamos a otra situación problemática. Esta vez la resolvimos inventando el plano inclinado. Así, estuvimos frente a sucesivas y nuevas situaciones problemáticas, a las cuales encontramos sus soluciones correspondientes. De igual modo, estaremos frente a una situación problemática cuando no podamos sostener a nuestra familia o cuando no podamos resolver un problema matemático. Una situación problemática es, entonces, una situación de dificultad ante la cual hay que buscar y dar reflexivamente una respuesta coherente, es decir, encontrar una solución.

La dificultad señalada puede haber sido generada porque es una situación nueva o desconocida, o siendo conocida, no disponemos de estrategias o medios conocidos de solución.

2.4.5.¿Qué es resolver una situación problemática planteada? Resolver una situación problemática planteada es:     

Encontrar una solución para la situación problemática. La manera de superar un obstáculo. Encontrar una estrategia allí donde no se disponía de estrategia alguna. Hallar la forma de salir de una dificultad. Lograr lo que uno se propone utilizando los medios adecuados.

2.4.6.La pedagogía problémica y su didáctica En el marco de la pedagogía problémica, la didáctica consiste en lo siguiente:  Se les presenta a los estudiantes una situación problemática. Ellos, conformados en grupos, organizan sus ideas y el conocimiento previo relacionados con la situación problemática y tratan de definir su naturaleza.

30

 A través del diálogo, los estudiantes hacen preguntas sobre aspectos específicos de la situación problemática que no les ha quedado del todo claros. El grupo se encarga de anotar estas preguntas. Los estudiantes son animados por el profesor para que puedan definir lo que saben y lo que no saben.  Los estudiantes seleccionan, en orden de prioridad e importancia, los temas a investigar que se generan por medio de las preguntas acerca de la situación problemática. Ellos deciden qué interrogantes serán contestadas por todo el grupo y cuáles serán investigadas por algunos miembros para después socializarlas en el equipo. Los estudiantes y el docente dialogan acerca de la búsqueda de los recursos necesarios para investigar las preguntas.  Los estudiantes vuelven a juntarse en grupo y exploran las interrogantes previamente establecidas integrando su nuevo conocimiento al contexto de la situación problemática. Ellos también deben resumir ese conocimiento y conectar los nuevos conceptos y procedimientos a los previos. De este modo, continúan definiendo nuevos temas a investigar mientras progresan en la resolución de la situación problemática planteada. Los estudiantes comienzan a ver que el aprendizaje es un proceso en curso progresivo de naturaleza compleja y que siempre existirán temas para investigar cuando se enfrentan a situaciones problemáticas.

2.4.7.El juego y la pedagogía problémica En el marco de la pedagogía problémica, los juegos, en general (y, en particular, los juegos de contenido matemático), se presentan como un excelente recurso didáctico para plantear situaciones problemáticas con contexto lúdico. Tales situaciones permiten articular aspectos tan importantes como las actividades matemática y lúdica, en contextos de interacción grupal. Las situaciones problémicas lúdicas son actividades predominantes, especialmente, de los niños de los primeros ciclos. A esa edad, es posible dirigir la atención y esfuerzo de los niños hacia metas de naturaleza matemática mediante el juego. Por las consideraciones señaladas, especialmente durante los primeros ciclos, el juego constituye un valioso instrumento de intervención pedagógica para iniciar a los niños en la construcción de las nociones y procedimientos matemáticos básicos. La estimulación del niño para que aprenda a resolver situaciones problémicas se puede efectuar a través de actividades, no necesariamente lúdicas, que le interesen. Sin embargo, a fin de satisfacer su necesidad de jugar, es recomendable que, en lo posible, se le proponga al niño situaciones problémicas a través del juego. En esta dinámica, los estudiantes tienen la oportunidad de escuchar a los otros, explicar y justificar sus propios descubrimientos, confrontar ideas y compartir emociones, corregir y ser corregidos por sus compañeros. Tales juegos tienen un doble aliciente, la actividad lúdica en sí misma y la actividad matemática que la acompaña, además del hecho de realizarse en grupo. El juego es una estrategia indispensable para desarrollar y aprender la matemática. Tratar el tema lúdico resulta indispensable especialmente en los niveles de educación inicial y primaria a partir de las siguientes constataciones:  El juego es la primera actividad natural que desarrollan los niños y niñas para aprender, desarrollando sus primeras actividades y destrezas.  Los juegos permiten dinamizar los procesos de pensamiento, pues generan interrogantes y motivan la búsqueda de soluciones.  Presentan desafíos y estímulos que incitan la puesta en marcha de procesos intelectuales.

31

 Los juegos estimulan la competencia sana y actitudes de tolerancia y convivencia que crean un clima de aprendizaje favorable.  Favorece la comprensión.  Facilita la consolidación de contenidos matemáticos.  Ejercitan capacidades.  Conectan el juego con la vida y el aprendizaje.

2.4.8.Los materiales concretos en la pedagogía problémica Para la pedagogía problémica, los materiales manipulativos o concretos son considerados hoy en día, especialmente en los primeros grados, un avance irrenunciable en el aprendizaje de la matemática. De acuerdo con esta pedagogía, se debe entregar a los estudiantes materiales concretos sobre los que ejerzan acciones también concretas. A través de dicha práctica, estas acciones se irán internalizando progresivamente de manera que, al retirarles paulatinamente los materiales concretos, los estudiantes las van a reproducir mentalmente, ligándolas a otras. Reconociendo el rol mediador e irrenunciable de los materiales concretos, estructurados o no, deben quedar claras dos conclusiones didácticas, a saber:  El uso de los materiales educativos no es el objetivo de la enseñanza-aprendizaje de la matemática, sino el medio del que nos valemos para que las acciones de nuestros estudiantes, especialmente las acciones infantiles, se hagan manifiestas.  La mayoría de los conceptos matemáticos no tienen su origen en los objetos, sino en las relaciones entre dichos objetos que establecen los estudiantes. El color “rojo” es un ejemplo de abstracción física por cuanto se abstrae de los objetos de que disponen. El concepto “dos”, sin embargo, no está presente en las fichas que tienen los estudiantes frente a ellos, sino en la relación entre las fichas, que establecen al entender que una es la primera y la otra es la segunda, y que el “dos” al que llegamos en el conteo resume la cantidad del conjunto de fichas de que los estudiantes disponen. De esta manera, no son los materiales lo que ha de hacerse manifiesto, sino las acciones y las relaciones establecidas sobre ellos.

2.5.1.La etnomatemática y el enfoque centrado en resolución de problemas Los seres humanos nos hemos enfrentado siempre a una serie de situaciones problemáticas o dificultades a lo largo de nuestra existencia. Y para resolverlas hemos usado nuestro cerebro, o más exactamente hemos recurrido a una de sus funciones que es algo que no se ve, inmaterial o invisible. En suma, hemos apelado a algo que es inaccesible a nuestros sentidos. Pues bien, eso que no podemos ver ni tocar se llama pensamiento. Cuando pensamos, a su vez, interactuamos con mediadores (signos, símbolos, grafías, nociones, conceptos, etc.). Es decir, pensamos apoyándonos en herramientas o “cosas” (significantes) que nos remiten o nos evocan a otras. Esta dualidad es una característica humana: operamos con representaciones de objetos que nos remiten a los objetos mismos; resolvemos diversas situaciones problemáticas gracias a signos y símbolos que evocan en nosotros representaciones mentales y suscitan emociones o sentimientos; socializamos la información que generamos durante el proceso de resolución de situaciones problemáticas a través de huellas inscritas (íconos, gráficos, letras, signos, símbolos, números, etc.) que provocan ideas en el receptor del mensaje; utilizamos garabatos específicos cuando nuestro pensamiento va dirigido a situaciones problemáticas, por ejemplo, de cantidad: cifras. Esas huellas gráficas de todo tipo, palabras, íconos, símbolos, etc., son visibles o audibles, es decir, se muestran a nuestros sentidos. Matizando más, como diría Manuel Alcalá, son creaciones humanas en 32

un contexto cultural determinado que vamos haciendo nuestras a medida que crecemos. Son signos. Pero carecerían de poder instrumental si no fuera porque son los soportes de construcciones mentales, de significados personales. En efecto, los signos se convierten en significantes, es decir, en portadores de significado, pues vinculan los significados y, con ello, hacen posible el pensamiento conceptual y la comunicación. En suma, el ser humano es un animal que posee un distintivo: el pensamiento representacional. De manera que la formación del pensamiento representacional, que va de la mano con el lenguaje, es un hecho cultural. En el contexto de lo dicho, cada pueblo ha expresado la realidad o una porción de ella en un lenguaje matemático a su modo. Es decir, cada pueblo la ha matematizado de una manera singular, propia. Igualmente, la ha representado con sus propios recursos simbólicos; la ha descrito y explicado también a su manera. Asimismo, ha hecho inferencias y ha predicho sobre los fenómenos de la realidad a su modo. Ha diseñado y elaborado estrategias de resolución de problemas. Ha construido instrumentos propios para operar sobre la realidad. En suma, cada pueblo ha construido su pensamiento matemático a su modo, ya que la matematización, representación, comunicación, argumentación, razonamiento, uso y elaboración de estrategias y recursos matemáticos, como capacidades matemáticas fundamentales que son, constituyen rutas que permiten construir el pensamiento matemático. En el sentido estricto de la palabra: El pensamiento matemático no es otra cosa que la facultad de actuar, operar con imágenes ideales (nociones, conceptos matemáticos), colocadas entre sí en diferentes relaciones que corresponden a las que se dan entre los objetos reales.

En consecuencia, ya no son las cosas o los procesos reales los que se convierten en objeto de la actividad, sino sus imágenes ideales que se forman en la conciencia. Los resultados de estos procesos mentales no son otra cosa que saberes matemáticos, los cuales, construidos por cada pueblo, constituyen, a su vez, lo que se llama etnomatemática.

3. Herramientas para la nueva práctica 3.1

Actividad de reflexión individual y/o grupal

Actividades para ser desarrollados en forma individual o en equipo. a) ¿La pedagogía problémica toma en cuenta aspectos del calendario comunal y el aporte de los sabios de la comunidad? b) ¿Qué enfoque pedagógico utilizas en tu práctica pedagógica?, ¿de qué manera? c) ¿Qué cambios propones realizar en tu práctica pedagógica a partir del desarrollo de la unidad? Justifica. d) ¿Cómo evaluarías en una enseñanza de la pedagogía problémica? 3.2

Actividades a distancia.

A continuación, se presentan actividades para que pongas en práctica lo desarrollado en esta unidad, las cuales serán ejecutadas en el componente a distancia.

33

a) En un GIA institucional, realiza tertulias con tus pares sobre el enfoque centrado en la resolución de problemas y pedagógica problemática. b) Elabora un organizador visual sobre sobre tu práctica pedagógica actual, con relación al enfoque centrado en la resolución de problemas. c) Organizar las evidencias en portafolio. 3.3

Actividades metacognitivas.

a)

¿Qué aspectos de la unidad general piensas que te van a servir para mejorar tu práctica cotidiana? b) ¿En qué medida la reflexión desde la práctica y la reflexión teórica piensas que influirá en la mejora en tu trabajo cotidiano? c) ¿Cómo fue tu participación durante el desarrollo de la unidad? ¿Cuál es el compromiso asumido? 4. Glosario 









Pensamiento matemático: El pensamiento matemático es la facultad de actuar, operar con imágenes ideales (nociones, conceptos matemáticos), colocadas entre sí en diferentes relaciones que corresponden a las que se dan entre los objetos reales. Actuar matemáticamente: El actuar matemáticamente no es otra cosa que plantear y resolver diversas situaciones problemáticas, a saber: situaciones de cantidad, situaciones de regularidad, equivalencia y cambio; situaciones de forma, movimiento y localización y situaciones de gestión de datos e incertidumbre. Competencia matemática: La competencia matemática es un saber actuar complejo en un contexto particular, es decir, es un conjunto de capacidades que permiten a las personas actuar con eficiencia en un contexto determinado y resolver allí problemas. Capacidad matemática: La capacidad matemática es un atributo síquico, una herramienta mental o función del cerebro que permite plantear y resolver diversas situaciones problemáticas. Situación problemática: La situación problemática es una situación de dificultad ante la cual hay que buscar y dar reflexivamente una respuesta coherente, es decir, encontrar una solución.

5. Referencias Aitkenhead, A.M. y J.M. Slak (1985). Issues in cognitive modelling. New Jersey:Lawrence Erlbaum Associates Ltd, Publishers Hillsdale. Chifflet, María Ofelia (1999). El paradigma de las competencias. Ginebra: Unión Internacional de Comunicaciones –UIT. Costa, Arthur (1991). Developing Minds. Alexandria (Virginia): ASCD. Dante, Luiz. (1991). Didáctica de resolucao de problemas de Matemática. Editora Atica. S.A. Sao Paulo. Brasil. De Sánchez, Margarita A. (1994). Desarrollo de habilidades del pensamiento: Procesos básicos del pensamiento. México: Ed. Trillas. Durch, B.J. (1995).What is Problem-Based Learning? About Teaching 47. Gardner, Howard (2000). Una introducción formal a la enseñanza para la comprensión.En: H. Gardner. La educación de la mente y el conocimiento de las disciplinas. Barcelona: Paidós, págs. 136 -157. Kolmogorov, A.S. (1984). Kak mui poznaem bsë. Ukrania: Izdatelstvo Nauka. Le Boterf, Guy (2000). Ingeniería de las competencias. Barcelona: Ediciones Gestión.

34

Lévy-Leboyer, Claude (2003). Gestión de las competencias: Cómo analizarlas, cómo evaluarlas, cómo desarrollarlas. Barcelona: Ediciones Gestión 2000. OCDE (2004). La definición y selección de competencias clave. Proyecto de Definición y Selección de Competencias (DeSeCo) de OECD. www.OECD.org/edu/statistics/deseco Pimm, D. (1990): El lenguaje matemático en el aula. Madrid. Morata. Ministerio de Educación del Perú. (2013). Rutas de aprendizaje. Fascículo 2. Schleicher, Andreas (2009). Lo que el Perú puede aprender de los resultados com parados de las pruebas Pisa. En: Boletín CNE N° 21, junio 2009. Tobón, Sergio (2007). El enfoque complejo de las competencias y el diseño curricular por ciclos propedéuticos. En: Acción Pedagógica, Nº 16 / Enero-diciembre, 2007, pp.14-28. Villavicencio Ubillús, Martha. Et. Al. (1995). Guía Didáctica: Resolución de problemas matemáticos .La Paz – Bolivia. Zabala, Antonio y Arnau, Laia. (2007). 11 ideas clave. Cómo aprender y enseñar competencias.Editorial GRAÓ, de IRIF, S.L. España.

35

ESTRUCTURA DEL BLOQUE TEMÁTICO 1

BLOQUE TEMÁTICO 1 Resolvemos situaciones de cantidad en un contexto intercultural

UNIDAD I: Conceptos básicos En esta unidad, se desarrollan conceptos teóricos de la competencia relacionados con cantidad, desde una perspectiva intercultural con un enfoque crítico-reflexivo.

UNIDAD II: Sistemas de numeración En esta unidad, se presentan los fundamentos básicos del sistema de numeración a través de la historia de la matemática y los sistemas de numeración N, Z y Q, con orientaciones didácticas que apoyan a la labor docente, utilizando materiales y recursos disponibles del contexto.

UNIDAD III: Magnitudes Proporcionales En esta unidad, se presentan situaciones de proporcionalidad a partir de actividades cotidianas, donde se dan las orientaciones para la construcción de conceptos de magnitudes directa e inversamente proporcionales, planteando orientaciones para una evaluación por competencias a través de instrumentos de autoevaluación y el uso de materiales y recursos de su contexto.

UNIDAD IV: Teoría de números En esta unidad, se presenta la historia de los números y el proceso de construcción de los criterios de divisibilidad, máximo común divisor y mínimo común múltiplo, empleando el método de Pólya para resolver problemas con materiales y recursos del contexto, y utilizando instrumentos pertinentes para evaluar competencias.

UNIDAD V: Matemática financiera En esta unidad, desarrollamos el significado y uso del interés simple y compuesto a partir de la resolución de situaciones problemáticas, con orientaciones didácticas de resolución, evaluando competencias a través de instrumentos pertinentes, y utilizando materiales y recursos del contexto.

36

Conceptos básicos

PRESENTACIÓN Estimados participantes del programa de actualización docente, dirigido a profesores de secundaria rural, en esta oportunidad te presentamos la unidad I, que tiene como objetivo reflexionar sobre las bases teóricas y construir herramientas para la nueva práctica, que te permitirá brindar una acertada intervención pedagógica para desarrollar la competencia “Actúa y piensa matemáticamente en situaciones de cantidad”, dentro de un enfoque intercultural, centrado en la resolución de problemas, para construir una matemática funcional, útil para la vida.

ESQUEMA DE LOS CONTENIDOS Reflexión desde la práctica

Reflexión teórica Competencia "Actúa y piensa matemáticamente en situaciones de cantidad"

CO N CEPTO S BÁSICO S

Capacidades del área

Orientaciones didácticas Estrategias de evaluación

Estándares e indicadores de desempeño

Materiales y recursos

Herramientas para la nueva práctica Actividades de reflexión inidvidual/equipo, a distancia, de metacognición y evaluación GLOSARIO, TEXTO COMPLEMENTARIO, REFERENCIAS Y EVALUACIÓN DE LA UNIDAD

37

1. Reflexión desde la práctica Estimado docente, estamos próximos a realizar nuestra planificación del año académico 2016; por ello, te presentamos la siguiente situación: Utilizo libros preuniversitarios en mi planificación.

La unidad finaliza con una evaluación bimestral a los estudiantes, sobre problemas tipo y ejercicios aritméticos.

Qué complicado. ¿Y los estudiantes comprenden aritmética?

Selecciono métodos prácticos de resolución de problemas tipo y ejercicios.

Estoy seleccionando los contenidos de aritmética que colocaré en mi unidad.

Estoy planificando mi unidad didáctica

A partir de la observación de la imagen, responde las siguientes preguntas: a) b) c) d) e) f) g) h)

¿El docente centra su planificación en contenidos o competencias? Fundamenta. ¿Qué estrategias de aprendizaje considera el docente en la imagen? ¿Qué enfoque matemático está utilizando el docente en su planificación? ¿Cómo evalúa la unidad? Desde tu experiencia, ¿qué competencias desarrollarías con el campo temático de la imagen? ¿Qué escenario utilizarías para planificar situaciones de cantidad? ¿A qué actores de la comunidad incluirías en tu planificación curricular? Desde tu experiencia, ¿qué estrategias didácticas considerarías en tu planificación para que los estudiantes se empoderen del campo temático mostrado en la imagen?

2. Reflexión teórica 2.1 ¿Cómo se relaciona la competencia con el enfoque del área?. Actuar matemáticamente en situaciones de cantidad implica manipular objetos concretos utilizando nuestros sentidos; en otras palabras, es la acción de agregar, quitar, ordenar, repartir, fraccionar

38

objetos reales. El preámbulo del proceso de reflexión sobre la realidad es la actuación directa con elementos concretos, los que son representados como símbolos, conceptos, nociones en el cerebro; la combinación de esos conceptos pertenecientes a la realidad son lo que llamamos pensar matemáticamente. A partir de lo anterior, podemos manifestar que cuando planteamos situaciones problemáticas del contexto real relacionado con número y operaciones, estamos actuando y pensando matemáticamente en situaciones de cantidad. En nuestra sociedad actual, la utilidad que tienen los números y datos es prácticamente infinita. Ya que en las actividades cotidianas utilizamos medidas cuantitativas para comunicar situaciones, como las horas de trabajo diario, cantidad de cosechas al año, precios a los productos, cantidad de estudiantes que asisten diariamente a la escuela, número de ganados que tiene una familia, etc., por citar algunos ejemplos. Lo que da sentido al desarrollo de la competencia es la utilización de números y operaciones para resolver los problemas de su contexto real. Así pues, en el contexto rural, por ejemplo, los agricultores estiman presupuestos para ciertos productos; para ello, deben analizar el costo de producción, donde intervienen precios de semillas y nutrientes, el jornal de los trabajadores, entre otros, para poner el precio de venta de sus productos, que son los únicos ingresos económicos que generan. Y de manera implícita, sin rigor matemático, los ciudadanos, campesinos y comerciantes, resuelven situaciones problemáticas de compra y venta que se ajustan al modelo “precio venta = precio costo + ganancia”.

La competencia “Actúa y piensa matemáticamente en situaciones de cantidad” implica desarrollar modelos de solución numérica, comprendiendo el sentido numérico y de magnitud, la construcción del significado de las operaciones, así como la aplicación de diversas estrategias de cálculo y estimación al resolver un problema. (Minedu, 2015a, p. 19)

2.1.1

Historia de la mano y la cabeza: resolución de problemas como práctica social.

El ser humano, desde su aparición en la Tierra y por la necesidad de sobrevivencia, se ha dedicado a la caza, la pesca y la recolección de frutos; por ello, se dice que las manos fueron maestras de la cabeza, al haberlas utilizado para esas actividades por mucho tiempo. Así, con el paso de los años, las manos fueron adiestrándose y la cabeza, despejándose. Muchas teorías científicas fueron descubiertas producto de la experimentación, ya que la práctica social antecede a la teoría. Como un ejemplo concreto tenemos el principio de Arquímedes. “Cuenta la historia que mientras se bañaba en su tina descubrió la ley del empuje y salió desnudo por las calles de Siracusa gritando ¡Eureka!, ¡Eureka! (lo encontré)”. (Perero, 1994, p. 8). He aquí otro Si se desea empujar un objeto pesado con la mano empleando la fuerza, al no poder moverlo nuestro sentido común (nuestra cabeza) aconseja colocar una palanca. Esta solo puede ayudar a mover el objeto, mas ¿cómo subirlo a lo alto?, entonces nos enfrentamos a nueva situación problemática. La cabeza interviene de nuevo: crea el plano inclinado. Nos recomienda entonces colocar troncos redondos bajo el objeto, pues ¡hacer rodar es más fácil que arrastrar! De esta manera, nuestras necesidades cotidianas nos permite experimentar con las manos, y nuestra cabeza ayuda a nuestras manos, y ambos se complementan. En el campo pedagógico, podemos

39

citamos el siguiente Si pido al estudiante que diga el número de pasos que hay desde el aula hasta la dirección, no podrá calcular con exactitud; entonces, es necesario que el estudiante contraste contando los pasos.

2.1.2 Historia del sistema numérico en el Perú. Nuestros antecesores en la época del Imperio incaico utilizaron los números naturales. Una de las evidencias del uso del sistema de numeración decimal de la época es la existencia de términos en lengua quechua para nombrar los números naturales así como sus reglas de conformación, teniendo como base el número 10. El análisis de los vocablos numéricos nos permite caracterizar el sistema de numeración incaico como sistema decimal aditivo. Números 1 2 3 4 5 6 7 8 9 10

Quechua juk iskay kimsa tawa pishqa suqta qanchis pusaq isqun chunka

Números 11 12 13 14 15 16 17 18 19 20

Quechua chunka jukniyuq chunka iskayniyuq chunka kimsayuq chunka tawayuq chunka pisqayuq chunka suqtayuq chunka qanchisniyuq chunka pusaqniyuq chunka isqunniyuq iskay chunka

Números 30 40 50 60 70 80 90 100 1000 1.000.000

Quechua kimsa chunka tawa chunka pisqa chunka suqta chunka qanchis chunka pusaq chunka isqun chunka pachak waranqa junu

Fuente: https://es.wikipedia.org/wiki/Matem%C3%A1tica_incaica

Sistema de numeración shawi Números 1

shawi a’nara

2

kahtu

3

kara

4

kahtapini

5

a´naterahpu

6

sahuta

7

chanchise

8

puhsa

9

iskun

10

chunka

Los números mayores que 10 podríamos deducirlos haciendo combinaciones. Chunka kahtu sería igual a doce (12).

2.1.2.1 Los quipus Los quipus formaron parte de un sistema nemotécnico basado en cuerdas anudadas, mediante las cuales se registraba todo tipo de información cuantitativa o cualitativa. Si bien una de sus funciones se relaciona con la matemática al ser un instrumento capaz de contabilizar, también eran utilizados para guardar información de noticias censales, de montos de productos y de subsistencias

40

conservadas en los depósitos estatales. “Si se trataba de resultados de operaciones matemáticas, solo se anulaban las realizadas anteriormente en los ábacos incas”. [CITATION Fun15 \l 10250 ].

Fuente: http://paulyjavier.blogspot.pe/2015/09/elquipu_30.html

Fuente: https://www.google.com.pe/search?q=los+quipus&source

2.1.2.2 La yupana La palabra yupana, derivada del quechua yupay (contar), se define comúnmente como un ábaco utilizado para realizar operaciones aritméticas, el cual se remonta a la época de los incas. El doctor Carlos Radicati, en su obra El sistema contable de los incas: yupana y quipu, señala que el estudio del tema comenzó en 1869 al descubrirse en Ecuador, provincia de Cuenca, un objeto semejante a la yupana, citada por Guamán Poma en 1613 y mencionada por el cura Juan Velasco en 1789. Posteriormente, surgieron otros hallazgos en las ruinas de Chan-Chan, en la sierra de Áncash y en la provincia de Pisco. Estos hallazgos presentaban diferencias en diseño, material con que estaban construidos, tamaño, forma y disposición de los escaques o cuadrículas en bajo relieve. Hechas de piedra, arcilla, madera o hueso, decoradas a veces o no, con motivos humanos y de animales que podrían indicar que provenían de lugares tan diversos como sus formas. Reflexión En los sistemas de numeración quechua y shawi, observamos los sistemas decimales y aditivos. a) ¿Por qué es importante revalorar su uso en los primeros grados? b) ¿Cómo se podrán resolver problemas utilizando la yupana en un idioma originario?

2.2 ¿Cómo desarrollamos capacidades en la competencia de situaciones de cantidad?. Esta competencia se desarrolla a través de las cuatro capacidades matemáticas, las que se interrelacionan para manifestar formas de actuar y pensar matemáticamente, en situaciones de contexto real del estudiante. “Esto involucra el desarrollo de modelos matemáticos de solución numérica, empleando operaciones aritméticas, así como la utilización de diversas estrategias de cálculo y estimación al resolver un problema”. (Minedu, 2015). 41

2.2.1 Capacidad 1. ¿Qué significa “matematizar situaciones” en esta competencia? Matematizar implica expresar problemas de contexto real, en diversos modelos matemáticos relacionados con número y operaciones. Las situaciones problemáticas pueden ser del ámbito familiar, económico, social o científico. El modelo implica identificar datos, usar el modelo en otras situaciones y evaluar para contrastar la validez.

1 huairuro 2 huairuros …. 8 huairuros

En total tenemos 23 semillas.

1 ojo de vaca 2 ojos de vaca 3…

Las actividades asociadas a estar en contacto directo con situaciones problemáticas reales caracterizan más la capacidad de matematización.

La capacidad “Matematiza situaciones” implica expresar un problema, reconocido en una situación, en un modelo matemático. En su desarrollo se usa, interpreta y evalúa el modelo matemático, de acuerdo a la situación que le dio origen. (Minedu, 2015)

2.2.2 Capacidad 2. ¿Qué significa “comunica y representa ideas matemáticas” en esta competencia?. Esta capacidad implica comprender las ideas matemáticas y expresarlas en diversas formas de representación vivencial, con material concreto, pictórico, gráfico y simbólico; de modo que el uso de expresiones y símbolos matemáticos constituyen formas de lenguaje matemático.

42

Representación

pictórica

=

Representación gráfica

Material concreto

Vivencial

D(8)

1

2

4

8

D(6)

1

2

3

6

Representación simbólica

La capacidad “comunica y representa idea matemáticas” es la capacidad de comprender ideas matemáticas, y expresarlas en forma oral y escrita usando el lenguaje matemático y diversas formas de representación con material concreto, gráfico, tablas, símbolos y recursos TIC transitando así de una representación a otra. (Minedu, 2015)

2.2.3 Capacidad 3. ¿Qué significa “elabora y usa estrategias”? Ello implica:

 Elaborar y diseñar un plan de solución.  Seleccionar y aplicar procedimientos y estrategias de diverso tipo, que pueden ser heurísticos, de cálculo mental o escrito.

 Valorar las estrategias, procedimientos y recursos disponibles.

Es la capacidad de planificar, ejecutar y valorar una secuencia organizada de estrategias y diversos recursos (entre ellos, las tecnologías de información y comunicación), empleándolos de manera flexible y eficaz en el planteamiento y resolución de problemas, incluidos los matemáticos. Esto implica ser capaz de elaborar un plan de solución y monitorear su ejecución, pudiendo incluso reformular el plan en el mismo proceso con la finalidad de llegar a la meta. (Minedu, 2015)

43

Para valorar las estrategias, procedimientos y recursos disponibles, y comprender el significado de divisores de 12, se pide a los estudiantes que junten semillas ojo de vaca, cashapa y huairuros.

Luego se plantea la siguiente situación: ¿De cuántas maneras diferentes se pueden formar filas con 12 semillas de huairuros? Se espera que los estudiantes realicen esta actividad: 1.a forma

1 de 12

1 x 12

2.a forma

2 de 6

2x6

3.a forma

3 de 4

3x4

Podemos observar que los factores de cada forma son en total 6. Luego:

D(12) ={ 1, 2 , 3 , 4 , 6 , 12 }

2.2.4 Capacidad 4. ¿Qué significa “razona y argumenta generando ideas matemáticas”? Es la capacidad de plantear supuestos, conjeturase hipótesis de implicancia matemática mediante diversas formas de razonamiento (deductivo, inductivo), así como el verificarlos y validarlos usando argumentos. Esto implica partir de la exploración de situaciones vinculadas a la matemática para establecer relaciones entre ideas, establecer conclusiones a partir de inferencias y deducciones que permitan generar nuevas conexiones e ideas matemáticas. (Minedu, 2015)

La capacidad de razonar y argumentar generando ideas matemáticas implica que el estudiante:

   

Explique sus argumentos al plantear supuestos, conjeturas e hipótesis. Observe los fenómenos y establezca diferentes relaciones matemáticas. Elabore conclusiones a partir de sus experiencias. Defienda sus argumentos y refute otros, sobre la base de sus conclusiones.

44

Del ejemplo anterior, podemos deducir:

D(8 )={ 1,2,4,8 } D(9)= {1,3,9 } D (6 )= {1,2,3,6 } Los cuales se pueden verificar haciendo la tabla anterior para cada número. Analiza el siguiente cuadro:

Sí es matematizar

No es matematizar

Reconocer características, datos y condiciones de situaciones que permitan reproducir el comportamiento de la realidad. Utilizar modelos usando conexiones con nuevas situaciones en las que puedan ser aplicables.

Juntar semillas, piedritas y tapitas.

Contrastar y verificar la validez del modelo seleccionado en relación con una nueva situación. Expresar problemas diversos en modelos matemáticos.

El juego libre sin reglas.

Hacer gráficos y tablas de doble entrada en un papel.

Expresar el significado de los números y operaciones en forma oral y escrita.

Reflexión: a) Mencione otros ejemplos relacionados con cantidad, que implique la movilización de la capacidad de matematización. b) ¿Por qué consideras que los estudiantes no pueden explicar, de manera verbal, los procedimientos seguidos al resolver problemas? c) ¿Consideras que es mejor proponer problemas a los estudiantes y explicarles los procedimientos a seguir o que ellos deban crear y resolver los problemas?. Fundamenta su respuesta.

2.3 ¿Cuáles son las orientaciones didácticas que emplearías para el desarrollo de la competencia?. Consideramos que es importante iniciar las actividades con situaciones de la vida real, situaciones de contexto, para que el estudiante no tenga temor e interactúe con naturalidad; además, debe utilizar materiales concretos para realizar actividades matemáticas, y que estas sean de su interés y respondan a su realidad. La experiencia nos demuestra que el éxito del enfoque se da a partir de las orientaciones que hace el docente para que los estudiantes puedan elaborar, crear, diseñar y construir sus propios problemas a partir de su contexto, tomando en cuenta los actores y los escenarios de su realidad, para luego resolver los problemas propuestos. Todo ello es significativo para el estudiante y el aprendizaje es de larga duración. En el ejemplo anterior, se les presentó a los estudiantes semillas de la región amazónica, tales como huairuros, ojos de vaca y cashapas, para que puedan interactuar, clasificando, haciendo el conteo,

45

etc. Luego el docente presentó la situación para que puedan hacer una lista sistemática de la siguiente manera: Si se tienen 12 huairuros, ¿de cuántas maneras diferentes se pueden formar filas de huairuros? Los estudiantes manipulan, discuten entre sus compañeros y, al llegar 3 filas de 4 y 4 filas de 3, generan discusión entre ellos mismos, aclarando la situación de la propiedad conmutativa de la multiplicación; asimismo, ponen en juego los conocimientos previos de la multiplicación y división. De esta manera, los estudiantes van construyendo sus conocimientos matemáticos partiendo desde un nivel concreto hasta un nivel abstracto, al escribir los divisores de 12, y con facilidad pueden generalizar para cualquier número; además, estarían consolidando su aprendizaje sobre divisores. Asimismo, da coherencia al enfoque, ya que se están empleando materiales de la zona amazónica, los cuales, de igual manera, se reemplazan en las zonas andinas con los granos de maíz y/o frijoles. A esto se llama contextualizar y usar recursos y materiales de su contexto.

Reflexión a) ¿Qué otros materiales de tu contexto puedes emplear para desarrollar la competencia “Actúa y piensa matemáticamente en situaciones de cantidad”? b) ¿Qué estrategias didácticas empleas para el desarrollo de la competencia?

2.4 ¿Cómo debemos evaluar la competencia?. En un enfoque por competencias, también la evaluación tiene que ser necesariamente por competencias. En tal sentido, es necesario definir previamente ¿cómo se evidencia la competencia? Parece que hay un gran número de interpretaciones. “Las competencias se evidencian en el desempeño, la actuación del sujeto que involucra todo el ser: lo que sabe, lo que hace y los valores que subyacen a las actitudes que se manifiestan en las conductas. La competencia y el contexto están íntimamente relacionados”. (Pimienta, 2015) Una de las formas de clasificar la evaluación por sus agentes considera la autoevaluación y la coevaluación. Mediante la autoevaluación, los sujetos pueden reflexionar y tomar conciencia acerca de sus propios aprendizajes y de los factores que intervienen, valorando su desempeño con responsabilidad. La coevaluación se realiza entre pares, con la finalidad de controlar la participación activa de los miembros del equipo de trabajo; sin embargo, para que tenga éxito, es importante resaltar los aspectos positivos de la actitud. Caso contrario, la coevaluación puede convertirse en rechazo a la cultura de evaluación que tanto aspiramos. Ficha de coevaluación Esta ficha tiene por finalidad recoger información sobre la participacion de cada miembro del equipo de trabajo.

46

Nombre del equipo ……………………………………………….. Nombre de integrantes

Fortalezas

Aspectos que puede mejorar

Ficha de autoevaluación Estimado docente, marca con un aspa los niveles alcanzados según los descripotores propuestos. Nombres y apellidos del participante: ……………………………………………………………………………………. BUEN O

DESCRIPTOR

ADECUADO

PUEDO MEJORAR

Participo activamente en el desarrollo de actividaes de la unidad. Utilizo recursos y materiales de mi contexto en la planificación y ejecución de mis sesiones de aprendizaje. Considero mapas de progreso y rutas del aprendizaje en la planificación curricular. Articulo, en mis actividaes curriculares, actores y escenarios del contexto donde laboro.

Reflexión a) ¿Por qué es importante la autoevaluación y coevaluación en los docentes participantes? b) ¿Qué aspectos de la autoevaluación consideras que puedes mejorar?

2.5

¿Cuáles son los estándares e indicadores nacionales de evaluación?.

Los estándares de aprendizaje nacionales son expectativas de aprendizaje que deben ser alcanzadas por todos los estudiantes para que puedan desenvolverse eficientemente y en igualdad de condiciones, en los distintos ámbitos de su vida. Los estándares de aprendizaje nacionales del Perú son los Mapas de Progreso del Aprendizaje. (IPEBA, 2012). Los mapas de progreso describen la secuencia típica en que progresa el aprendizaje en las competencias que se consideran fundamentales en las distintas áreas curriculares, a lo largo de la trayectoria escolar.

Niveles de progresión VI CICLO 1.° y 2.° de Secundaria  Representa cantidades discretas o continuas mediante números enteros y racionales en su expresión fraccionaria y decimal en diversas situaciones.

47

 Compara y establece equivalencias entre números enteros, racionales y porcentajes; relaciona los órdenes del sistema de numeración decimal con potencias de base diez. Selecciona unidades convencionales e instrumentos apropiados para describir y comparar la masa de objetos en toneladas o la duración de un evento en décadas y siglos.  Resuelve y formula situaciones problemáticas de diversos contextos referidas a determinar cuántas veces una cantidad contiene o está contenida en otra, determina aumentos o descuentos porcentuales sucesivos, relaciona magnitudes directa o inversamente proporcionales; empleando diversas estrategias y explicando por qué las usó.  Relaciona la potenciación y radicación como procesos inversos. Ejemplo adecuado de IPEBA En este ejemplo, se han identificado los siguientes actores, escenarios y recursos y/o materiales. Actores identificados: Como actor local, el comerciante de Chupaca, ya que participó en el proceso de aprendizaje. Escenarios del proceso: Mercado donde se realiza la feria dominical. Recursos: Billetes y monedas que sirven para representar la situación.

Actividad Relación entre tema de numeración decimal y sistema monetario Situación Al finalizar la feria dominical en la ciudad de Huancayo, un comerciante de Chupaca cuenta sus monedas y billetes según se muestra en la tabla. El docente pide a los estudiantes: a) Identificar la cantidad de billetes y monedas, así como las unidades del sistema de numeración decimal. b) Expresar su valor numérico utilizando las potencias de base diez. c) Averiguar la relación completando las equivalencias entre diferentes unidades. d) Completar la tabla según el ejemplo dado:

Denominación

N.° de monedas

a×10n

87

87×10

−2

Unidades del SND

Equivalente en (S/)

céntimos

0,87

décimos 219

93

48

45 x 101

45

450

5700,00 57

TOTAL EN SOLES

6265,77 FUENTE. Mapas de Progreso, IPEBA, 2012

VII CICLO 3.°, 4.° y 5.° de Secundaria    

Interpreta el número irracional como un decimal infinito y sin periodo. Argumenta por qué los números racionales pueden expresarse como el cociente de dos enteros. Interpreta y representa cantidades y magnitudes mediante la notación científica. Registra medidas en magnitudes de masa, tiempo y temperatura, según distintos niveles de exactitud requeridos, y distingue cuándo es apropiado realizar una medición estimada o una exacta.  Resuelve y formula situaciones problemáticas de diversos contextos referidas a determinar tasas de interés, relacionar hasta tres magnitudes proporcionales, empleando diversas estrategias y explicando por qué las usó.  Relaciona diferentes fuentes de información. Ejemplo 2 adecuado de IPEBA Actividad 2 Relación entre el sistema de numeración decimal y sistema monetario Situación problemática Un padre de familia desea repartir un terreno de forma rectangular entre sus tres hijos, de manera que, al dividir el lado mayor del terreno en tres partes iguales, cada uno de los lotes generados sea proporcional al terreno original:

a) Determinar la medida de los lados que podría tener el terreno rectangular ABCD.

49

b) ¿Cuál es la razón de proporcionalidad entre los lados del terreno rectangular ABCD? ¿A qué conjunto numérico pertenece? Explica.

2.6

¿Cuáles son los recursos y materiales que permiten el desarrollo de la competencia?

Recursos y materiales El enfoque tiene relación con la etnomatemática. En tal sentido, los recursos y materiales a utilizar con fines pedagógicos son los que tenemos al alcance. Por ejemplo, los huairuros, cashapas y ojos de vaca son semillas de la Amazonía de nuestro país; asimismo, en las zonas andinas, los recursos y materiales son aquellos que la naturaleza provee, los que son utilizados por los docentes para el trabajo pedagógico, didáctico propiamente. También los textos distribuidos por el Ministerio de Educación para los docentes y estudiantes forman parte del recurso, así como los materiales elaborados por los mismos docentes, como separatas, diapositivas, material estructurado y no estructurado, entre otros. Escenarios de aprendizaje Los escenarios más directos son el aula y, en general, la escuela; sin embargo, las actividades pedagógicas trascienden el ámbito escolar y tienen lugar en otros espacios, como son las postas médicas o centros médicos, la comisaría, el mercado, la tienda, el campo de trabajo, las granjas, entre otros, que permiten interactuar y despertar esa curiosidad de indagación, desarrollando las capacidades comunicativas de los estudiantes y el hábito de la investigación. Estos escenarios son valiosos para el desarrollo de capacidades matemáticas y ayudan al proceso de construcción de aprendizajes, significativos y reales. De esta manera, los docentes contextualizan situaciones y descubren junto con los estudiantes situaciones problemáticas, dando sentido y coherencia a la propuesta de la matemática centrada en la resolución de problemas.

Fuente:http://www.fao.org/agronoticias/ag ro-noticias/detalle/en/c/232365/

Fuente:http://www.machupicchu-toursperu.com/blog/sierra-soberbio-reinoandes

Fuente:http://sanmiguelcajamarca.blogspot. pe/2013_06_01_archive.html

Actores que intervienen Los actores principales son los estudiantes y docentes, porque cumplen un rol muy importente y determinante en el proceso de construccion de aprendizajes. También los padres de familia, comuneros, autoridades locales y funcionarios, entre otros. El desarrollo de las capacidades de razonar y argumentar radica, principalmente, en hacer la visita y recoger información relevante para la contrucción de aprendizajes.

50

3. Herramientas para la nueva práctica 3.1 Actividad de reflexión individual/grupal a)

b) c) d)

¿Qué enfoque pedagógico emplearás, de ahora en adelante, en tu nueva práctica pedagógica para el desarrollo de la competencia “Actúa y piensa matemáticamente en situaciones de cantidad”?. ¿cómo lo utilizarás? ¿Qué capacidad o capacidades crees que se deben priorizar para el desarrollo de la competencia? Argumenta tu respuesta. ¿Cómo articularás el calendario comunal y el aporte de los sabios de la comunidad en el desarrollo de las competencias? ¿Cómo será la evaluación por competencias en su nueva práctica y qué instrumentos empleará?

3.2 Actividades a distancia A continuación, se presenta una situación problemática, la cual será ejecuta en el componente a distancia. Mario y Marco acuerdan ahorrar S/. 4200 soles cada uno, durante cuatro años, en una institución financiera que les ofrece el 8 % de interés anual. Además, les da la opción de retirar los intereses al final de cada año. Mario optó por retirar sus intereses al final de cada año. Marco pensó en dejarlos para que sus intereses sean mayores cada año. ¿Cuánto de interés habrán producido ambos depósitos al cabo de cuatro años?. ¿Quién tomó la mejor decisión? ¿Qué diferencia hay entre los intereses de Mario y Marco?. A partir de la situación presentada, completa información en la tabla para el desarrollo de una sesión de aprendizaje. Competencia

Capacidad

Instrumento de evaluación

Indicadores

Recursos Materiales (actores escenarios)

3.3 Actividades metacognitivas. a) ¿Qué lecciones aprendidas puedes señalar en relación con la práctica pedagógica presentada en la imagen del inicio de esta unidad? b) ¿Qué aspectos de la unidad te han servido para mejorar tu práctica pedagógica?. c) ¿En qué medida la reflexión desde la práctica pedagógica y la reflexión teórica influirán en la mejora de tu desempeño? d) ¿Las actividades para la nueva práctica te servirán para mejorar tus actividades pedagógicas? e) ¿Cómo fue tu participación durante el desarrollo de la unidad?. ¿Cuál es tu compromiso?

51

3.4

Autoevaluación.

Indicaciones Estimado docente, evalúa tu participación durante el taller, marcando con un aspa según sea el caso. Apellidos y nombres: …………………………………………………………………………………………………………………… N.o 1 2 3 4 5

Descriptores Participé activamente en las actividades indicadas durante el desarrollo de la sesión. Escuché con atención las indicaciones y respeté las normas de trabajo señaladas. Cumplí con el tiempo previsto con las actividades señaladas. Respeté a mis colegas y contribuí en las actividades grupales. Me interesé en el contenido y aclaré mis dudas.



No

4. Glosario Cantidad. Capacidad matemática relacionada con el uso de números y operaciones para resolver problemas. Operaciones. Uso de algoritmos en el proceso de resolución de problemas. Actores. Son sujetos que intervienen en una acción o suceso de aprendizaje. Escenarios de aprendizaje. Son los espacios que contribuyen al desarrollo del aprendizaje. Recursos de aprendizaje. Son el conjunto de procedimientos y estrategias que el estudiante debe poner en funcionamiento cuando se enfrenta con una tarea de aprendizaje. Niveles de progresión. Indican los aprendizajes que van logrado los estudiantes al concluir cada ciclo, según los mapas de progreso. Estrategia didáctica. Conjunto de situaciones, actividades y experiencias a partir del cual el docente traza el recorrido pedagógico que necesariamente deberán transitar sus estudiantes junto con él para construir y reconstruir el propio conocimiento, ajustándolo a las demandas socioculturales del contexto.

5. Texto complementario De hablar de la educación intercultural a hacerla (…) El defecto principal de la pedagogía intercultural dominante, basada en la prédica ética y la eficiencia del maestro, reside en su concepción puramente verbal de la cultura y de los conocimientos indígenas. Revalorarlos equivale, según esa visión, a hablar de ellos, enunciarlos en el aula, expresarlos por escrito o en dibujos y representaciones teatrales, como si se tratara de imitar algo ajeno. La palabra objetiviza el pensamiento, toma distancia de la realidad y hace posible su evaluación crítica. Sin embargo, en una escuela imbuida de valores urbanos, con maestros que se atribuyen un papel “civilizador” frente al indígena, la evaluación crítica de los elementos culturales indígenas mediante una práctica pedagógica exclusivamente discursiva resultará siempre en desmedro de los valores socioculturales indígenas. Desde luego, incitar a los maestros a que hablen de la cultura indígena en la escuela no garantiza de ninguna manera un efecto revalorado. Una educación intercultural que se limita a incluir en su currículo un tratamiento de contenidos culturales

52

indígenas de manera meramente verbal o escrita –como “temas motivadores” enunciados por el maestro e implementados en el aula con ejercicios participativos de los alumnos– solo persigue la ilusión de una revaloración cultural y no alcanza su objetivo real. (...) Gasche, Jorge. “De hablar de la educación intercultural a hacerla”. Mundo amazónico 1, 2010 / 111 – 134

6. Referencias IPEBA. (2012). Mapa de Progreso del Aprendizaje. Lima: Ministerio de Educación. Mariano, P. (1994). Historia e Historias de Matemáticas.México: Iberoamérica S. A. Ministerio de Educación. (2012). Módulo de Resolución de problemas. Lima: Autor. Ministerio de Educación. (2013a). Rutas del Aprendizaje Fascículo N.° 1. Lima: Autor. Ministerio de Educación. (2013 b). Rutas del Aprendizaje Fascículo N° 2. Lima: Autor. Ministerio de Educación. (2013 c). Matemáticas en Educación Intercultural Bilinguie, Orientaciones pedagógicas. Lima: Autor. Ministerio de Educación. (2015). Rutas del Aprendizaje Versión 2015. Lima: Autor.

Wikipedia, (2015). https//es.wikipedia.org/wiki/Matem%C3%A1tica_incaica#Sistema_de_numeraci. C3.B3n_de_los_incas. Recuperado el 8 de diciembre de 2015, de: https//es.wikipedia.org Vigotsky, L. S. (1979). El desarrollo de los porcesos psicológicos superiores. Barcelona: Critica S.L. Academia.edu. (2016). Investigación Científica. Recuperado el 12 de noviembre de 2015, de: http://www.academia.edu/6907195/INVESTIGACI%C3%93N_CIENT%C3%8DFICA.

53

UNIDAD II Sistemas de numeración

PRESENTACIÓN Estimados participantes, en esta oportunidad te presentamos la unidad II, que parte del análisis de casos para promover la reflexión desde la práctica docente y formula preguntas que permitirán contrastar con la teoría de los sistemas numéricos. La estrategia didáctica central de la unidad está basada en la construcción de los conocimientos numéricos a partir de situaciones problemáticas de contexto real, que permitirá al docente mejorar su desempeño en la didáctica para construir una matemática funcional. En esta dirección, se construye el significado y uso de operaciones aditivas y multiplicativas en N, Z, Q, planteando y resolviendo problemas de sistemas numéricos. Después de reflexionar sobre su propia práctica docente y contrastar con la teoría, se proponen actividades para la nueva práctica, donde los maestros promueven nuevas formas de construcción teórica, formulando y resolviendo problemas a partir de la interacción con la naturaleza, situaciones problemáticas de contexto real, etc.

ESQUEMA DE LOS CONTENIDOS

54

Reflexión desde la práctica

SISTEM SISTEMAS AS DE DE NUM NUMERACIÓN ERACIÓN

Reflexión teórica

Sistema de los números enteros Z

Sistema de los números racionales Q

Orientaciones didácticas

Evaluación de los aprendizajes

Herramientas para la nueva práctica

Actividades de reflexión individual/equipo, a distancia, de metacognición y autoevaluación

Glosario, texto complementario, referencias

1. Reflexión desde la práctica Observa las siguientes viñetas:

Aprenderemos sistemas numéricos.

Para resolver problemas, deben aprender propiedades.

¿Qué es sistema?

No comprendo nada, profesor.

No creo que pueda soportar 2 horas de clase.

Al término de la clase, les tomaré una 55 práctica.

Hay problemas tipo muy interesantes que se

por favor.

Me prestas tu cuaderno, ¿sí?

¿Cómo haré la tarea si no entendí?

56

A partir de la observación de las viñetas, responde las siguientes preguntas: a) b)

c) d) e) f)

¿Cómo se definen los sistemas numéricos de los números naturales, enteros y racionales en la primera viñeta? ¿La estrategia empleada por el docente, según la situación presentada, crees que es pertinente para desarrollar el significado de números naturales, enteros y racionales? Fundamenta tu respuesta. ¿Cómo consideras que está evaluando el docente en la segunda viñeta?, ¿evalúa competencias?, ¿por qué? ¿Cómo defines los sistemas numéricos de los números naturales, enteros y racionales? ¿Qué estrategias empleas con tus estudiantes para el proceso de construcción de los sistemas numéricos de los números naturales, enteros y racionales? ¿Qué técnicas e instrumentos de evaluación utilizas para evaluar los sistemas numéricos?

2. Reflexión teórica

2.1. Fundamento teórico Desde la aparición del ser humano, la técnica de contar nace como necesidad social de comunicarse, de expresar la cantidad. Para ello, la humanidad empleó diversas formas de representar las cantidades. Estas técnicas han dado origen al concepto de número y a la aritmética. Surgen ligadas a la necesidad de:  Comunicar información referente al tamaño (la numerosidad) de las colecciones de objetos (cardinal de la colección).  Indicar el lugar que ocupa o debe ocupar un objeto dentro de una colección ordenada de objetos (ordinal del objeto). En las sociedades prehistóricas –de cazadores y recolectores– se plantea ya, aunque a pequeña escala, la necesidad de responder a la pregunta ¿cuántos hay? o ¿cuántos son? También aparece la necesidad de establecer un orden de actuación: ¿qué se hace primero?, ¿quién interviene en segundo lugar?, etc. “A partir de esas necesidades sociales se desarrollan diferentes técnicas de recuento que han ido evolucionando a lo largo de la historia” (Godino, 2003, p. 15). En nuestra sociedad, se utiliza predominantemente una técnica de recuento con palabras, aun cuando se conservan vestigios de otras varias técnicas. En este proceso, aparecen nuevos conceptos, como ordinales y cardinales, para dar inicio a las operaciones de adición. La aparición en el neolítico de sociedades estatales y de su entramado administrativo conlleva a plantear las necesidades de:  Obtener el cardinal de colecciones formadas por muchos objetos (colecciones muy numerosas).  Recordar los cardinales correspondientes a muchas colecciones. Como ejemplos de algunos sistemas decimales de numeración escritos, tenemos:

57

Sistema egipcio

Sistema chino

Sistema hindú

Sistema incaico

Tomado de

“Sistemas numéricos y

Las actividades vivenciales del entorno Son muy importantes (...), ya que está asociado al contacto directo con situaciones problemáticas reales, en ellas los estudiantes interpretan la realidad haciendo uso de conceptos y procedimientos matemáticos para resolver la situación planteada. (Minedu, 2013)

2.1.1

¿Cómo surgieron los números enteros?

Los números negativos, antiguamente conocidos como “números deudos” o “números absurdos”, datan de una época donde el interés central era convivir con los problemas cotidianos de la naturaleza. Las primeras manifestaciones de su uso se remontan al siglo V, en Oriente, y no llega hasta Occidente sino hasta el siglo XVI. En Oriente se manipulaban números positivos y negativos; estrictamente, se utilizaban los ábacos, usando tablillas o bolas de diferentes colores.

58

Sin embargo, los chinos no aceptaron la idea de que un número negativo pudiera ser solución de una ecuación. Corresponde a los indios la diferenciación entre números positivos y negativos, que interpretaban como créditos y débitos, respectivamente, distinguiéndolos simbólicamente. La notación muy difundida para los números positivos y negativos fue gracias a Stifel (1487-1567). Los símbolos germánicos (+) y (-) se popularizaron con este matemático alemán en el siglo XV. Antes de ello, se utilizaba la abreviatura de p para los positivos y m para los negativos. Hasta fines del siglo XVIII, los números negativos no eran aceptados universalmente. Gerolamo Cardano, en el siglo XVI, los llamaba “falsos”; pero en su Ars Magna (1545) los estudió exhaustivamente. John Wallis (1616-1703), en su Aritmética Infinito (1655), “demuestra” la imposibilidad de su existencia diciendo que “esos entes tendrían que ser a la vez mayores que el infinito y menores que cero”. 2.1.2 ¿Cómo surgieron los números racionales? Los babilónicos utilizaban fracciones cuyo denominador era una potencia de 60; mientras que los egipcios usaron, sobre todo, las fracciones con numerador igual a 1. En la escritura, la fracción la expresaban con un óvalo, que significaba parte o partido, y debajo, o al lado, colocaban el denominador; el numerador no se ponía por ser siempre 1. Los griegos y romanos usaron también las fracciones unitarias, cuya utilización persistió hasta la época medieval. En el siglo XIII, Leonardo de Pisa, llamado Fibonacci, famoso, entre otras cosas, por la serie de Fibonacci, introdujo en Europa la barra horizontal para separar numerador y denominador en las fracciones. A principios del siglo XV, el árabe Al Kashi fue quien generalizó el uso de los números decimales tal y como los conocemos hoy. A finales del siglo XVI, Simón Stevin desarrolló y divulgó las fracciones decimales que se expresaban por medio de números decimales: décimas, centésimas, milésimas, etc., pero los escribía de una forma complicada. Así para 456,765, escribía 456 (0) 7(1) 6(2) 5(3). A principios del siglo XVII, los números decimales ya aparecieron tal y como los escribimos hoy, separando con un punto o una coma la parte entera de la parte decimal. Los números decimales se impusieron, en casi todos los países, al adoptarse el sistema métrico decimal, en el siglo XVIII, concretamente, en 1792.

2.1.3 Conceptos y procedimientos

Definición 1: Se llama sistema de los números naturales a un conjunto N= { a , b , c , … } provisto de dos operaciones totalmente definidas: Adición: N × N → N ( a , b ) → a+b Multiplicación: N × N → N ( a , b ) → a . b≡ a ×b Y una relación de orden llamada menor o igual: a ≤ b; que satisfacen los axiomas que se presentan a continuación.

Axiomas de la adición 1. (a + b) + c = a + (b + c); (asociatividad) 2. a + b = b + a; (conmutatividad)

59

3. Existe un único número natural llamado cero o elemento neutro de la adición, denotado por 0; tal que a + 0 = 0 + a = a; para todo a ϵ N. 4. a + c = b + c; implica que a = b; (cancelación) Axiomas de la multiplicación 1. (a  b)  c = a  (b  c) (asociatividad) 2. a  b = b  a (conmutatividad) 3. Existe un único número natural diferente de cero llamado uno o elemento neutro de la multiplicación, denotado por 1; tal que a  1 = 1  a = a; para todo aϵ N. 4. a  c = c  a y c  0 implica que a = b (cancelación) 5. a(b + c) = a  b + a  c (distributividad)

Definición 2: Se dice que a es menor o igual que b o que b es mayor o igual que a y se escribe a ≤ b ó b ≥ a si existe un número natural c tal que: a + c = b. En caso contrario, se dice que a no es menor o igual que b; o que b no es mayor o igual que a.

Axioma para la relación menor o igual Si A ϵ N y A≠ ∅ , entonces existe un único elemento m ϵ A; tal que m ≤ a para todo a ϵ A. El elemento m recibe el nombre de mínimo del conjunto A. El axioma anterior es conocido como el principio de la buena ordenación.

Sistema de los números enteros Para comprender los números enteros, analizamos las temperaturas bajo cero, sobre cero, alturas sobre el nivel del mar, profundidad bajo el nivel del mar, al nivel del mar, ingresos, egresos, entre otros. Nos dan la idea de situaciones opuestas, cuyos significados podemos representar simbólicamente con los números enteros. A partir del contexto, podemos utilizar números con signos positivos, negativos, o cero, según sea el caso.

Históricamente, durante mucho tiempo fueron rechazados por creer que "no existían" y no fue sino hasta el siglo XVII que tuvieron aceptación en trabajos científicos. El hecho de que un número sea entero significa que no tiene parte decimal. La representación en la recta es la siguiente:

…. -∞

-3

-2

-1

0

60

1

2

3

…. ∞

Z={−∞ ...−5 ,−4 ,−3 ,−2 ,−1, 0 ,+1 ,+2 ,+3 ,+4 ,+5 , …+∞ } Las ganancias nos inducen a un solo resultado de signo positivo, mientras que todas las pérdidas también nos llevan a un solo resultado, pero de signo negativo. Es decir, si tenemos dos negativos, el resultado de la suma seguirá siendo negativo. Pero si sumamos dos cantidades de signos contrarios, el resultado será una cantidad que llevará el signo del número de mayor valor. En otras palabras, si mis pérdidas superan a mis ganancias, eso quiere decir que aún estoy perdiendo. ¿Menos por menos es más? Como dijimos, Jhon Wallis (1616-1703) dice demostrar la imposibilidad de la existencia de los números negativos afirmando que “estos entes tendrán que ser a la vez mayores que el infinito y menores que cero”. Leonhard Euler es el primero en darles estatuto legal. En 1770, trata de demostrar que (-1)(-1) = +1. Argumenta que el producto tiene que ser +1 ó -1 y que, sabiendo que se cumple (1)(-1) = -1, tendrá que ser (-1)(-1)= +1. Es difícil encontrar una respuesta sencilla y convincente, ya que la regla es puramente arbitraria y se adopta solo para que no aparezcan contradicciones; pero existen varias justificaciones claras aceptables: El equivalente lingüístico de la doble negativa equivale a una afirmación: No es cierto que Pepito no tenga libro <==> Pepito tiene libro. Un ejemplo fácil de visualizar es el de la isla de Barataria, donde hay ciudadanos “buenos” a los que se les asigna (+) y ciudadanos “malos” a los que se asigna (-). También se acuerda que: “salir” de la isla equivale al signo (-) y “entrar” a la isla equivale al signo (+). Si un ciudadano bueno (+) entra (+) a Barataria, el resultado para la isla es positivo: (+)(+) = + Si un ciudadano malo (-) sale (-) de Barataria, el resultado para la isla es positivo: (-)(-) = + Si un ciudadano bueno (+) sale (-) de Barataria, el resultado para la isla es negativo: (+)(-) = (-) Si un ciudadano malo (-) entra (+) a Barataria, el resultado para la isla es negativo: (-)(+) = (-)

Ciudadano bueno (+) Ciudadano malo (-)

Entra a la isla (+)

Sale de la isla (-)

+

-

-

+

Fracciones En la vida diaria, nos enfrentamos a situaciones que tienen que ver con fracciones en la mayoría de los casos. Fracciones en sus diversas formas, como medida de área, decimal y porcentaje. Sistema de números racionales En la vida real, no es suficiente realizar operaciones con los números enteros, ya que hay situaciones, como lo descrito en el caso anterior, que no nos permite hacer las divisiones de manera que sean Fracción como relación parte-todo exactas. Por ello, hay la necesidad de extender el sistema de números enteros a otro más amplio que se conoce como el sistema de números racionales Q, cuya representación está dada en fracciones de la forma

a b

61

En la representación parte-todo, se toma unas barras de chocolate de cacao producido de manera artesanal en la selva peruana, de modo que las barras son iguales y tienen divisiones para facilitar las particiones. Según el gráfico, ¿qué parte de chocolate se piensa partir en cada caso?

1/4

1/4 3/4

1/4

Según los estudios de Piaget, la noción de fracción desde la relación parte-todo se fundamenta en siete atributos básicos:

      

Un todo está formado por elementos separables. La unidad “todo” se puede dividir en un número de “partes” determinado. La reunión de todas las “partes” forman la unidad “todo”. El número de partes no es igual al número de cortes. Las partes iguales deben ser congruentes. Las partes se pueden considerar, a su vez, como unidad “todo”. La unidad “todo” se conserva. En la forma inicial de abordar las fracciones como parte-todo, según Piaget, Inhelder y Szeminska, se tiene como unidad una forma continua; pero también se puede tener como un “todo” una unidad discreta, por ejemplo, un grupo de 12 canicas, de 30 palillos, de 40 estudiantes. Fracción como razón, decimal y porcentaje La generalidad de la interpretación de la fracción como razón consiste en comparar cantidades de magnitudes diferentes; mientras que en la interpretación parte-todo, en un contexto de medida, solo se permite comparar cantidades del mismo tipo. Ejemplo para dar claridad al concepto del significado de fracción como razón: este significado se usa comúnmente con la idea de formar proporciones y permite también desarrollar o integrar los conceptos de fracciones equivalentes, probabilidad y porcentajes. La relación entre los significados de la fracción como razón y como medida se explica por medio de los siguientes ejemplos:

 La estatura de Ana equivale a 2/3 la estatura de María. Establece equivalencias entre decimales, fracciones y porcentajes con soporte concreto, gráfico y simbólico. (IPEBA, 2013).

Representa la mitad de 6 pelotas (cantidad

Representa las tres cuartas partes de una

62

discreta)

torta (cantidad continua)

1 de 6=3 2 50 % de 6=3

Representación gráfica

3 =0,75=75 % 4

Representación simbólica

Representación gráfica

Representación simbólica

La fracción decimal es toda fracción que tiene por denominador potencias de 10:

3 9 7 6 , , , 10 10 10 10 Un décimo

Un centésimo

Un milésimo

1 10

1 100

1 1000

Reflexión a) ¿Por qué crees que es importante la historia de la matemática en el proceso de enseñanzaaprendizaje? b) ¿Qué estrategias didácticas empleas para el desarrollo de la competencia?

2.2

¿Cuáles son las orientaciones didácticas que empleas para el desarrollo de los aprendizajes?

A continuación, se realiza el desarrollo de una competencia a partir de situaciones problemáticas, utilizando orientaciones metodológicas, materiales, medios y verificando el logro de los aprendizajes por medio del uso de instrumentos. 2.2.1

Producción de papas en Coyllurqi

Competencia Actúa y piensa matemáticamente en situaciones de cantidad.

Capacidades Elabora y usa estrategias.

Indicadores Diseña y ejecuta un plan orientado a la investigación y resolución de problemas.

INICIO El docente da la bienvenida a los estudiantes y presenta a un poblador quien manifiesta sus dificultades en la agricultura.

Situación problemática

63

Del 2010 a la fecha no me ha ido tan bien, debido al empobrecimiento de las tierras agrícolas, la carencia de materias orgánicas, la helada que afecta en estaciones de verano y la excesiva lluvia que no favorece la buena producción. Una parte de mi siembra es para autoconsumo y el resto, para el mercado interno. El año 2010 la producción fue de 146 sacos, cuyo precio en el mercado fue de S/. 75,00 por saco. En el 2011, la producción anual disminuyó a 120 sacos; además, el precio se incrementó en S/. 5.00 por saco respecto al año anterior. El año 2012 la producción disminuyó en 40 sacos respecto al año anterior; mientras que el precio se incrementó en S/. 10,00 por saco respecto al año anterior. En el 2013, la producción llegó a 79 sacos con un precio de S/. 100,00 por saco. Finalmente, en el 2014, la producción apenas llegó a 30 sacos con un precio en el mercado de S/. 120,00. Este año ya no sembré papas porque que no me resultaba rentable. Ante toda esta situación, como agricultor tengo muchas interrogantes: ¿Qué haré para saber la producción de papas entre los años 2010 y 2014? ¿A cuánto ascendieron las ventas totales desde el 2010 al 2014? ¿Qué me recomendarían ustedes? A continuación, el docente debe formar equipos de trabajo para que los estudiantes se ayuden entre ellos. Así, cuando trabajan entre compañeros, hay mayor confianza y se ponen en práctica las normas de convivencia. Seguidamente, debe repartir a los equipos la situación problemática. Sugerimos realizar la motivación y el recojo de saberes previos relacionados con la situación problemática. La motivación se debe realizar para generar el interés de los estudiantes y la recepción de la información; mientras que el recojo de saberes previos se debe hacer para diagnosticar hasta dónde llega la zona de desarrollo próximo de los estudiantes. ¿Qué les pareció la visita al agricultor? ¿Cómo saber la variación de la producción de papa de un año a otro? A continuación, el docente, a través de una pregunta, debe generar conflicto cognitivo en los estudiantes. ¿Cómo organizaré los datos del problema para realizar los cálculos? En seguida, el docente dará a conocer el propósito de la sesión, que debe estar relacionado con el conocimiento que desea que logren los estudiantes y la forma como serán evaluados, entregando a cada grupo las fichas de evaluación que serán registradas por los grupos durante el proceso. Los estudiantes construirán el significado y uso de operaciones en N a partir de la situación problemática 1. Los estudiantes serán evaluados durante el proceso de resolución de la situación problemática a través de fichas de autoevaluación. DESARROLLO En esta parte, el docente debe sugerir a los estudiantes utilizar los cuatro pasos de Pólya para resolver la situación problemática; previamente, debe pegar en la pizarra los cuatro pasos.  Primer paso: Comprensión del problema El docente debe pedir a los estudiantes que lean detenidamente la situación problemática hasta familiarizarse.

64

Para verificar si los estudiantes comprendieron el problema, el docente debe hacer preguntas sobre identificación de datos, condiciones e interrogantes.

¿Qué datos se puede deducir de la situación problemática 1? Los estudiantes serán inducidos a responder: Tenemos: años, sacos y precios. ¿Qué condiciones tenemos? El docente debe promover en los estudiantes para que respondan de la siguiente forma: La producción de papa varía desfavorablemente años tras año. ¿Qué interrogante tenemos? Los estudiantes responderán: ¿Qué haré para saber la producción de papas entre los años 2010 y 2014? ¿A cuánto ascendió las ventas totales desde el 2010 al 2014? ¿Qué me recomendarían ustedes respecto a la producción de papas?  Segundo paso: Configurar un plan El docente debe pedir a los estudiantes que propongan estrategias tomando como punto de partida, los datos, las condiciones y las interrogantes, induciendo al descubrimiento de estrategias a través de las siguientes preguntas: ¿Alguna vez resolvieron un problema parecido?. ¿Observan alguna relación entre los datos identificados?. ¿Identifican alguna relación entre los datos y la incógnita?. Si los estudiantes aun no construyeron sus estrategias, el docente debe proporcionar a cada grupo un problema parecido pero de menor dificultad, en el cual deben identificar las estrategias utilizadas.

Los estudiantes descubrieron que pueden hacer una lista sistemática de los años, sacos y precios. Elaboran un cuadro de doble entrada para los datos. Realizan operaciones combinadas con los datos organizados en el cuadro. 

Tercer paso: Ejecutar el plan

Se debe pedir a los estudiantes que desarrollen cada una de las estrategias propuestas, teniendo en cuenta los datos, condiciones y preguntas. En este proceso, el docente debe estar atento para que todos los grupos trabajen.

Lista sistemática de años, sacos y precios por sacos (2010, 146, 75); (2011, 120, 80); (2012, 80, 90), (2013, 79, 100), (2014, 30, 120). Cuadro de doble entrada con los datos anteriores

65

Años Sacos Precio por saco (variación anual) Importe en soles

2010 146 S/ 75

2011 120 S/ 80

2012 80 S/ 90

2013 79 S/ 100

2014 30 S/ 120

10950

9600

7200

7900

3600

Con los datos del cuadro, teniendo en cuenta la pregunta de la situación problemática 1, resolvemos la operación combinada. S = 1 46  75  120  80  80  90  79 100  30 120 = 39250 

Cuarto paso: Examinar la solución obtenida

El docente debe pedir a los estudiantes que verifiquen sus resultados, que se cercioren si cometieron algún error en el proceso y, además, que propongan nuevas formas de resolver el mismo problema. La docente pide a los estudiantes leer nuevamente el problema, revisar los cálculos y analizar las preguntas, para tener claro lo que se pide. Las respuestas deben considerar las tres dimensiones de los saberes, saber, hacer y ser. Finalmente, los equipos de trabajo deben socializar sus resultados a los estudiantes del aula. CIERRE En esta etapa, el docente debe sistematizar la actividad desarrollada por los estudiantes con los conocimientos más relevantes. Seguidamente, el docente debe entregar una ficha metacognitiva a los estudiantes para que reflexionen. Las fichas metacognitivas deben estructurarse en base a los conocimiento desarrollados y deben ser a través de preguntas. ¿En qué medida los pasos de Pólya te ayudaron a resolver la situación problemática? ¿En qué otras situaciones utilizarás lo que aprendiste hoy? ¿Qué dificultades tuviste en el proceso? Evaluación Finalizada la sesión, los equipos de trabajo entregarán las fichas de evaluación que recibieron al inicio de la sesión. –

Instrumento: Escala de estimación para la autoevaluación

ESCALA ÍTEMS 1 Me integre en el desarrollo de la situación problemática. 2 He diseñado un plan para desarrollar la situación problemática. 3 Utilicé el libro escolar como fuente de consulta. 4 Identifiqué datos, condiciones e interrogantes. 5 Elaboré un cuadro para los datos del problema. 6 Establecí la jerarquía de las operaciones combinadas.

66

0

1

2

3

7 8

Desarrollé las operaciones combinadas adecuadamente. El desarrollo de la situación problemática me permitió empoderarme de los números naturales y sus operaciones

Escala de valoración Siempre: 3 Casi siempre: 2 A veces: 1 Nunca: 0

2.2.2

Previniéndonos contra las bajas temperaturas

Competencia Capacidades Indicadores Actúa y piensa Comunica y representa Expresa el significado del signo en el matemáticamente en ideas matemáticas. número entero en situaciones diversas. situaciones de cantidad. Expresa en forma gráfica y simbólica las relaciones de orden entre números enteros empleando la recta numérica. INICIO

Situación problemática 2 Un grupo de estudiantes de la promoción del colegio 6 de Agosto de Junín desea realizar una excursión a la ciudad de Lima; para ello, buscan información que les permitirá tomar medidas de prevención. En un libro de la biblioteca, encuentran que “Ticlio está a 4818 m s. n. m. y la temperatura promedio oscila según las estaciones del año, como se muestra en el siguiente cuadro”.

  

Fuente:http://www.clubdemochileros peru.com/category/alta-montana-3/

¿Qué deben hacer para saber la variación de temperaturas cada mes? ¿Cuál es la diferencia entre la temperatura máxima y la mínima en febrero? ¿Qué recomendaciones darías respecto al paseo?

El docente debe formar equipos de trabajo para que los estudiantes se apoyen entre ellos y, en seguida, repartir la situación problemática. Entregamos a cada equipo la situación problemática 2.

67

Sugerimos realizar la motivación y el recojo de saberes previos relacionados con la situación problemática 2. ¿Cómo es la temperatura en Ticlio? ¿Te gustaría conocer Ticlio?, ¿por qué? ¿En qué mes sería recomendable viajar? Seguidamente, el docente debe generar conflicto cognitivo en los estudiantes, a través de una pregunta: ¿Qué operaciones realizas para saber la variación de la temperatura cada mes? A continuación, el docente debe presentar el propósito de la sesión, mencionando los conocimientos que deben lograr los estudiantes y la forma como serán evaluados. En esta etapa, el docente entregará a cada equipo la ficha de evaluación para que los estudiantes se evalúen durante el proceso. Los estudiantes construirán el significado y uso de operaciones de adición y sustracción en Z, a partir de la situación problemática 2. Los estudiantes reciben una ficha de coevaluación. DESARROLLO Para que los estudiantes resuelvan la situación problemática, recomendamos al docente proponer los 4 pasos de Pólya.  Primer paso: Comprensión del problema El docente debe pedir a los estudiantes que lean detenidamente la situación problemática, hasta familiarizarse con ella. Para verificar si los estudiantes comprendieron el problema, el docente debe hacer preguntas sobre identificación de datos, condiciones e interrogantes. ¿Qué datos tenemos en la situación problemática? Los estudiantes serán inducidos a responder: Tenemos: meses, temperaturas máxima y mínima. ¿Qué condiciones tenemos? El docente debe inducir a los estudiantes a responder: La temperatura en Ticlio es variable según el paso de los meses. ¿Qué interrogante tenemos? Los estudiantes responderán: ¿Qué deben hacer para saber la variación de temperaturas cada mes? ¿Cuál es la diferencia entre la temperatura máxima y la mínima en febrero? ¿Qué recomendaciones darías respecto al paseo?

68

 Segundo paso: Configurar un plan El docente debe pedir a los estudiantes que propongan estrategias, tomando como punto de partida los datos, las condiciones y las interrogantes, induciendo al descubrimiento de estrategias a través de las siguientes preguntas: ¿Alguna vez resolvieron un problema parecido?. ¿Observan relación entre los datos?. ¿Identifican relación entre los datos y la incógnita?. Si los estudiantes aún no construyeron sus estrategias, el docente debe proporcionar a cada grupo un problema parecido de menor dificultad, para identificar las estrategias a utilizar. Los estudiantes escriben las temperaturas con la ayuda de la recta numérica.                                                                                                                                     -5°

-4°

-3°

-2°

-1°

0



















10°

11°

12°

13°

14°

15°

16°

Es decir, ubica 15° en la recta numérica a la derecha del cero, y -5° hacia la izquierda del cero. 

Tercer paso: Ejecutar el plan

Se sugiere pedir a los estudiantes que desarrollen la estrategia propuesta, teniendo en cuenta los datos, las condiciones y las preguntas. En este proceso, el docente debe estar atento a que todos los grupos trabajen.

Con ayuda de la recta numérica, los estudiantes desarrollan la operación y dan respuesta a la pregunta formulada de la siguiente manera:

                                        -5°

-4°

-3°

-2°

-1°

0

                                                                                            1°

















10°

11°

12°

13°

14°

15°

16°

15° - (-5°) = +20° Es decir, que de 15° ha retrocedido en la recta numérica hasta que la temperatura sea -5°, lo que significa 20 espacios hacia la izquierda. 

Examinar la solución obtenida

El docente debe pedir a los estudiantes que verifiquen sus resultados, que se cercioren si cometieron algún error en su proceso; además, que propongan nuevas formas de resolver el mismo problema.

La docente pide a los estudiantes que lean nuevamente el problema, así como la pregunta para tener claro lo que se les pide, y revisar los cálculos. Finalmente, los grupos deben socializar sus resultados a los estudiantes del aula.

69

CIERRE Construcción del aprendizaje En esta etapa, el docente debe sistematizar los conocimientos desarrollados durante las actividades de la sesión, lo que permitirá a los estudiantes redondear la idea. En la situación planteada, a las temperaturas menores que cero se les conoce como “bajo cero”, para ello se le antepone el signo menos (-). Así, por ejemplo, en el mes de febrero, la temperatura mínima absoluta T = -5° C, que se lee “5 grados bajo cero”.

Seguidamente, el docente debe generar, en los estudiantes, reflexión metacognitiva de todo el proceso a través de preguntas, que deben estar relacionadas con los conocimientos trabajados.

¿En qué medida te ayudaron los pasos de Pólya a resolver la situación problemática?. ¿En qué otras situaciones utilizarías la adición y sustracción de números enteros?. ¿Qué dificultades tuviste en el proceso?. ¿Cómo las superaste? Finalmente, los estudiantes entregarán al docente la ficha de coevaluación que registraron durante el proceso de la sesión. (Instrumento: escala de estimación para la coevaluación). Ficha de coevaluación Ítems Escala 1 Se implica y compromete en el cumplimiento de las tareas. 2 Colabora en el trabajo de equipo. 3 Utiliza signos de números enteros para expresar temperaturas. 4 Elabora la recta numérica. 5 Ubica las temperaturas en la recta numérica. 6 Realiza operaciones de adición y sustracción de números enteros.

0

1

2

3

Escala de valoración Siempre: 3 Casi siempre: 2 A veces: 1 Nunca: 0

2.2.3

El terreno de don César

Competencia Actúa y piensa matemáticamente en situaciones de cantidad.

Capacidades Matematiza situaciones.

Indicadores Reconoce relaciones en problemas aditivos de comparación e igualación con fracciones y las expresa en un modelo.

70

INICIO El docente debe dar la bienvenida y entregar la situación problemática a los estudiantes. Situación problemática 3 Don César, hace dos años, compró un terreno de una hectárea, el cual distribuyó de la siguiente manera: La mitad del terreno para sembrar papaya, la cuarta parte para café, la quinta parte para cacao y el resto del terreno para construir su vivienda. El señor César necesita conocer: ¿Cómo haré para saber los metros cuadrados para vivienda? ¿Cuántos metros cuadrados de terreno queda para mi vivienda? ¿Qué recomendaciones me darían ustedes?

Fuente:http://casas.mitula.pe/detalle/666/ 99761999622134456/4/1/terrenos-cacaochacra

Seguidamente, el docente debe formar equipos de trabajo para que los estudiantes puedan compartir responsabilidades; en seguida, se debe repartir a los equipos la situación problemática. Entregamos a cada equipo la situación problemática 3. Sugerimos realizar la motivación y el recojo de saberes previos relacionados con la situación problemática 3. Motivación: Los estudiantes reciben tangram (rompecabezas) y arman un cuadrado, rectángulo y paralelogramo. Esta actividad se realiza para que reconozcan, en cada ficha, la fracción a la que representa. Además, se produce la manipulación de material concreto. Otra motivación podría ser: Entregar una hoja de papel para que los estudiantes lo dividan por la mitad, nuevamente por la mitad hasta hacer 1/8 de papel, y cada vez van preguntando: ¿Qué parte de la unidad es?. Saberes previos. El docente debe formular las siguientes preguntas: ¿Qué frutos se siembran en la chacra de don César?. ¿A qué llamamos hectárea?. ¿Cómo expresas la parte del todo?. ¿Qué operaciones se deben realizar para ayudar a resolver la inquietud de don César?. A continuación, el docente debe presentar el propósito de la sesión y la forma de cómo serán evaluados los estudiantes. Construiremos el significado y uso de fracciones como parte-todo, decimales y porcentaje, a partir de la situación problemática 3. El docente debe entregar a cada grupo una ficha de coevaluación para que registren el proceso de desarrollo según los criterios.

71

DESARROLLO: Para el desarrollo de la sesión, el docente debe proponer a los estudiantes utilizar el método de Pólya.  Comprender el problema. El docente debe pedir a los estudiantes que lean analíticamente el problema hasta familiarizarse y, para cerciorarse que entendieron, deben parafrasear, mencionar la información, indicar la relación entre los datos y las interrogantes.  Configuración del plan. El docente debe pedir a los estudiantes que propongan estrategias, tomando como punto de partida los datos, las relaciones entre datos y las interrogantes, induciendo al descubrimiento de estrategias a través de las siguientes preguntas: ¿Alguna vez resolvieron un problema parecido?. ¿Observan relación entre los datos?. ¿Identifican relación entre los datos y las interrogantes?. Si los estudiantes aún no encontraron sus estrategias, el docente debe proponer a cada grupo un conjunto de ellas para que se decidan por una. Los estudiantes escogieron diagrama de tiras, el cual representará la división del terreno.  Ejecución del plan. El docente debe acompañar a los estudiantes durante el desarrollo de la construcción de sus aprendizajes a través de preguntas guiadoras. Los estudiantes utilizan el diagrama de tiras dividiendo el terreno 20 partes Papaya 1/2 1/20

1/20

1/20

1/20

1/20

Café 1/4 1/20

1/20

1/20

1/20

1/20

1/20

1/20

*

1/20

1/20

1/20

1/20

1 ×10000 Papaya: 2 1 ×10000 20

Cacao 1/5 1/20

1/20

1/20 1/20

1 ×10000 Café: 4

Cacao:

1 ×10000 5

Vivienda (*):

 Mirar hacia atrás. En esta etapa, el docente debe pedir a los estudiantes que verifiquen el diagrama; además, leer nuevamente las preguntas y preguntar si existen otras formas de resolver el problema. ¿Cómo haré para saber los metros cuadrados para vivienda? ¿Cuántos metros cuadrados de terreno queda para mi vivienda? ¿Qué recomendaciones me darían ustedes? Otra forma de obtener la segunda pregunta es:

1 1 1 19 + + = 2 4 5 20 19 1 1− = 20 20

72

1 ×10 000=500 m2 20 Respuesta: Los estudiantes deben socializar sus resultados para validarlos. CIERRE: El docente debe sistematizar los conocimientos desarrollados durante la sesión con la participación de los estudiantes para afianzar los conocimientos adquiridos. Seguidamente, el docente debe generar, en los estudiantes, la reflexión metacognitiva de todo el proceso a través de preguntas.

En la resolución del problema de la situación 3, ¿en qué medida te ayudaron el uso de cuadros y la reducción a la unidad?. ¿Qué dificultades tuviste en el proceso?. ¿Cómo las superaste?. El proceso de la sesión debe ser registrado en una ficha de coevaluación. ESCALA

0

1

2

3

ÍTEMS 1 Se implica y compromete en el cumplimiento de las tareas. 2 Utiliza el texto escolar durante el desarrollo. 3 Identifica fracciones como parte de un todo. 4 Reconoce situaciones aditivas con fracciones. 5 Aporta con ideas durante el proceso de desarrollo. 6 Apoya sus compañeros que tienen dificultades. Escala de valoración Siempre: 3 Casi siempre: 2 A veces: 1 Nunca: 0 Reflexión a) ¿Consideras que es necesario presentar la historia de la matemática en el proceso de construcción de conocimientos? b) ¿Las orientaciones didácticas propuestas en cada situación te ayudarán para tu nueva práctica docente?

3. Herramientas para la nueva práctica 3.1. Actividades de reflexión individual Te proponemos algunas interrogantes para que respondas con propuesta de actividades:

a) ¿Qué materiales o recursos disponibles de la zona donde llevas a cabo tu práctica pedagógica utilizarás para la construcción de conocimiento de fracciones? b) ¿Qué estrategias emplearías para que tus estudiantes comprendan el sistema de los números racionales, las fracciones?

73

c) ¿Cómo aprovecharías el aporte de los sabios de la comunidad en tu proyecto con los estudiantes? d) ¿Qué parte de la unidad te pareció muy interesante e innovador y que te ayudará a mejorar tu práctica pedagógica?

74

3.2. Actividades a distancia A continuación, se presentan actividades para poner en práctica aspectos desarrollados en esta unidad, las cuales serán ejecutadas en el componente a distancia. a) Formula una situación problemática de acuerdo al contexto intercultural, que implique desarrollar conocimientos relacionados con sistemas numéricos. b) Selecciona una competencia, capacidades, indicadores, estrategias, recursos y materiales a partir de una situación problemática creada. c) Elabora un diseño de sesión de aprendizaje, tomando en cuenta la actividad anterior. d) Ejecuta la sesión elaborada y recoge las evidencias para que puedan ser adjuntadas en el portafolio (fotos, videos, fichas, etc.). e) Archiva el diseño de la sesión y el instrumento de evaluación en su portafolio.

3.3. Actividades de metacognición

a) b) c) d) e) f) g)

¿Tuviste dificultades en el desarrollo de esta parte?, ¿cómo las superaste? ¿Cuál es tu compromiso para innovar tu práctica? ¿Cómo fue tu participación en el desarrollo de esta unidad? ¿Cuál es el rol protagónico que asumirás en tu institución educativa? ¿Qué importancia tiene la historia en el aprendizaje de la matemática? ¿Consideras que el juego es importante como estrategia de aprendizaje. ? ¿Qué lecciones aprendidas te quedan al término de la unidad?

3.4. Actividad para reflexionar a)

En el diseño, elaboración y ejecución de tus sesiones de aprendizaje, ¿qué cambios has experimentado como producto de los talleres?

3.5. Autoevaluación Estimado docente, evalúa tu participación durante el taller, marcando con un aspa según sea el caso. Apellidos y nombres: ……………………………………………………………………………………………………………………

N.o

Descriptores

1

Participé activamente en las tareas indicadas durante el desarrollo de la sesión. Escuché con atención las indicaciones y respeté las normas de trabajo señaladas. Cumplí con el tiempo previsto en las actividades señaladas. Respeté a mis colegas y contribuí en las actividades grupales. El desarrollo del contenido fue de mi interés. Aclaré mis dudas.

2 3 4 5 6

4. Glosario Cardinal. Cantidad de elementos de objeto. Ordinal. Orden que ocupa un objeto dentro de una colección ordenada. Fracción decimal. Denominador 10 ó múltiplos de 10. 75



No

Fracción propia. Numerador menor que su denominador. Fracción impropia. Si el numerador es mayor que su denominador. Se puede expresar como un número mixto formado por un número natural más una fracción propia.

5. Texto complementario La chacra achuar: un mundo de conocimientos “… la chacra es un lugar productivo del cual se benefician el ser humano y algunos animales. Es un espacio hecho y conservado con mucho trabajo por el hombre y la mujer, en el que se cultivan y producen diferentes plantas: la mayoría son comestibles, otras son muy importantes para las curaciones y para la comunicación con el espíritu, y otras se utilizan para o en la elaboración de objetos y artefactos. Es en ese espacio en el que a los niños (hombres y mujeres) se les va enseñando durante el desarrollo de cada una de las actividades propias de http://dioniciomarquez.blogspot.pe/2007/12 la agricultura (incluyendo la misma hechura de la chacra), /mi-experiencia-en-la-selva.html cómo deben ellos proceder cuando les corresponda hacer o cuidar su propia chacra. Además, las jornadas cotidianas de trabajo en la chacra también son aprovechadas por los padres para aconsejar a los hijos e hijas sobre las normas y comportamientos que deben tener en la comunidad. La chacra se aprovecha también para realizar la caza de los animales que la acostumbran visitar como: majás, añuje, punchana y otros. Allí, la cacería de estos animales se hace con escopeta y con trampas: la de lazo y la de tapaje. La chacra es el lugar en el que se acostumbra enterrar, cuando muere, al perro cazador que ha ayudado a conseguir el alimento. Anteriormente, allí se recitaban y se enseñaban los cantos o discursos relacionados con la actividad de la chacra, ahora estas prácticas han disminuido. Era igualmente el lugar más apropiado para cuidar a una persona cuando había sido mordida por una víbora venenosa. La chacra era el lugar para preparar el tseas. Este es el veneno (curare) de bejucos que se utilizaba para la cacería colocándolo en la punta de un virote o dardo. En las viviendas no se podía preparar porque era muy peligroso; el alma de los bejucos chupa la sangre de las personas y de los animales. Ahora, ya no se prepara este veneno. Se caza muy poco con pucuna y la gente tiene escopetas y compra municiones…”. Tomado de FORMABIAP. La chacra achuar, un espacio de aprendizaje. 2013, pp. 24 -26

6. Referencias CRUZ, C. A. (2013). La fracción como relación parte todo. Bogotá: Universidad Nacional de Colombia. GODINO J. (2003). Sistemas numéricos y didáctica para maestros. Granada: Repro Digital. Facultad de Ciencias. IPEBA. (2013). Mapas de Progreso . Lima: CEPREDIM. MINISTERIO DE EDUCACIÓN. (2013). Rutas del Aprendizaje. ¿Qué y como aprenden nuestros adolescentes? Lima: Autor. MARIN, R. (2012). Matemática 1.° Secundaria. Lima: Norma.

76

Anexo EVALUACIÓN DE LA UNIDAD II

Tabla de especificaciones La tabla de especificaciones es un instrumento de evaluación que contiene información sobre el contenido esencial de lo que se va a evaluar en esta unidad. Funciones de la tabla de especificaciones La tabla de especificaciones sirve para: 1. Construir los ítems del instrumento de evaluación. 2. Orientar la interpretación de los resultados.

Tabla de especificaciones

CÓDIGO

DESCRIPTORES

PSPCRN 1

Plantea situaciones problemáticas de cantidad relacionadas con números naturales, utilizando operadores aditivos y multiplicativos, considerando recursos, actores y escenarios rurales. Plantea situaciones problemáticas de cantidad relacionadas con números enteros, utilizando operadores aditivos y multiplicativos, considerando recursos, actores y escenarios rurales. Plantea situaciones problemáticas de cantidad relacionadas con números racionales, utilizando operadores aditivos y multiplicativos, considerando recursos, actores y escenarios rurales. Resuelve situaciones problemáticas de cantidad relacionadas con números enteros, en un contexto intercultural, utilizando los cuatro pasos de Pólya y describiendo las estrategias empleadas en cada una de las fases. Resuelve situaciones problemáticas de cantidad relacionadas con números racionales, en un contexto intercultural, utilizando los cuatro pasos de Pólya y describiendo las estrategias empleadas en cada una de las fases.

PSPCRZ 2

PSPCRQ 3

RSPCRZ 4

RSPCRQ 5

TOTAL

N.° DE ÍTEMS 1 1 1 1

1

6

Pruebas de desempeño A diferencia de las pruebas de opción múltiple, que son instrumentos de evaluación cerrados, las pruebas de desempeño son instrumentos de evaluación abiertos que permiten evaluar los procesos de planteamiento y resolución de diversas situaciones problemáticas de cantidad.

Consistencia de las pruebas de desempeño Una prueba de desempeño es consistente si corresponde a la tabla de especificaciones, es decir, si cada uno de los ítems de la prueba responde a los descriptores y es un representante significativo del descriptor.

PSPCRN1 Plantea una situación problemática de cantidad que tenga las siguientes características:

77

a) b) c) d) e)

Los datos deben partir de alguna actividad socioproductiva de una comunidad rural. Debe considerar recursos y actores locales. Los datos deben inducir a la construcción del significado de números naturales. Los datos deben contener operadores aditivos y multiplicativos en N. Deben permitir formular preguntas del saber, hacer y ser.

PSPCRZ 2 Plantea un problema sobre una situación de cantidad que tenga las siguientes características: a) Los datos deben partir de alguna actividad socioproductiva de una comunidad rural. b) Deben considerar recursos y actores locales. c) Deben inducir a la construcción del significado de números enteros. d) Los datos deben contener operadores aditivos y en Z. e) Deben permitir formular preguntas del saber, hacer y ser.

PSPCOAMQ 3 Plantea un problema sobre una situación de cantidad que tenga las siguientes características: a) Los datos deben partir de alguna actividad socioproductiva de una comunidad rural. b) Debe considerar recursos y actores locales. c) Deben inducir a la construcción del significado de números racionales. d) Los datos deben de contener operadores aditivos y multiplicativos en Q. e) Deben permitir formular preguntas del saber, hacer y ser.

RSPCRZ 4 Averiguando la temperatura en la ciudad de Puno, se tuvo la siguiente información: Un día de invierno amaneció 4 grados bajo cero. A las doce del mediodía ha subido 6 grados, y hasta las cuatro de la tarde bajó 2 grados. Desde las cuatro hasta las doce de la noche bajó 4 grados, y desde las doce a las 6 de la mañana bajó 5 grados más. Sabemos que la temperatura en el Altiplano varía según las estaciones y el horario. De la información proporcionada: a) ¿Qué temperatura se registra a las 6:00 de la mañana? b) ¿A las 12 de la noche, qué temperatura se registra en la ciudad de Puno? c) ¿Cuál es el procedimiento seguido para dar respuesta a cada pegunta? d) ¿Cuál es la variación promedio de la temperatura del ambiente en la localidad donde trabajas? e) ¿Qué recomendaciones le darías a un turista que llega al Altiplano?

RSPCRQ 5 Un docente de la zona rural ha visto por conveniente contribuir en la solución del problema de presupuesto familiar que presentan los padres de familia de los estudiantes de 2. o grado de secundaria. Dichos estudiantes acuden al mercado más cercano de la comunidad para recoger información sobre los precios de los productos de primera necesidad, considerando un almuerzo para un día familiar de cuatro integrantes, tal como se presenta en el siguiente cuadro:

PRODUCTO

CANTIDAD KG

PRECIO UNITARIO S/

78

PRECIO TOTAL S/

Charqui

1

3,00

3,00

Huevo

1/4

1,20

1,20

Quinua

1/2

2

2

Morón

1/4

0,50

0,50

Papa

2

0,80

1,60

Maíz

1/2

0,60

0,60

Atado

0,80

0,80

Verduras

a) b) c) d)

¿A cuánto asciende el gasto para el almuerzo de un día? ¿Cuál es el monto del gasto que representa al mes? ¿Cómo haré para saber cuál es el monto del gasto que representa al año? Si el gasto del desayuno y la cena es equivalente a los ¾ del almuerzo, ¿a cuánto asciende el gasto total al mes? e) ¿Qué opinión te merece la actitud del docente?

79

UNIDAD III Proporcionalidad, magnitudes directa e inversamente proporcionales

PRESENTACIÓN DE LA UNIDAD Estimado participante del programa de actualización docente dirigido a maestros de Secundaria Rural, en esta oportunidad te presentamos el módulo de la unidad III “Proporcionalidad, magnitudes directa e inversamente proporcionales”, que parte del análisis de casos para promover la reflexión desde la práctica docente, lo que permite contrastar con la teoría para el proceso de construcción e interpretación de magnitudes proporcionales. La estrategia didáctica central de la unidad está basada en la construcción e interpretación de magnitudes proporcionales a partir de situaciones problemáticas de contexto real, que permitirá al docente reflexionar desde su práctica sobre su desempeño en la didáctica para construir una matemática funcional. En esta dirección, se construye e interpreta el significado de magnitudes proporcionales, así como el planteamiento y resolución de problemas sobre proporcionalidad.

Fuente:http://bach1amates.blogspot. pe/2013/10/la-proporcion-aurea-enel-arte.html

ESQUEMA DEL LOS CONTENIDOS Reflexión desde la práctica

PR O PO R CIO N A LID A D

Reflexión teórica

Magnitudes directamente proporcionales

Magnitudes inversamente proporcionales

Orientaciones didácticas

Evaluación de los aprendizajes

Herramientas para la nueva práctica Actividades de reflexión individual/equipo, a distancia, de metacognición y autoevaluación Glosario, texto complementario, referencias

80

81

1. Reflexión desde la práctica Observa las siguientes imágenes:

Creo que sí.

3.1.1. Trabajo individual. A partir de la observación de las imágenes, responde las siguientes preguntas: a) ¿Qué competencias y capacidades crees que está desarrollando el docente según el gráfico? b) ¿Consideras que el aprendizaje de proporcionalidad es significativo para los estudiantes?, ¿por qué? c) ¿Qué estrategias está empleando el docente para desarrollar proporcionalidad? d) ¿Se evidencia el trabajo organizado y en equipo de sus estudiantes? e) ¿Cómo diseñas y ejecutas tu sesión para que el aprendizaje de proporcionalidad sea significativo para los estudiantes?

82

a) ¿Qué estrategias empleas para el desarrollo de competencias relacionadas con proporcionalidad?

83

2. Reflexión teórica 2.1. Fundamento teórico 2.1.1. ¿Cómo surgió la proporcionalidad? Tales de Mileto es uno de los “siete sabios” de la Antigüedad. Destacó tanto en la filosofía como en la matemática. “En matemática se le atribuyen las primeras demostraciones de teoremas geométricos mediante el razonamiento lógico” [CITATION Per94 \l 10250 ]; aquí mencionamos algunas: “Todo diámetro biseca a la circunferencia”. “Los ángulos de la base de un triángulo isósceles son congruentes ”. “Todo triángulo inscrito en una semicircunferencia es recto”. “Los segmentos determinados por una serie de paralelas cortadas por dos transversales son proporcionales”. Teorema de Tales, conocido también como el teorema de la proporcionalidad de segmentos, nos informa que los segmentos generados por dos secantes son proporcionales. Bajo este teorema, se auspició la creación de la ciencia griega de las proporciones, pues, básicamente, nos viene a decir que los lados de los triángulos semejantes son proporcionales. Para medir la distancia -en la costa- de los barcos en alta mar, la leyenda dice que Tales fue el primero en emplear la proporcionalidad de los lados de triángulos semejantes. Hay dudas muy grandes al respecto, ya que estas ideas se habían manejado con mucha anterioridad en Egipto y Mesopotamia, donde Tales invirtió una parte de su vida. (Díaz, 2002). Euclides fue otro de los matemáticos famosos cuya obra cumbre es Elementos de geometría, que se compone de trece libros. Del libro I al libro VI se dice que se dedica a la geometría elemental; del libro VII al X, a cuestiones numéricas; del XI al XIII, a la geometría de los sólidos. En el libro VI, como tercera definición, aparece el texto: “se dice que la recta está dividida en medida y extrema razón cuando la longitud de la línea total es a la de la parte mayor, como la de esta parte mayor es a la de la menor”. Dicho de otro modo: “El todo es a la parte como la parte, al resto” [CITATION GEO12 \l 10250 ]. Esta definición dio origen al famoso número de oro o número áureo que está presente en diversas manifestaciones de la naturaleza como aplicación de las proporciones. Así, interpretando lo anterior, tratamos de dividir un segmento en dos partes desiguales de la forma más general y directa posible de la siguiente manera: Dado el segmento X1

1

x

84

Aplicando lo expresado por Euclides se tendría:

x 1 = 1 x−1

x 2−x−1=0

∅ 1=

1+ √ 5 1− 5 =1.6180339 … ∅2 = √ =−0.6180339 … 2 2

PROPORCIONALIDAD

Magnitud. Es todo aquello que puede ser medido o cuantificado. Así, por ejemplo, el área de un terreno, la edad de una persona, la altura de la pared, etc. Proporcionalidad directa. Dos magnitudes son directamente proporcionales (DP) cuando el cociente de sus valores correspondientes es una constante. Es decir, si consideramos dos magnitudes A y B:

A es D . P .a B ↔

A =k B

Donde k es una constante de proporcionalidad. Magnitud A

a1

a2

a3

a4

Magnitud B

b1

b2

b3

b4

a1 a2 a3 a4 = = = =k . Es decir, si A es DP a B, entonces b1 b2 b3 b4 A

. Gráficamente: K

a3 a2 a1 b1

b2

b3

B

Entonces la razón de proporcionalidad directa es k. A este tipo de proporción directa se le conoce como función lineal; es decir: A = Bk, donde k es la constante de proporcionalidad. Además, si trazamos una línea recta por los puntos, esta pasa por el origen de las coordenadas, lo cual es requisito para ser una función lineal.

85

Ejemplo, en la vida cotidiana, de dos magnitudes directamente proporcionales: Si vendo un cuy a 30 soles, 2 cuyes venderé a 60 soles. Podemos observar que al aumentar la cantidad de cuyes, aumenta el precio en la misma proporción. Proporcionalidad inversa: Dos magnitudes son inversamente proporcionales (IP) cuando el producto de sus valores correspondientes es una constante. Es decir, si consideramos dos magnitudes A y B:

A es I . P . a B ↔ A . B=k

Donde k es una constante de proporcionalidad. Podemos observar en el cuadro que cuando la magnitud A se duplica o triplica, la magnitud B se reduce a la mitad o a la tercera parte, respectivamente. Veamos la siguiente tabla: Magnitud A

a1

a2

a3

a4

Magnitud B

b1

b2

b3

b4

a1.b1 = a2.b2 = a3.b3 = a4.b4 = k. Es decir, si A es IP a B, entonces Gráficamente:

A x B,

A a3

a2

K

a1

b1 b2

b3

B

La gráfica de dos magnitudes inversamente proporcionales es una curva. Ejemplo, en la vida cotidiana, de dos magnitudes inversamente proporcionales: Dos campesinos pueden barbechar una chacra en 4 horas, mientras que un solo campesino barbechará la misma chacra en 8 horas. Podemos observar que al disminuir el número de campesinos, las horas de trabajo aumentarán en la misma proporción.

86

2.2. ¿Cuáles son las orientaciones didácticas que emplearías para el desarrollo de los aprendizajes? A continuación, explicaremos el desarrollo de una competencia, a partir situaciones problemáticas, utilizando medios, materiales y verificando el logro de los aprendizajes.

87

2.2.1. La producción del biohuerto Competencia Capacidades Actúa y piensa Comunica y representa matemáticamente en ideas matemáticas. situaciones de cantidad.

Indicadores Organiza datos en tablas para expresar relaciones de proporcionalidad directa e inversa.

INICIO Situación problemática Los estudiantes de la institución educativa de la localidad de Cotahuasi, Arequipa, visitan el huerto escolar, donde hubo cosecha de papas como parte del proyecto de biohuertos escolares. Allí el coordinador de EPT informa que se están embolsando papas en bolsas de 20 kilos para la venta, además se sabe que la cosecha total es de 840 kg. También informa que con 120 kg de papa se abastece a un restaurante durante una semana. a) ¿Qué debo hacer para saber cuántos kilos de papas se necesitan para abastecer tres semanas a un restaurante? b) ¿Cuántas bolsas se necesitan para embolsar 140 kg de papa? c) Si aumento la cantidad de kilos de papa, ¿se necesitan más o menos bolsas? d) Si vendemos al por mayor, ¿crees que es justo mantener el mismo precio por kilo de papa? El docente debe formar equipos de trabajo para que se ayuden en las reflexiones de las actividades; seguidamente, debe entregar las preguntas relacionadas con la situación problemática y las fichas de coevaluación. A continuación, sugerimos realizar la motivación y el recojo de saberes previos relacionados con la situación problemática. ¿Qué les pareció la visita al huerto escolar? ¿Qué producto se cosechó en el huerto? ¿A mayor cantidad de papas, se requieren más bolsas o menos bolsas? Seguidamente, la docente, a través de algunas preguntas, debe generar un conflicto cognitivo en los estudiantes. ¿Cómo se relacionan la cantidad de papas con el número de bolsas? A continuación, la docente debe presentar el propósito de la sesión, la forma como serán evaluados los estudiantes, y mencionar que se evaluarán entre ellos a través de una ficha de coevaluación. Los estudiantes construirán el significado y uso de la proporcionalidad directa, a partir de la situación problemática generado en el huerto.

DESARROLLO Para que los estudiantes resuelvan la situación problemática presentado al inicio, recomendamos al docente proponer los cuatro pasos de Pólya.

88

Primer paso: Comprensión del problema El docente debe pedir a los estudiantes que lean detenidamente la situación problemática hasta familiarizarse con ella. Para verificar si los estudiantes comprendieron el problema, el docente debe pedir que lo parafraseen y hacer preguntas sobre identificación de datos, condiciones e interrogantes. a) ¿Qué datos tenemos en la situación problemática? Se debe inducir a los estudiantes para que respondan los siguientes datos: Casa bolsa debe contener 20 kg de papa. Total de producción, 840 kg de papa. 120 kg de papas abastecen por una semana a un restaurante. b) ¿Qué condiciones tenemos? El docente debe inducir a los estudiantes para que respondan: Producción de papas en el huerto escolar c) ¿Qué interrogante tenemos? Los estudiantes responderán: ¿Qué debo hacer para saber cuántos kilos de papas se necesitarán para abastecer tres semanas a un restaurante? ¿Cuántas bolsas se necesitan para embolsar 140 kg de papa? Si aumento la cantidad de kilos de papa, ¿se necesitan más o menos bolsas? Si vendemos al por mayor, ¿crees que es justo mantener el mismo precio por kilo de papa?

Segundo paso: Configurar un plan El docente debe pedir a los estudiantes que propongan estrategias tomando como punto de partida los datos, las condiciones y las interrogantes, induciendo al descubrimiento de estrategias a través de las siguientes preguntas: ¿Alguna vez resolvieron un problema parecido? ¿Qué relación encuentran entre los datos? ¿Identifican relación entre los datos y la incógnita? Si los estudiantes aún no construyeron sus estrategias, el docente debe proporcionar, a cada grupo, un problema parecido de menor dificultad, en el cual deben identificar las estrategias utilizadas. Utilizar un cuadro de doble entrada para hacer una lista de las cantidades de papa con sus respectivos números de bolsas. Representar un gráfico de función lineal para su interpretación. Elaborar un cuadro de doble entrada para número de días de consumo y kilogramos de papa. Representar con un gráfico curvo la función inversa para su interpretación.

Tercer paso: Ejecutar el plan Se debe pedir a los estudiantes que desarrollen cada una de las estrategias propuestas teniendo en cuenta los datos, condiciones y preguntas. En este proceso, el docente debe estar atento para que todos los grupos trabajen. En un cuadro, colocan las proporciones de los kilos de papa y las cantidades de bolsas necesarias: Bolsas

01

02

3

……..

89

210

420

Kg de papa

20

40

60

……..

420

840

El resultado de la división entre kilogramos de papa y número de bolsas permanece constante. Luego, para embolsar 840 kg de papa, se necesitan 420 bolsas. Esta situación la visualizamos en la siguiente gráfica:

Kg de papa

K

840

40 20

420

2

1

Bolsas

Elaboramos un cuadro de doble entrada para número de días de consumo y kg de papa: Días

07

08

Kg de papa

120

105

09

10

11

12

13

14 60

Del gráfico, observamos que a más días de consumo, menos cantidad de kilogramos de papa; además, los productos son iguales. Cuya gráfica en el plano cartesiano es: K g 120 de pa pa 105

K

60

7

8

14

Días

Cuarto paso. Examinar la solución obtenida El docente debe pedir a los estudiantes que verifiquen sus resultados, se cercioren si cometieron algún error en su proceso y, además, propongan nuevas formas de resolver el mismo problema. Los estudiantes leen nuevamente el problema y la pregunta para tener claro lo que se les pide.

90

Finalizados los cuatro pasos de Pólya, recomendamos al docente que solicite a los estudiantes que construyan otra situación problemática parecida, que tenga datos, condiciones, interrogantes y contexto. Finalmente, los estudiantes en grupo deben socializar sus resultados a los estudiantes del aula.

CIERRE El docente sistematiza los conocimientos trabajados en la sesión con participación de los estudiantes. Seguidamente, el docente debe generar en los estudiantes reflexión metacognitiva de todo el proceso a través de preguntas.

¿En qué medida te ayudaron los pasos de Pólya a resolver la situación problemática? ¿En qué otras situaciones utilizarás proporcionalidad? ¿Qué dificultades tuviste en el proceso? El proceso de la sesión debe ser registrada en una ficha de evaluación.

LISTA DE COTEJO Criterios

Estudiantes

Identifica, datos, condiciones e interrogantes.



No

Plantea estrategias de resolución de problemas.



No

Utiliza proporción directa e inversa en la resolución de problemas. Sí No

Representa a través de gráficos proporción directa e inversa. Sí No

2.2.2. ¿A más trabajadores, menos tiempo? Competencia Capacidades Actúa y piensa Comunica y representa matemáticamente en ideas matemáticas. situaciones de cantidad.

Indicadores Organiza datos en tablas para expresar relaciones de proporcionalidad directa e inversa.

INICIO El docente debe dar la bienvenida y entregar la situación problemática a los estudiantes.

91

Situación problemática Los pobladores de Tambocucho-Tambillo se dedican al cultivo de cereales, tubérculos, entre otros. En la época de sembrío, tienen que contratar personas para que ayuden en labores de siembra. Por ello, don Andrés contrata 4 trabajadores para que puedan trabajar durante 14 días y terminar de sembrar según su cálculo. ¿Cuánto se tardaría en hacer ese mismo trabajo si se contrataran 7 trabajadores? ¿Qué recomendaciones darías a don Andrés? El docente forma equipos de trabajo heterogéneos para que se ayuden y entrega la situación problemática. Puede utilizar alguna dinámica para el caso. El docente debe motivar a los estudiantes y recoger los saberes previos, ambos momentos deben estar relacionados con la situación problemática a resolver. La motivación es importante porque permite al estudiante interesarse por el tema y generar la recepción de la información. El recojo de saberes previos es necesario porque permite diagnosticar las potencialidades de los estudiantes, de acuerdo a lo cual el docente utilizará una didáctica pertinente. ¿Alguien de ustedes tiene un cultivo de cereales en su chacra?. ¿Alguien conoce la chacra de don Andrés?. ¿Cuántos trabajadores contratará? ¿De qué trata el problema? El docente debe generar conflicto cognitivo en los estudiantes a través de interrogantes.

Si 4 trabajadores pueden sembrar una chacra en 8 días, ¿cuántos trabajadores serán necesarios para sembrar la misma chacra en solo 4 días? Seguidamente, se debe mencionar el propósito, que debe relacionar la competencia, la situación problemática y el conocimiento a desarrollar. Construir el significado y uso de magnitud inversamente proporcional a partir de la situación problemática entregada. Por último, el docente debe mencionar a los estudiantes cómo serán evaluados y qué instrumento de evaluación se utilizará. Para el caso particular, el docente entregará fichas de coevaluación a cada equipo de trabajo, e indicará que deben ser llenadas según la participación en el avance de las actividades.

DESARROLLO Es la etapa de la construcción del aprendizaje de los estudiantes. El docente debe pedir a los estudiantes que realicen una lectura analítica de la situación problemática. Para verificar su comprensión, puede hacer las siguientes preguntas: ¿Qué datos nos muestra la situación problemática? ¿Qué condiciones aparecen en el problema?

92

¿Qué interrogantes se visualizan? El docente debe pedir a los estudiantes que propongan estrategias, tomando como punto de partida los datos, las condiciones y las interrogantes, induciendo al descubrimiento de estrategias a través de las siguientes preguntas: ¿Alguna vez resolvieron un problema parecido? ¿Observan relación entre los datos? ¿Identifican relación entre los datos y la incógnita? Si los estudiantes aún no construyeron sus estrategias, el docente debe proporcionar a cada grupo un problema parecido de menor dificultad, en el cual deben identificar las estrategias utilizadas. El estudiante utilizará un cuadro de doble entrada para anotar el número de máquinas excavadoras y el número de días. Una vez determinada la estrategia, empezamos a completar el cuadro. Días Número de trabajadores

14 4

7 8

28 ¿?

A partir del cuadro, reflexionan: ¿Qué tipo de magnitudes son el tiempo y la cantidad de trabajadores? Se debe inducir a que los estudiantes respondan: Son magnitudes inversamente proporcionales y se cumple que los productos son iguales 14 x 4 = 7 x 8 = 28 x 2 = 56 Se debe leer nuevamente el problema y responder la pregunta. CIERRE El docente debe sistematizar los conocimientos tratados en la sesión con la participación de los estudiantes. Seguidamente, debe hacer preguntas metacognitivas para que reflexionen sobre el trabajo desarrollado. ¿Qué otras utilidades puedes darle a la proporcionalidad inversa? ¿El cuadro de doble entrada te facilitó del desarrollo del problema? Todo el proceso debe ser registrado a través de un instrumento de evaluación. Finalmente, los estudiantes entregan al docente la ficha de coevaluación. Criterios

Identifica datos, condiciones e interrogantes.

Plantea estrategias de resolución de problemas.

Utiliza tablas de doble entrada para colocar datos.

Da respuesta coherente al problema planteado.

Estudian tes



No



No



No



No

2.2.3. Cuidemos nuestras carreteras Competencia

Capacidades

Indicadores

Actúa y piensa matemáticamente en

Elabora y usa estrategias.

Emplea estrategias heurísticas, recursos gráficos y otros, al resolver problemas

93

situaciones de cantidad.

relacionados con proporcionalidad.

INICIO El docente debe dar la bienvenida y entregar la situación problemática.

Situación problemática En el distrito de Muquiyauyo, provincia de Jauja, región Junín, la municipalidad provincial viene realizando trabajos de mantenimiento de la carreta central. Por ese motivo, se han contratado a 10 obreros para terminar el trabajo en 100 días. El alcalde provincial quiere reducir el tiempo a la mitad. ¿Qué debe hacer para saber cuántos obreros será necesario agregar para terminar la obra en 50 días? ¿Cuántos obreros serán necesarios contratar para terminar la obra en el tiempo previsto? ¿Qué recomendaciones darías al alcalde? El docente debe formar equipos de trabajo para que los estudiantes puedan interactuar. A continuación, realizar la motivación y el recojo de saberes previos relacionados con la situación problemática.

¿Mediante qué operaciones podré ayudar al alcalde para resolver el problema ? ¿Cuántos obreros serán necesarios contratar para terminar la obra en 50 días? Luego el docente, a través de preguntas, debe generar el conflicto cognitivo en los estudiantes. ¿Cómo se relacionan el tiempo y el número de trabajadores? Seguidamente, el docente debe presentar el propósito de la sesión y la forma como serán evaluados los estudiantes. ... de propósito: Los estudiantes construirán el significado y uso de la proporcionalidad inversa, a partir de la situación problemática 3. ... de evaluación: Los estudiantes serán evaluados durante el proceso de resolución de la situación problemática a través de una lista de cotejo. DESARROLLO El docente debe pedir a los estudiantes que lean detenidamente la situación problemática, que la digan con sus propias palabras y que identifiquen los datos, las incognitas y las interrogantes. A continuación, deben elaborar estrategias. Solamente en caso extremo, el docente sugerirá algunas de ellas.

94

Para dar respuesta a la pregunta, el docente induce a los estudiantes que analicen las magnitudes tiempo y número de obreros, donde concluyen que estas son inversamente proporcionales, ya que al aumentar el número de obreros, la obra terminará en menos tiempo. El docente presenta, en un paleógrafo, el siguiente gráfico: T 100

50

K

n 10

50 Obrero

m

Como son inversamente proporcionales, sabemos que los productos de las magnitudes son constantes: I.P Tiempo #de Obreros 100 días 10 obreros 50 días m

Respuesta: Se requieren 20 obreros. El docente propone una nueva pregunta: Si el alcalde contratara 50 obreros, ¿en cuánto tiempo terminarían la obra? Como son inversamente proporcionales sabemos que los productos de las magnitudes son constantes: I.P Tiempo #de Obreros 100 días 10 obreros X 50 obreros

Respuesta: El trabajo terminaría en solo 20 días.

CIERRE El docente debe sistematizar los conocimientos desarrollados para que los estudiantes consoliden la información. Asimismo, debe hacer preguntas metacognitivas a los estudiantes.

95

¿En qué otras situaciones de la vida podrás aplicar los conocimientos de proporcionalidad inversa? Todo el proceso debe ser registrado por los mismos estudiantes a través de una ficha de coevaluación.

Apellidos y nombres

Identifica datos, condiciones e interrogantes. (0-5)

Plantea estrategias de resolución de problemas. (0-5)

Utiliza tablas de doble entrada para colocar datos. (0-5)

Da respuesta coherente al problema planteado. (0-5)

Puntaje total

3. Herramientas para la nueva práctica 3.1. Actividades de reflexión individual a) ¿Qué materiales o recursos disponibles de la zona donde trabajas utilizarás para la construcción de proporciones y magnitudes directa o inversamente proporcionales? b) ¿Qué parte de la presente unidad te pareció muy interesante e innovador y que te ayudará en tu práctica pedagógica? c) ¿Tuviste dificultades en el desarrollo de esta sección? ¿Cómo las superaste? d) ¿Cuál es tu compromiso de innovar tu práctica?

3.2. Actividades a distancia A continuación, se presentan actividades para poner en práctica aspectos desarrollados en esta unidad, las cuales serán ejecutas en el componente a distancia. a) Crea una situación problemática de acuerdo a tu contexto intercultural que implique desarrollar conocimientos relacionados con proporcionalidad. b) Selecciona una competencia, capacidades, indicadores, estrategias, recursos y materiales (actores y/o escenarios) a partir de una situación problemática. c) Elabora un diseño de sesión de aprendizaje, tomando en cuenta la actividad anterior. d) Ejecuta la sesión elaborada y recoge las evidencias para ser incluidas en el portafolio (fotos, videos, audios, etc.). e) Archiva el diseño de la sesión e instrumentos de evaluación en tu portafolio.

3.3. Actividades metacognitivas a) b) c) d)

¿Cómo calificas tu participación en esta unidad, en el proceso de construcción de conocimiento? ¿Qué procedimientos te han permitido mejorar tu práctica pedagógica? ¿Qué estrategia utilizaste para participar activamente en el proceso? ¿Qué compromiso asumes para tu nueva práctica profesional?

3.4. Reflexión

96

a) b)

¿Has experimentado cambios sustanciales en el diseño de tus sesiones de aprendizaje? ¿Consideras que la metacognición permite reflexionar y comprometerse con los cambios que se proponen? ¿Consideras oportuno e importante mostrar en un paleógrafo el gráfico de magnitudes directa e inversamente proporcionales?, ¿por qué?

c)

3.5. Autoevaluación Estimado docente, evalúa tu participación durante el taller, marcando con un aspa según sea el caso. Apellidos y nombres: …………………………………………………………………………………………………………………… N.o 1 2 3 4 5

Descriptores Participé activamente en las actividades indicadas durante el desarrollo de la sesión. Escuché con atención las indicaciones y respeté las normas de trabajo señaladas. Cumplí con el tiempo previsto con las actividades señaladas. Respeté a mis colegas y contribuí en las actividades grupales. Me interesé en el tema y aclaré mis dudas.



No

4. Glosario Proporción. Comparación de dos razones. Magnitud. Todo aquello que se puede medir. Magnitudes directamente proporcionales. Cuando una magnitud aumenta, también la otra magnitud aumenta. Magnitudes inversamente proporcionales. Cuando una magnitud aumenta, la otra magnitud disminuye.

5. Texto complementario Pensamiento como intento de resolver problemas (Rogoff) La estructura de los problemas que los humanos intentan resolver, el conocimiento base que proporciona recursos para lograrlo y las estrategias más o menos eficaces que permiten alcanzar la solución se sitúan en una matriz social de propósitos y valores. Los problemas que se plantean, los instrumentos disponibles para resolverlos y las tácticas elegidas para abordarlos se construyen a partir de definiciones socioculturales y tecnologías disponibles de las que se sirve el individuo en un momento dado. La resolución del problema se produce, a veces, en situaciones sociales que los definen y que, además, proporcionan oportunidades para aprender de los intercambios sociales. Por tanto, considera a los niños como aprendices del conocimiento, activos en sus intentos de aprender a partir de la observación y de la participación en las relaciones con sus compañeros y con miembros más hábiles de su grupo social. De este modo, los niños adquieren destrezas que les permiten abordar problemas culturalmente definidos, con la ayuda de instrumentos a los que fácilmente pueden acceder, y construyen, a partir de lo que han recibido, nuevas soluciones en el contexto de la actividad sociocultural. […]

97

La resolución de problemas resalta la naturaleza activa del pensamiento, frente a la idea de considerar el proceso de conocimiento como la posesión pasiva de objetos mentales, por ..., contenidos de conocimiento y percepciones. Es decir, la gente, más que limitarse a adquirir recuerdos, percepciones y habilidades, explora, resuelve problemas y recuerda. El objetivo del proceso cognitivo no es producir pensamiento, sino guiar la acción inteligente, interpersonal y práctica. Una aproximación centrada en el proceso de resolución de problemas implica conceder primacía a los intentos de la gente de negociar el curso de la vida, trabajar en ello, o transformar los problemas que surgen en el camino para conseguir las diversas metas de la vida.

La distinción tradicional entre procesos cognitivos, afectivos y sociales resulta imprecisa al considerar el pensamiento como el intento de determinar medios inteligentes para alcanzar metas. Las metas que se proponen los humanos implican la presencia de otras personas y llevan consigo sentimientos. Pensar, sentir y actuar están integrados en la aproximación a la resolución de problemas que utilizo en este libro. Rogoff, B. (1993). Aprendices del pensamiento: el desarrollo cognitivo en el contexto social. Paidós.

6. Referencias Cruz, C. A. (2013). La fracción como relación. Bogotá: Universidad Nacional de Colombia. Editorial Colombia. DÍAZ, J. (2002). http://hernanrestrepoestrada.blogspot.pe/2011/04/historia-de-laproporcionalidad.html. Recuperado el 27 de octubre de 2015. EXTREMIANA, I. (s. f.). http://www.sectormatematica.cl/arte/divina_proporcion.pdf. Recuperado el 27 de octubre de 2015, de http://www.sectormatematica.c NATIONAL, G. (2012). La proporción áurea. El lenguaje matemático de la belleza. Editorial: RBA, Barcelona.

98

Anexo EVALUACIÓN DE LA UNIDAD III

Tabla de especificaciones La tabla de especificaciones es un instrumento de evaluación que contiene información sobre el contenido esencial de lo que se va a evaluar en esta unidad.

Funciones de la tabla de especificaciones La tabla de especificaciones sirve para: 1. Construir los ítems del instrumento de evaluación. 2. Orientar la interpretación de los resultados. TABLA DE ESPECIFICACIONES DESCRIPTORES

CÓDIGO PSPCRPD 1

PSPCRPI 2

RSPCRPD 3

RSPCRPI 4

Plantea situaciones problemáticas de cantidad relacionadas con proporción directa, considerando recursos, actores y escenarios rurales. Plantea situaciones problemáticas de cantidad relacionadas con proporción inversa, considerando recursos, actores y escenarios rurales. Resuelven situaciones problemáticas de cantidad relacionadas con proporción directa en un contexto intercultural, utilizando los cuatro pasos de Pólya y describiendo las estrategias empleadas en cada una de las fases. Resuelven situaciones problemáticas de cantidad relacionadas con proporción inversa en un contexto intercultural, utilizando los cuatro pasos de Pólya y describiendo las estrategias empleadas en cada una de las fases.

TOTAL

N.° DE ÍTEMS 1 1

1

1

4

Pruebas de desempeño A diferencia de las pruebas de opción múltiple que son instrumentos de evaluación cerrados, las pruebas de desempeño son instrumentos de evaluación abiertos que permiten evaluar los procesos de planteamiento y resolución de diversas situaciones problemáticas de cantidad.

Consistencia de las pruebas de desempeño Una prueba de desempeño es consistente si corresponde a la tabla de especificaciones, es decir, si cada uno de los ítems de la prueba responde a los descriptores y es un representante significativo del descriptor.

PSPCRPD 1 Plantea una situación problemática de cantidad que tenga las siguientes características: a) Los datos deben partir de alguna actividad del contexto rural. b) Debe considerar recursos y actores locales. c) Los datos deben inducir la utilización de proporción directa.

99

d) Deben permitir formular preguntas del saber, hacer y ser.

PSPCRPD 2 Plantea situaciones problemáticas de cantidad que tengan las siguientes características: a) Los datos deben partir de alguna actividad del contexto rural. b) Debe considerar recursos y actores locales. c) Los datos deben inducir la utilización de proporción inversa. d) Deben permitir formular preguntas de saber, hacer y ser.

RSPCRPD 3 Ana es comerciante de frutas. En una ocasión, compra cierto número de naranjas: la mitad del total, a 5 por S/ 6, y la otra mitad, a 6 por S/ 7. Y luego vende todas las naranjas a 4 por S/ 7. Si ganó S/ 170: a) ¿Cuántas naranjas compró? b) ¿Cuánto es el precio de costo de cada naranja? c) ¿Cómo sabré el precio de venta de cada naranja? d) Si hubiera vendido a 5 por 8 soles, ¿cuánto hubiera sido la ganancia? e) ¿Qué recomendaciones darías a Ana?

RSPCRPI 4 El señor Bartolomé Guillén, de Satipo, Junín, tiene un terreno de cultivo que produce café, arroz, yuca, papaya, plátano, maní, entre otros productos, de una extensión de 10 hectáreas. Él decide venderlo en un precio, según tasación, de S/ 18 600. Este dinero quiere donarlo a sus cuatro hijos en proporción inversa a las edades, que son 6, 8, 12 y 18 años, respectivamente. a) b) c) d)

¿Qué tipo de magnitud representa el reparto? ¿Cómo haré para saber cuánto le corresponde a cada hijo? ¿Será equitativo la distribución de dicho dinero?, ¿por qué? Si el reparto fuese directamente proporcional a sus edades, ¿sería justo?

100

UNIDAD IV Teoría de números

PRESENTACIÓN DE LA UNIDAD Estimado participante del programa de actualización dirigido a docentes y acompañantes pedagógicos de secundaria en el ámbito rural, en esta oportunidad te presentamos la unidad IV, que tiene como objetivo mejorar tu práctica docente, a partir de la construcción del significado y uso de la teoría de números dentro de un enfoque crítico, reflexivo e intercultural. Este abordaje te permitirá reflexionar sobre tu desempeño en esta noble carrera y sobre la teoría de números, su didáctica y evaluación, para construir herramientas de tu nueva práctica docente, que te fortalecerá, a través de una acertada intervención pedagógica, para desarrollar en tus estudiantes la competencia “Actúa y piensa matemáticamente en situaciones de cantidad”, dentro de un enfoque de resolución de problemas. La estrategia didáctica central de esta unidad está basada en la construcción del significado y uso de la teoría de números, planteando y resolviendo situaciones problemáticas del contexto real, y así construir una matemática funcional, útil para la vida. Esperamos que esta unidad te sirva de apoyo para mejorar tu práctica docente y contribuir al fortalecimiento de tus competencias profesionales en favor de los estudiantes del lugar donde laboras.

ESTRUCTURA DE LOS CONTENIDOS Reflexión desde la práctica Reflexión Teórica Divisibilidad

TEORÍA TEORÍA DE DE NÚMEROS NÚMEROS

Mínimo común múltiplo Máximo común divisor Criterios de divisibilidad Orientaciones metodológicas Evaluación del aprendizaje Herramientas para la nueva práctica Actividades de reflexión individual/equipo, a distancia, de metacognición y autoevaluación Glosario, texto complementario, referencias.

101

1. Reflexión desde la práctica Observa las siguientes situaciones: Situación 1

,

Situación 2

A partir de la observación de las viñetas, responde las siguientes preguntas: a) ¿Cómo se definen divisibilidad, mínimo común múltiplo, máximo común divisor y números primos? b) ¿Crees que la estrategia es pertinente para desarrollar el significado de divisibilidad, mínimo común múltiplo, máximo común divisor y números primos?, ¿por qué? 102

c) ¿Cómo es la relación docente-estudiante? d) ¿Cómo recogerías los saberes previos de los estudiantes, relacionados con el tema tratado? e) ¿Qué estrategia utilizarías para desarrollar teoría de números?

2. Reflexión teórica 2.1. Fundamento teórico 2.1.1. ¿Cómo surgió la teoría de los números? La historia de la matemática nos permite comprender el comportamiento de los números, “buen comportamiento aritmético”: así los pares siempre se alternan con los impares, los múltiplos de tres aparecen cada tres números, los cuadrados perfectos tienen una ley de formación, así como los números triangulares, los cuadrangulares, entre otros. De esta manera, podemos hacer una larga lista. En particular, los números primos son una esencia de la aritmética, dado que no es complicado comprenderlos, ya que solo requieren de un http://acopba.blogspot.pe/2012_08_01_archiv e.html sistema de numeración y conocer las cuatro operaciones básicas; sin embargo, siguen siendo uno de los retos más ¿De cuántas maneras podemos fabulosos de la ciencia. Además, su influencia no solo está determinar la cantidad de borregos? presente en el universo particular de las matemáticas, sino que, aunque no seamos conscientes de ello, los números primos desempeñan un papel decisivo en la vida cotidiana: como en las transacciones bancarias, protección de nuestro ordenador personal, o en conversaciones privadas en la telefonía móvil, etc. En un sentido metafórico, los números primos son como un virus maléfico que, cuando ataca a la mente de un matemático, es muy difícil de erradicar. Euclides, Fermat, Euler, Gauss, Rieman, Ramanujan y una larga lista de matemáticos de los más renombrados de la historia cayeron en sus redes. Algunos consiguieron zafarse de él de manera más o menos exitosa, pero todos ellos sucumbieron a la obsesión de encontrar la “fórmula mágica”, una regla de formación que decidiera cuál es el número primo que sigue a un número cualquiera; sin embargo, ninguno lo consiguió.[CITATION GEO11 \l 10250 ] A lo largo de la historia de las matemáticas, los números primos han ido dejando un extenso rastro de conjeturas. En cierto modo, se podría decir que la historia de los números primos es la historia de un fracaso, pero un fracaso maravilloso, que, durante toda su andadura, ha generado nuevas teorías, nuevos paradigmas, nuevos hitos que han marcado un antes y un después.

¿Qué son los signos del diablo? En las épocas más oscuras de la cultura europea, las cifras eran consideradas como signos misteriosos de una escritura “secreta”, de ahí que, actualmente, se siguen llamando a los mensajes codificados “mensajes cifrados”. Cuando en Europa se introdujeron las primeras cifras árabes en las columnas de los ábacos, los “abacistas” puros volvieron a sustituir por números romanos. No podían permitir la presencia de aquellos “signos diabólicos con los que satanás habían pervertido a los árabes”. Seis siglos después de la muerte del papa Silvestre II, la Iglesia mandó a abrir su tumba para ver si todavía permanecían en ella los demonios que le habían inspirado la ciencia sarracena de los muertos.[CITATION GEO11 \p 13 \l 10250 ].

103

Los números primos son los elementos primordiales con los que se construyen todos los números. La palabra “primo” proviene del latín primus, que quiere decir “primero” y alude al concepto primario, primitivo en el sentido de origen, ya que todos los números se pueden construir a partir de ellos.

¿Los números primos, invento o descubrimiento? National Geographic (2011), en su publicación de los números primos de su colección “El mundo es matemático”, refiere que una vez establecido un sistema de numeración pareciera lógico que la primera propiedad que se detectara en un número fuera la de ser par o impar, un concepto intuitivo e inmediato. El siguiente proceso era la factorización de los números, que lleva a establecer los criterios de divisibilidad. De esta forma, ya se tenía una colección de números controlados por unas pocas propiedades fáciles de establecer. Excepto los números primos. Lo único que se sabía a ciencia cierta es que no pueden ser pares, excepto el dos, el único primo par.

Teorema fundamental de la aritmética Euclides, en el “Teorema fundamental de la aritmética”, dice: “Todo número natural se puede descomponer de forma única como producto de sus factores primos”.

El número 20 se puede expresar así: 2

20=2×2×5=2 ×5

, es lo mismo que

20=5×2×2=5×2

2

¿Cómo descubrir los números primos? 120 2 Para hacer una descomposición en factores primos, el método que se sigue es 60 2 30 2 colocar el número en cuestión a la izquierda de la línea vertical, en el cual se 15 3 calcula si el número es divisible por 2, 3, 5, etc. Es decir, empezando desde el 5 5 número primo más pequeño, hasta que el número de la izquierda quede en la 1 unidad. Entonces, en la columna de la derecha, aparecen los números primos que factorizan al número dado. Fuente: Los números primos un largo camino al infinito. National Geographic, 2011.

Criba de Eratóstenes Eratóstenes (273-194 a. C.), matemático, astrónomo y geógrafo griego, muy conocido por su famosa criba (un procedimiento heurístico para determinar los número primos), fue el primero en medir la longitud de la circunferencia de la Tierra, formulando dos hipótesis muy atrevidas para aquella época.

 La Tierra tiene forma esférica.  Los rayos del Sol son paralelos. Sabía que en el solsticio de verano (21 de junio), en un lugar del río Nilo, Siena (hoy Aswan), los rayos del Sol caían perpendicularmente.[ CITATION Per941 \l 10250 ]. 104

El procedimiento para obtener los primeros cien números primos, propuesto por Eratóstenes, fue: En primer lugar, se construye una tabla de 1 al 100. Se empiezan a eliminar todos aquellos que son múltiplos de dos, 4, 6, 8, 10, … ; después, los que son de tres: 6 (ya eliminado), 9, 12, 15, … ; le seguirán los múltiplos de cinco y luego de siete, y así sucesivamente.

1 11 21 31 41 51 61 71 81

2 12 22 32 42 52 62 72 82

3 13 23 33 43 53 63 73 83

4 14 24 34 44 54 64 74 84

5 15 25 35 45 55 65 75 85

6 16 26 36 46 56 66 76 86

7 17 27 37 47 57 67 77 87

8 18 28 38 48 58 68 78 88

9 19 29 39 49 59 69 79 89

91 92 93 94 95 96 97 98 99

Los números que han quedado sin eliminar son todos primos. Obsérvese que la criba finaliza cuando se llega al número 10, que es la raíz cuadrada de 100. En general, para encontrar los números primos menores que un número N dado, basta con realizar la criba para los números menores o iguales √ N . Esto nos proporciona un método para encontrar primos menores que otro dado. [ CITATION GEO11 \l 10250 ]

10 20 30 40 50 60 70 80 90 10 0

¿Cuáles son sus conceptos y procedimientos? Divisibilidad Si en un corral tenemos 3 cuyes, 6 conejos, 9 gallinas y 12 patos, diremos que todas las cantidades anteriores son divisibles por 3, ya que el cociente de dividir a cada una entre 3 es número entero. Es decir, el número 3 divide exactamente a los números 3, 6, 9 y 12. Dicho de otra forma, los números 3, 6, 9 y 12 son múltiplos del número 3. Si quisiéramos repartir 6 gatos y 5 loros entre 3 personas, no podríamos, debido a que el número 5 no es divisible por 3, es decir, el cociente de dividir 5 entre 3 no es un número entero. Cuando queremos repartir equitativamente objetos entre varias personas, surge la necesidad de utilizar la divisibilidad. Pero esto será posible solamente si el número de objetos a repartir es múltiplo del número de personas al que se repartirá.

 Un número es divisible por otro si y solo si resulta ser su múltiplo.  Llamamos múltiplo de un número al resultado de la multiplicación de dos números. Así diremos que el 3, 6, 9 y 12 son múltiplos de 3 debido a que: 3 × 1 = 3; 3 × 2 = 6; 3 × 3 = 9; 3 × 4 = 12 De lo anterior, podemos decir:

    

12 es múltiplo de 3 3 es divisible por 12 12 contiene exactamente a 3 3 es divisor de 12 3 está contenido exactamente en 12 o

 12 = 3 (se lee 12 es múltiplo de 3)

105

Para que un número sea divisible por otro, el divisor del número tiene que dividir exactamente a su múltiplo, es decir, el cociente debe ser una cantidad entera. Así, los divisores de 12 son: 1, 2, 3, 4, 6, 12. En el conjunto de los números enteros Z, la operación de división no está totalmente definida, o sea, en Z no siempre existe el cociente de dividir dos números enteros. Su existencia está condicionada a la multiplicación, por lo que la división es operación inversa de la multiplicación.

Definición 1: Sean a; b y q números enteros con b ≠ 0. Se dice que q es el cociente de dividir a entre b si a = bq: se denota q = a ÷ b o q = a/b

El cociente de dividir 30 entre 6 es 5; pues 30 = 6 x 5.

Criterios de divisibilidad

 Divisibilidad por 2. Todo número entero par (cuya cifra de unidades es 0; 2; 4; 6; 8) es divisible por 2. Los números 128, 244, 450 son divisibles por 2, debido a que su última cifra es par.

Observación 1 En general, para que un número sea divisible por 2 n, el número formado por sus “n” últimas cifras deberá ser un múltiplo de 2 n; en caso contrario, el número así formado determina el residuo de dividir a entre 2 n.

 Divisibilidad por 3 y 9. Un número entero es divisible por 3 o por 9, si la suma de todas sus cifras es múltiplo de 3 o 9, respectivamente.

El número 2466 es divisible por 3 y por 9 debido a que 2 + 4 + 6 + 6 = 18. 18 es múltiplo de 3 y de 9; por lo tanto, 2466 es divisible por 3 y por 9.

 Divisibilidad por 5. Un número es divisible por 5, si la cifra de las unidades es múltiplo de 5; es decir, si es 0 o 5.

635 y 45 610 son divisibles por 5, porque sus últimas cifras cumplen con la condición anterior.

Observación 2 En general, para que un número sea divisible por 5 n, el número formado por sus “n” últimas cifras deberá ser un múltiplo de 5 n; en caso contrario, el número así formado determina el residuo de dividir a entre 5n. 106

 Divisibilidad por 7. Para saber si un número es divisible por 7, hay que restar el número sin la cifra de las unidades y el doble de la cifra de las unidades. Si el resultado es cero o múltiplo de 7, entonces el número es divisible por 7. Si el resultado es diferente, el número no es divisible por 7.

Determinar si el número 105 es divisible por 7. (10)-(2 ¿ 5)= 0 El número 105 no es divisible por 7.

 Divisibilidad por 11. Un número es divisible por 11, si la diferencia entre la suma de sus cifras de lugar par y la suma de las cifras de lugar impar es múltiplo de 11.

10 582 es divisible por 11, debido a que 1 + 5 + 2 – (0 + 8) = 8 – 8 = 0, y cero es múltiplo de todo 11; por lo tanto, 10 582 es divisible por 11.

Mínimo común múltiplo (MCM)

Repartir 35 cuyes vivos entre 3 campesinas para que los críen sería imposible si damos la siguiente condición: a Orfila, la mitad; a Lucinda, la tercera parte, y a Valentina, la novena parte. Es imposible, porque el número 35 no es múltiplo común de 2, 3, 9, respectivamente. Veamos el siguiente cuadro:

M2

2

4

6



20 22

24

26

28

30

32

34

36

38

M3

3

6

9

12

15 18

21

24

27

30

33

36

39

42

M9

9

18

27

36

45 54

63

72

81

90

99

108 117

126

En el cuadro, observamos que el primer múltiplo común de 2, 3 y 9 es 36. Los primeros múltiplos comunes de 2, 3 y 9 son: 36, 72, 108, 144, y así sucesivamente; pero el menor múltiplo común es el número 36. De lo anterior, definimos el mínimo común múltiplo como el menor múltiplo común de dos o más números. Como podemos observar, para hallar el mínimo común múltiplo, hemos utilizado un cuadro de lista sistemática para cada uno de los múltiplos de 2, 3 y 6; pero es muy laborioso obtener el resultado. Existen otras formas, una de ellas es descomponiendo los números en sus factores primos. Las campanas de las tres capillas del pueblo hoy tocaron simultáneamente. ¿Luego de cuánto tiempo volverán a tocar en el mismo instante, si en la primera capilla toca cada 4 días; en la segunda, cada 8 días, y en la tercera, cada 12 días? Descomponiendo en sus factores primos, tenemos:

4=22

;

8=2

3

;

2

12=2 ×3

De los números primos obtenidos, multiplicamos los que tiene mayor exponente, es decir: 107

3

2 ×3=24

. Luego, las tres capillas tocarán simultáneamente sus campanas a los de 24 días.

Máximo común divisor (MCD)

Fuente:http://www.ensumesa. com/historiasensumesa/tendencias/quesosmaduros-grandes-aliados-entodo-y-para-todo

Fuente:https://todoayacucho.files.w ordpress.com/2010/11/img_0741.jp g

Fuente: http://www.puratossoftrmelting.com/pan-tanta-guagua

Si quisiéramos repartir de forma equitativa 20 tajadas de queso, 24 guaguas y 32 chaplas calientes en máximas porciones, ¿de cuántos sería cada porción? Observemos el siguiente cuadro:

D20

2

4

5

10

20

D24

2

4

6

8

12

D32

2

4

8

16

32

24

Observamos los divisores comunes 2 y 4, de los cuales el máximo es 4; esto quiere decir que cada porción contaría con 4 tajadas de queso, 4 guaguas y 4 chaplas calientes. De lo anterior, definimos que el máximo común divisor de dos o más números es el mayor divisor que los divide simultáneamente sin dejar residuo. Otra forma de obtener el máximo común divisor es a través de la descomposición de los números en sus factores primos. En el caso anterior: 2

20=2 × 5

;

3

24=2 × 3

;

32=2

5

De los números primos obtenidos, multiplicamos los factores comunes con su menor exponente, es 2

decir: 2 =4 . Como podemos observar, también nos da 4. La utilidad del máximo común divisor en la vida real es aplicable en situaciones de reparto. Números compuestos

108

No podríamos repartir 19 huevos a más de dos personas, debido a que tendríamos que romperlos, ya que los únicos números que los dividen exactamente son el 1 y 19. A estos números que solo son divisibles por la unidad y por sí mismos se les llama números primos. Si en vez de 19 tendríamos 20 huevos, es fácil dividirlos exactamente entre más de dos personas, porque el 20 es divisible por 1, 2, 4, 5, 10 y 20. A estos números que tienen más de dos divisores se les llama números compuestos.

Definición:

Sea p ∈ Z− { 0; ± 1 } . Se dice que p esun número primo si sus únicos divisores son ±1 y ± p , esdecir , D p={ ± 1; ±

A través de la historia, ha sido preocupación de muchos matemáticos conocer los números primos o encontrar propiedades que permitan saber si un número entero dado es primo o no. Existen tablas de números primos menores que un número dado en Z+; y la primera conocida es la Criba de Eratóstenes (matemático griego, 276-194 a. C.).

109

2.2. ¿Cuáles son las orientaciones didácticas que emplearías para el desarrollo de los aprendizajes? Los estudiantes deben construir el significado de teoría de números a partir de situaciones problemáticas reales, actuando y pensando matemáticamente durante todo el proceso de la sesión. Para el presente caso, utilizaremos los pasos de Pólya durante la resolución de situaciones problemáticas.

2.2.1. Una carta muy triste Competencia Capacidades Actúa y piensa Elabora y matemáticamente en estrategias. situaciones de cantidad.

Indicadores usa Emplea estrategias heurísticas para resolver problemas de divisibilidad.

INICIO El docente da la bienvenida y reparte la situación problemática a los estudiantes. Situación problemática 1

Félix, un estudiante de un caserío del distrito de Tingo María, recibe una carta que le envió su papá, quien es comerciante y había viajado a Iquitos por vía fluvial: “Hijo mío, no pude estar presente en tu cumpleaños, debido a que hubo un accidente en el barco que viajaba, en él perdí toda mi mercadería de ropas. En ese barco viajábamos 100 personas, de pronto se produjo un naufragio. El capitán nos informó que de los sobrevivientes la onceava parte eran niños y la quinta parte de los muertos eran casados. La gente lloraba mucho al ver a sus familiares fallecidos”. Al leer esta carta, Félix se puso muy triste, pero luego se le vinieron las siguientes preguntas a la mente: ¿Cuántos murieron en total? ¿Cuántos sobrevivieron? ¿No sé qué procedimiento matemático debo realizar para saber los resultados? ¿Qué hubiera hecho si hubiese estado en dicha situación? El docente debe formar equipos de trabajo para que los estudiantes se ayuden entre ellos. Sugerimos realizar la motivación para que los estudiantes estén interesados en recibir la información. También recoger los saberes previos relacionados con la situación problemática. ¿Cuántos de ustedes conoce el río Amazonas? ¿Alguien de ustedes viajó en esos hermosos barcos? Seguidamente, el docente, a través de preguntas, debe generar un conflicto cognitivo en los estudiantes. ¿Por qué números es divisible el número 100? ¿Cómo sé que un número es divisible por otro?

A continuación, el docente debe presentar el propósito de la sesión y la forma como serán evaluados los estudiantes. 110

... de propósito: Los estudiantes construirán el significado y uso de la divisibilidad, a partir de la situación problemática mencionada en la carta. ... de evaluación: Los estudiantes serán evaluados durante el proceso de resolución de la situación problemática, a través de una lista de cotejo. DESARROLLO Para que los estudiantes resuelvan la situación problemática presentada al inicio, recomendamos al docente proponer los 4 pasos de Pólya.

 Primer paso: Comprensión del problema El docente debe pedir a los estudiantes que lean detenidamente la situación problemática, hasta familiarizarse con ella. Para verificar si los estudiantes comprendieron el problema, el docente debe hacer preguntas sobre identificación de datos, condiciones e interrogantes. ¿Qué datos tenemos en la situación problemática de la carta? Los estudiantes responderán: Viajaron 100 personas. Niños: N Muertos: M Casados: C Sobrevivientes: S ¿Qué condiciones tenemos? El docente debe inducir a los estudiantes para que respondan de la siguiente forma: El número de niños es la onceava parte de los sobrevivientes. El número de casados es la quinta parte de los muertos Las operaciones con los datos deben ser números enteros. ¿Qué interrogantes tenemos? Los estudiantes responderán: ¿Cuántos murieron en el naufragio? ¿Cuántos sobrevivieron? ¿Qué procedimiento matemático debes realizar para saber los resultados? ¿Qué consejos le darías al padre?

 Segundo paso: Configurar un plan El docente debe pedir a los estudiantes que propongan estrategias, tomando como punto de partida los datos, las condiciones y las interrogantes, e induciendo al descubrimiento de estrategias a través de las siguientes preguntas: ¿Alguna vez resolvieron un problema parecido? ¿Observan relación entre los datos? ¿Identifican relación entre los datos y la incógnita? Si los estudiantes aún no construyeron sus estrategias, el docente debe proporcionar, a cada grupo, un problema parecido, en el cual deben identificar las estrategias utilizadas.

111

Los estudiantes descubrieron que podemos formar ecuaciones con los datos.

Utilizar ecuación: M + S = 100; N =

S 11

M 5

; C=

En un cuadro, hacer una lista sistemática de los múltiplos del número de casados y muertos, hasta encontrar el menor múltiplo común de 5 y 11.

 Tercer paso: Ejecutar el plan Se debe pedir a los estudiantes que desarrollen cada una de las estrategias propuestas, teniendo en cuenta los datos, condiciones y preguntas. En este proceso, el docente debe estar atento para que todos los grupos trabajen.

M: muertos; S: sobrevivientes N: niños; C: casados Estrategia de ecuaciones: M + S = 100 Además Como la cantidad de niños (N) es entera, el cociente de la cantidad de sobrevivientes (S) y 11 debe ser entero. Es decir, S debe ser divisible por 11. Simbólicamente: 0

S=11 Como la cantidad de casados (C) es entera, el cociente de la cantidad de muertos (M) y 5 debe ser entero. Es decir, M debe ser divisible por 5. Simbólicamente: 0

M =5 Estrategia de lista sistemática de los múltiplos de 11 y 5, respectivamente. S M

0

11 0

5

11

22

33

44

55

5

10

….

40

45

De la lista anterior, obtenemos como resultados: S = 55 y M = 45, debido a que ambas cantidades suman 100.



Examinar la solución obtenida

112

El docente debe pedir a los estudiantes que verifiquen sus resultados, que se cercioren si cometieron algún error en su proceso y, además, que propongan nuevas formas de resolver el mismo problema.

Los estudiantes leen nuevamente el problema y la pregunta, para tener claro lo que se les pide. Finalizados los cuatro pasos de Pólya, el docente deja a los estudiantes que construyan otra situación problemática parecida, que tenga datos, condiciones, interrogantes y contexto. Finalmente, los estudiantes en grupo deben socializar sus resultados para que validen su trabajo.

CIERRE El docente debe sistematizar los conocimientos trabajados en la sesión para que los estudiantes consoliden la información. Seguidamente, debe generar, en los estudiantes, reflexión metacognitiva de todo el proceso a través de preguntas. ¿En qué medida te ayudaron los pasos de Pólya a resolver la situación problemática? ¿En qué otras situaciones utilizarás divisibilidad? ¿Qué dificultades tuviste en el proceso? El proceso de la sesión debe ser registrado por los mismos estudiantes a través de una ficha de coevaluación.

Criterios

Identifica datos, condiciones e interrogantes.

Lista de cotejo Plantea estrategias de resolución de problemas.

Utiliza divisibilidad en la resolución de problemas.

Desarrolla la ecuación en el proceso de resolución.

Estudiantes Sí

No



No



No



No

2.2.2. Los vendedores de frutas Competencia Capacidades Actúa y piensa Elabora y matemáticamente en estrategias. situaciones de cantidad.

Indicadores usa Emplea el MCD y el MCM para resolver problemas de traducción simple y compleja.

INICIO El docente debe dar la bienvenida y entregar la situación problemática a los estudiantes. Situación problemática

113

Esteban y Hugo viajan constantemente a Huaraz a vender las frutas que produce su comunidad. Esteban viaja cada 18 días y Hugo, cada 24 días. Hoy en la tarde Esteban viajará, por lo que deben llenar 900 manzanas y 630 naranjas en distintas cajas, con la condición de que cada caja contenga la mayor cantidad de manzanas o naranjas; además, los cajones deben contener la misma cantidad de frutas. a) Esteban manifiesta: Si yo viajo hoy, ¿dentro de cuántos días ambos estaremos en Huaraz? b) ¿Qué debo hacer para saber cuántas frutas debe contener cada caja? c) ¿Cuántas cajas son necesarias para llenar todas las frutas? d) ¿Qué harían ustedes con el dinero del negocio? Describiremos la sesión, en el cual trabajaremos utilizando los 4 pasos de Pólya.

El docente debe formar equipos de trabajo para que los estudiantes se ayuden entre ellos. Sugerimos realizar una actividad de motivación para despertar el interés de los estudiantes y, seguidamente, recoger los saberes previos para trabajar en la zona de desarrollo próximo. ... de motivación: El docente desarrolla su clase explicando a los estudiantes sobre la producción de frutas y verduras. Asimismo, pregunta: ¿Quién de ustedes conoce Huaraz? ... de recojo de saberes previos: ¿Qué producen Esteban y Hugo? Si una canasta contiene 12 manzanas y 18 naranjas, ¿cuántos divisores comunes tienen ambas cantidades? El docente, por medio de preguntas, debe generar el conflicto cognitivo, que será resuelto durante la sesión. ...s de conflicto cognitivo: ¿Cuál es el máximo común divisor de 12 y 18? ¿En qué casos de la vida cotidiana se utiliza el máximo común divisor? ¿En qué casos de la vida cotidiana se utiliza el mínimo común múltiplo?

A continuación, el docente debe presentar el propósito de la sesión y la forma como serán evaluados los estudiantes. ... de propósito: Los estudiantes construirán el significado y uso del máximo común divisor, a partir de la situación problemática relacionada con la producción de frutas y hortalizas en la chacra. ... de evaluación: Los estudiantes serán evaluados durante el proceso de resolución de la situación problemática, a través de una lista de cotejo que estará pegada en la pizarra.

DESARROLLO 114

En esta etapa, los estudiantes construirán sus conocimientos resolviendo la situación problemática. Para este efecto, el docente debe sugerir a los estudiantes que utilicen los 4 pasos de Pólya hasta que se empoderen.



Comprensión del problema

El docente debe pedir a los estudiantes que lean hasta entender la situación problemática. Posteriormente, verificará si comprendieron el problema y si identificaron los datos, las condiciones, y las interrogantes. a) Los estudiantes deben expresar con sus propias palabras la situación problemática y, seguidamente, contestar la pregunta: ¿Qué datos tiene el problema? El docente inducirá a los estudiantes a responder: Esteban viaja a Huaraz cada 18 días. Hugo viaja a Huaraz cada 24 días. 900 manzanas y 630 naranjas

b) ¿Qué condición muestra el problema? El docente inducirá a los estudiantes a responder: Cada caja debe contener la mayor cantidad de manzanas o naranjas; además, las cantidades de frutas en cada cajón deben ser iguales. c) ¿Qué interrogantes aparecen en el problema? El docente inducirá a los estudiante a responder: Esteban manifiesta: Si yo viajo hoy, ¿dentro de cuántos días ambos estaremos en Huaraz? ¿Qué debo hacer para saber cuántas frutas debe contener cada caja? ¿Cuántas cajas son necesarias para llenar todas las frutas? ¿Qué harían ustedes con el dinero del negocio?



Configuración del plan

El docente debe pedir a los estudiantes que propongan estrategias, tomando como punto de partida los datos, las condiciones y las interrogantes, e induciendo al descubrimiento de estrategias a través de las siguientes preguntas: ¿Alguna vez resolvieron un problema parecido? ¿Observan relación entre los datos? ¿Identifican relación entre los datos y la incógnita? Si los estudiantes aún no construyeron sus estrategias, el docente debe proporcionar, a cada grupo, un problema parecido de menor dificultad, para que identifiquen las estrategias más pertinentes. Respecto a la primera pregunta, tenemos la estrategia de lista sistemática, que nos permitirá ubicar, en un cuadro de doble entrada, cada cuántos días viaja Esteban; seguidamente, escribiremos sus múltiplos. Lo mismo haremos en el caso de Hugo. Pintaremos de color diferente el mínimo múltiplo de 18 y 24 que es común a ambos. Respecto a la segunda y tercera pregunta, utilizaremos la estrategia de lista sistemática, que permitirá, en un cuadro de doble entrada, colocar los divisores correspondientes a número de 115

manzanas y número de naranjas, y pintaremos con otro color el mayor divisor común entre ambos (900 y 630, respectivamente).



Ejecución del plan

Se debe pedir a los estudiantes que desarrollen cada una de las estrategias propuestas, teniendo en cuenta los datos, condiciones y preguntas. En este proceso, el docente debe estar atento para que todos los grupos trabajen. a) Para la primera pregunta, elaboramos un cuadro de doble entrada (estrategia: hacer una lista sistemática), en el que colocamos los múltiplos correspondientes en cada fila y consideramos como respuesta el primer número común entre ambos.

Esteban

18

36

54

72

90

108



Hugo

24

48

72

96

120

144



Podemos observar que el mínimo común múltiplo de ambos, de 18 y 24, es 72. b) Respecto a la segunda pregunta, también elaboramos un cuadro de doble entrada, en el que colocamos los divisores correspondientes en cada fila y consideramos como respuesta el mayor divisor común entre ambos.

Manzanas 900

2



20

25

50

75

90



Naranjas 630

2



21

30

35

45

90



Podemos observar que el máximo común divisor de 900 y 630 es 90. c) Respecto a la tercera pregunta. Si dividimos 900 entre 90, tendremos la cantidad de cajas de manzanas, es decir, 10 cajas. Si dividimos 630 entre 90, tendremos la cantidad de cajas de naranjas, es decir, 7 cajas. Finalmente, para las manzanas y naranjas se necesitaron 17 cajas.



Examinar la solución obtenida

Debemos verificar si todo el proceso es correcto. Leer nuevamente la pregunta y responder. a) Respecto a la primera pregunta, Hugo y Esteban se encontrarán en Huaraz a los 72 días. b) Respecto a la segunda pregunta, en cada caja ingresarán 90 frutas, es decir, tendremos cajas de 90 manzanas y cajas de 90 naranjas. Para obtener el resultado, hemos utilizado el mcd. 116



Mirada retrospectiva del proceso

Buscar nuevas formas de resolver. Respecto a la primera pregunta, otra forma de resolver sería a partir de la descomposición en sus números primos: 2

18=2×3 3

24=2 ×3 Luego el mcm (18, 24) =

3

2

2 ×3

= 72

Esto indica que se encontrarán dentro de 72 días… Respecto a la segunda pregunta, otra forma de resolver sería a partir de la descomposición en sus factores primos: 2

2

2

900 =

2 ×3 ×5

630 =

2×3 ×5×7

2

Luego el MCD (900, 630) =

2

2×3 ×5

= 90

Es decir, tendremos cajas de 90 manzanas y cajas de 900 naranjas. Respecto a la tercera pregunta Existe una forma práctica para sacar el mcd de dos números:

En desarrollo adjunto, observamos la cantidad de cajas de manzanas (10) y la cantidad de cajas de naranjas (7), en total 17 cajas. El mcd se obtiene multiplicando 2  3  3  5 = 90

Observación en el esquema: los números 2, 3, 5, 7 son números primos. El docente debe proponer a los estudiantes resolver una situación problemática utilizando los 4 pasos de Pólya. (Aplicación) A los estudiantes, se les debe pedir que formulen un problema con las siguientes características: Datos, condiciones, interrogantes, contexto. (Extensión)

117

Observación: Durante todo el proceso, el docente debe actuar como facilitador de la construcción del aprendizaje y estar atento a que todos los estudiantes estén desarrollando el problema.

CIERRE El docente debe sistematizar el conocimiento trabajado en la sesión para que los estudiantes consoliden la información. También debe generar, en los estudiantes, reflexión metacognitiva de todo el proceso a través de preguntas.

¿En qué medida te ayudaron los pasos de Pólya a resolver la situación problemática? ¿En qué otras situaciones utilizarás mínimo común múltiplo y máximo común divisor? ¿Qué dificultades tuviste en el proceso? El proceso de la sesión debe ser registrado en una ficha de evaluación. Criterios

Identifica datos, condiciones e interrogantes.

Plantea estrategias de resolución de problemas.

Estudiantes Sí

No



No

Utiliza los criterios de divisibilidad en la resolución de la situación problemática. Sí No

Resuelve el problema empleando criterios de divisibilidad. Sí

No

Evaluación de teoría de números EVALUACIÓN FORMATIVA: Instrumento (escala descriptiva). Técnica (observación sistemática) CRITERIOS

Siempre

Participa en el trabajo cooperativo. Hace preguntas durante el proceso. Argumenta con claridad sus resultados. Muestra dificultades durante el proceso de generalización. Se empoderó del método de Pólya. Demuestra curiosidad, asombro e interés. Permite que todos los miembros de su equipo participen.

118

Muchas veces

Algunas veces

Nunca

EVALUACIÓN SUMATIVA: Instrumento (lista de cotejo). Técnica (observación sistemática) Competencia Actúa

y

matemáticamente situaciones de cantidad.

Capacidades piensa en

Elabora y usa estrategias.

Indicadores Emplea el MCD y MCM para resolver problemas.

3. Herramientas para la nueva práctica 3.1. Actividades de reflexión individual a) b) c) d) e) f)

¿Cómo cumplirás tus horas pedagógicas? ¿Cómo relacionarás el contexto con la sesión de aprendizaje? ¿Con qué características construirás el propósito de una sesión de aprendizaje? ¿Qué recomendaciones darás para que los estudiantes trabajen cooperativamente? ¿Qué estrategias utilizarás para trabajar con estudiantes inclusivos? ¿Qué materiales disponibles de la zona donde laboras utilizarás para la construcción del significado y uso de la divisibilidad, mcm, mcd y números primos? g) ¿En qué situaciones reales y de contexto, emplearás divisibilidad, mcm, mcd y números primos? h) ¿Cómo abordarás las definiciones de divisibilidad, mcm, mcd y números primos?

3.2. Actividades a distancia A continuación, se presentan actividades para poner en práctica lo desarrollado en esta unidad, las cuales serán ejecutas en el componente a distancia.

a) Crea una situación problemática de acuerdo a tu contexto intercultural que implique desarrollar conocimientos relacionados con teoría de números. b) Selecciona una competencia, capacidades, indicadores, estrategias, recursos y materiales (actores y/o escenarios) a partir de una situación problemática. c) Elabora un diseño de sesión de aprendizaje, tomando en cuenta la actividad anterior. d) Ejecuta la sesión elaborada y recoge las evidencias correspondientes para que sean adjuntadas al portafolio (fotos, videos, audios, etc.). e) Archiva el diseño de la sesión en su portafolio.

3.3. Actividades metacognitivas a) ¿Es necesario recoger los saberes previos de los estudiantes? b) En la competencia, ¿qué significado tienen “actúa” y “piensa”? c) ¿Cómo aprovecharás el aporte de los sabios de la comunidad, las actividades comunales y el calendario comunal en el desarrollo de tu práctica pedagógica? d) ¿Qué aspectos del módulo fueron aprendidos con mayor rapidez? ¿Por qué crees que fue así? e) ¿Dedicas suficiente tiempo para desarrollar con tus estudiantes el contenido disciplinar de la teoría de números? f) ¿Qué estrategia utilizaste para participar activamente en el proceso? 119

g) ¿Qué lecciones aprendidas puedes mencionar? h) ¿Cuál es el rol protagónico que asumirás en tu institución educativa?

120

3.4.

Autoevaluación

Indicaciones. Estimado docente, evalúa tu participación durante el taller. Apellidos y nombres: ……………………………………………………. N.o 1 2 3 4 5

Descriptores



No

Participé activamente en las actividades indicadas durante el desarrollo de la sesión. Escuché con atención las indicaciones y respeté las normas de trabajo señaladas. Cumplí con el tiempo previsto en las actividades señaladas. Respeté a mis colegas y contribuí en las actividades de equipo. Me interesé en el tema y aclaré mis dudas.

4. Glosario Cifras significativas. Son todos los números del 1 al 9. El cero también es cifra significativa, si no se encuentra al inicio del número. Conflicto cognitivo. Reconceptualización que se genera en el estudiante. Contexto rural. Relacionado con la vida en el campo. Material no estructurado. Material de contexto, reciclable. mcd. Máximo común divisor. mcm. Mínimo común múltiplo

5. Texto complementario Conteo y cálculo en el Imperio incaico Si bien en la civilización del Imperio inca no se llegó a desarrollar la escritura y, en consecuencia, se carecía de la posibilidad de guardar registros escritos, eso no fue impedimento para que desarrollasen una manera de registrar cantidades y de representar números. Antes de empezar a hablar del sistema de numeración decimal posicional que emplearon los incas, debe tenerse en cuenta que este imperio se fundó hacia el 1250 d. C. y que hacia el 1532 (antes de la conquista española) era extenso y vasto, ocupando un territorio que abarcaba parte de lo que hoy conocemos como Colombia, Ecuador, Perú, Bolivia, Chile y Argentina. El sistema de numeración inca: el quipu. Como se indicó anteriormente, no se desarrolló una escritura por parte de los incas, pero sí se les presentó la necesidad de contar objetos y de registrar la información numérica que iban obteniendo. Para ello, tuvieron que desarrollar una forma de consignar cantidades sin escribirlas. Así, crearon unos instrumentos que servían para registrar y almacenar números en ellos, los cuales consistían en unos conjuntos de cuerdas con nudos que se denominaban quipus. Estos tenían un papel primordial en la administración del Imperio inca, pues era el único instrumento de que disponían para almacenar cualquier tipo de información numérica. A continuación, se indicará cómo se construían y se interpretaban los quipus. Construcción e interpretación de un quipu. Gran parte de la información de que se dispone acerca de los quipus se debe a una carta de Felipe Guamán Poma de Ayala [1936] al rey de España, en la que aparecían dibujados varios de estos instrumentos. Un quipu consiste en un conjunto de cuerdas dispuestas de cierta manera y en las que se hacen una serie de nudos. A la hora de construirlo, debía tenerse en cuenta que se empleaban diferentes tipos de cuerda, cada una de las cuales tenía al menos dos hebras, de modo que un extremo acababa en forma de lazo y el otro en punta con un pequeño nudo. Según la disposición que presentase una cuerda, esta podía ser de uno de los tipos siguientes: • Cuerda principal: es la más gruesa de todas y de la que parten directa o indirectamente todas las demás. 121

• Cuerdas colgantes: son todas las cuerdas que penden de la principal hacia abajo. • Cuerdas superiores: son cuerdas que se enlazan a la principal, pero dirigiéndolas hacia arriba. • Cuerda colgante final: es una cuerda cuyo extremo en forma de lazo está unido y apretado al extremo de la cuerda principal. Era opcional, por lo que no aparecía en todos los quipus. • Cuerdas secundarias o auxiliares: son cuerdas que se unen a cualquiera de las que están enlazadas a la principal. A las cuerdas auxiliares se les podía, a su vez, unir otra cuerda auxiliar. Esta se ataba a la mitad de la cuerda de la que procedía. Eugenio, M., Martel, F. y Tenorio, Á. Villalón. Sevilla, España: Universidad Pablo de Olavide.

6. Referencias BOURBAKI, N. (1969) Elementos de la historia de la Matemáticas. España: Alianza Editorial. MARIANO, P. (1994). Historia e historia de las Matemáticas. México: Grupo Editorial Iberoamérica. NATIONAL, G. (2011). Los números primos. España: RBA Coleccionables, S. A.

122

Anexo

EVALUACIÓN DE LA UNIDAD IV

Tabla de especificaciones La tabla de especificaciones es un instrumento de evaluación que contiene información sobre el contenido esencial de lo que se va a evaluar en esta unidad.

Funciones de la tabla de especificaciones La tabla de especificaciones sirve para: 1. Construir los ítems del instrumento de evaluación. 2. Orientar la interpretación de los resultados.

Tabla de especificaciones CÓDIGO

DESCRIPTORES

PSPCRD 1

Plantea situaciones problemáticas de cantidad relacionadas con divisibilidad, considerando recursos, actores y escenarios rurales Plantea situaciones problemáticas de cantidad relacionadas con mínimo común múltiplo, considerando recursos, actores y escenarios rurales. Plantea situaciones problemáticas de cantidad relacionadas con máximo común divisor, considerando recursos, actores y escenarios rurales. Resuelven situaciones problemáticas de cantidad relacionadas con divisibilidad, en un contexto intercultural, utilizando los cuatro pasos de Pólya y describiendo las estrategias empleadas en cada una de las fases. Resuelven situaciones problemáticas de cantidad relacionadas con mínimo común múltiplo, en un contexto intercultural, utilizando los cuatro pasos de Pólya y describiendo las estrategias empleadas en cada una de las fases. Resuelven situaciones problemáticas de cantidad relacionadas con máximo común divisor, en un contexto intercultural, utilizando los cuatro pasos de Pólya y describiendo las estrategias empleadas en cada una de las fases.

PSPCRMCM 2 PSPCRMCD 3 RSPCRD 4

RSPCRMCM 5

RSPCRMCD 6

TOTAL

N.° DE ÍTEMS 1 1

1 1 1

1

6

123

Pruebas de desempeño A diferencia de las pruebas de opción múltiple que son instrumentos de evaluación cerrados, las pruebas de desempeño son instrumentos de evaluación abiertos que permiten evaluar los procesos de planteamiento y resolución de diversas situaciones problemáticas de cantidad.

Consistencia de las pruebas de desempeño Una prueba de desempeño es consistente si corresponde a la tabla de especificaciones, es decir, si cada uno de los ítems de la prueba responde a los descriptores y es un representante significativo del descriptor.

PSPCRD 1 Plantea situaciones de cantidad que tengan las siguientes características: a) Los datos deben partir de alguna actividad de contexto rural. b) Deben considerar recursos y actores locales. c) Deben inducir a la construcción del significado de divisibilidad. d) Deben permitir formular preguntas del saber, hacer y ser.

PSPCRMCM 2 Plantea situaciones de cantidad que tengan las siguientes características. a) Los datos deben partir de alguna actividad de contexto rural. b) Deben considerar los recursos y actores locales. c) Deben inducir a la utilización de mcm. d) Deben permitir formular preguntas del saber, hacer y ser.

PSPCRMCD 3 Plantea situaciones de cantidad que implique utilizar máximo común divisor. a) b) c) d)

Los datos deben partir de alguna actividad de contexto rural. Deben considerar recursos y actores locales. Deben inducir a la utilización de mcd. Deben permitir formular preguntas del saber, hacer y ser.

RSPCRD 4 Un barco que transportaba 180 personas sufre un naufragio cerca de Nauta. De los sobrevivientes, 2/5 fuman, 3/7 son casados y los 2/3 son ingenieros. a) ¿Cuántas personas murieron en dicho accidente? b) ¿Cuántos fuman? c) ¿Cómo sabré cuántos son ingenieros? d) En situaciones de naufragio, ¿qué actitud asumirías?

RSPCRMCM 5 Tres campesinas se asocian para producir flores en Tarma. En la medida que va saliendo la cosecha, Ana lleva sus productos a Huancayo cada 6 días, Juana viaja cada 12 días y Luisa, cada 18 días. Hoy, 29 de octubre, las tres amigas han coincidido en Huancayo. a) ¿Dentro de cuántos días volverán a coincidir Juana y Luisa? b) ¿Qué haré para saber dentro de cuántos días volverán a coincidir las tres? c) ¿Por qué las amigas optaron por el negocio de flores?

RSPCRMCD 6 Las dimensiones de un terreno rectangular son 894 m y 354 m. Se desea parcelarlo en terreno cuadrados, de tal modo que no sobre nada y se obtenga el menor número de parcelas. a) ¿Cuántas parcelas cuadradas resultarán? b) ¿Cómo sabré las dimensiones de cada parcela? c) ¿Es recomendable parcelar un terreno? 124

UNIDAD V Modelos financieros

PRESENTACIÓN DE LA UNIDAD Estimado participante del programa de actualización dirigido a docentes y acompañantes pedagógicos de secundaria en el ámbito rural, en esta oportunidad te presentamos la unidad V, que tiene como objetivo mejorar tu desempeño profesional, a partir de la construcción del significado y uso de los modelos financieros, dentro de un enfoque crítico, reflexivo e intercultural. Así pues, esta unidad te permitirá reflexionar sobre la práctica docente, sobre el interés simple y compuesto, su didáctica y evaluación, para construir herramientas orientadas a una nueva práctica pedagógica, que favorezca en tus estudiantes, a través de una acertada intervención, el desarrollo la competencia “Actúa y piensa matemáticamente en situaciones de cantidad”, dentro de un enfoque de resolución de problemas. La estrategia didáctica central de esta unidad está basada en la construcción del significado y uso del interés simple y compuesto, planteando y resolviendo situaciones problemáticas del contexto real, para así construir una matemática funcional, útil para la vida. Esperamos que esta unidad te sirva de apoyo para mejorar tu desempeño docente y contribuir al fortalecimiento de tus competencias profesionales en favor de los estudiantes del ámbito donde laboras.

CONTENIDOS DE LA UNIDAD Reflexión desde la práctica

Reflexión teórica

M ODELOS FINANCIEROS

Interés simple

Interés compuesto

Orientaciones didácticas

Evaluación de los aprendizajes

Herramientas para la nueva práctica Actividades de reflexión individual/equipo, a distancia, de metacognición y autoevaluación. Glosario, texto complementario, referencias .

125

1. Reflexión desde la práctica Observa la siguiente situación en las viñetas:

Atiende, se va a molestar

Profe, va a revisar, que dejó de tarea.

Estoy segura de que en este examen desaprobaré

A partir de la observación de las imágenes, responde las siguientes preguntas: a) b) c) d)

¿Qué conocimientos crees que el docente está desarrollando según las imágenes? ¿Qué estrategia está utilizando el docente para desarrollar interés simple y compuesto? ¿Cómo evalúa el interés simple y compuesto? ¿Qué estrategia de aprendizaje empleas al desarrollar interés simple y compuesto?, explica brevemente. b) ¿Cómo evalúas el interés simple y el interés compuesto? ¿Qué técnicas e instrumentos utilizas al desarrollar interés? 126

2. Reflexión teórica 2.1.

Fundamento teórico

2.1.1. ¿Cómo surgió el interés a través de la historia? Roma es considerada como uno de los grandes imperios de la historia, no solo por su poderío militar, sino también por una serie de leyes que iban alineados a su aparato burocrático. En esta época, aunque se conocía el concepto de tipo de interés, su uso no estaba regulado, dejando al libre albedrío de los contratantes tanto la tasa como muchos otros aspectos. En esta época, si una familia no podía pagar sus deudas, podía ser castigada con la esclavitud o incluso la muerte. Con el final del Imperio romano, el emperador Justiniano realizó un intento de regular el negocio de los préstamos, sobre todo en lo tocante a los tipos de interés, en un intento de evitar que estos fueran demasiado abusivos. Sin embargo, con el final del imperio y el advenimiento del cristianismo, las reglas cambiaron radicalmente. El interés en la Edad Media. La religión cristiana no veía con buenos ojos los préstamos con interés; de hecho, los consideraba casi un pecado (la usura), algo poco digno de un cristiano. Su punto de vista era justificado con el hecho de que la creación de algo desde la nada, es decir, dinero obtenido por el interés, era algo poco cristiano. Debido a esto, solo quien no perteneciera a esta religión, podría prestar “sin cargo de conciencia”. Así sabemos que el préstamo con interés y la usura son actividades de origen muy remoto. Por el contrario, el origen de la letra de cambio es mucho más reciente, pues surge Fuente http://www.taringa.net/posts/info/1646435/Historia-De-La-Usura.html hacia el siglo XII al hacerse cada vez más complicadas las transacciones comerciales y establecerse la práctica de pagar mediante un documento escrito, bajo promesa de una cantidad determinada, en un lugar distinto de aquel en el cual se contrae la deuda. El pago se efectuaba, o bien al representante del acreedor o bien mediante un representante del deudor (GALDÓS, 2005, p. 311). Con la intervención del cristianismo y la rigidez eclesiástica, los judíos eran casi los únicos que podían dedicarse al negocio de los préstamos, dando origen al sistema de bancos. Es así que, en la sociedad babilónica y en el resto de sociedades agrarias del Medio Oriente, estaba estipulada la costumbre del "jubileo" de las deudas, es decir, un periodo donde se daba una cancelación de las deudas pendientes, normalmente a cada generación (unos 20 o 30 años), y que solía coincidir con la coronación de un nuevo rey. Se trataba, en suma, de "un nuevo principio" donde los hijos se veían libres de las deudas.

127

2.1.2. Conceptos y procedimientos Operación financiera Una persona con más dinero del que necesita puede esconderlo bajo el colchón o depositarlo en una cuenta de cheques. De esta manera, el dinero estará siempre disponible para gastarlo, pero no aumenta su cuantía. Si, por el contrario, a alguien más se permite su uso, el prestamista experimenta la agradable sensación de verlo crecer. Por otra parte, el deudor, a pesar de pagar intereses por su utilización, puede satisfacer sus necesidades económicas; adquirir algún bien tangible (como, por ..., una casa), o bien utilizarlo en una empresa financiera que no solo pague el préstamo, sino que también permita obtener beneficios monetarios adicionales. El crédito representa en nuestros días un estilo de vida y un medio necesario en la mayor parte de los negocios, por lo que resulta importante utilizarlo de manera racional. Su uso apropiado requiere conocer su costo real y la capacidad para analizar su rentabilidad. Entonces, toma una decisión: o guarda en un banco o presta con intereses o simplemente lo guarda o lo gasta.

Del trueque a la usura

Fuente:http://lorgiohistoriaygeografiadigital.blogspot.pe/2 013/02/el-comercio-en-el-imperioinca-existio.html

Interés (I) Es la ganancia o beneficio que nos produce un capital al ser prestado a ciertas condiciones de tiempo y tasa preestablecidas.[ CITATION CVE05 \l 10250 ]

Tasa de interés (R%) Nos indica la parte del capital que se obtendría de interés y se expresa como un porcentaje por unidad de tiempo.

Monto (M) Es la suma obtenida añadiendo el interés al capital. M: Monto C: Capital I: Interés simple

M=C+I

INTERÉS SIMPLE El interés simple se cancela sobre el capital inicial que permanece invariable; en consecuencia, el interés que se obtiene en cada intervalo unitario de tiempo es siempre el mismo. Puede afirmarse también que el interés simple es la ganancia solo del capital (cantidad de dinero prestada) a una tasa de interés, durante todo el tiempo que dure el préstamo. El Sr. Ambrosio le presta al Sr. Cecinio la suma de S/ 2000 a una tasa de interés simple del 10 % mensual. Si el préstamo duró 3 meses, entonces:  Interés del primer mes :  Interés del segundo mes :  Interés del tercer mes :

10% 2000 = 200 10% 2000 = 200 10% 2000 = 200

Por lo tanto, el interés por los 3 meses será: 128

200 + 200 + 200 = 600 Los elementos que, además, podemos identificar son: C = 2000; r% = 10 % mensual; t(tiempo) = 3 meses Fórmula general I = C . r%. t r% y t deben estar expresados en las mismas unidades. Esto quiere decir que el interés simple que genere el capital invertido será igual en todos los periodos de duración de la inversión, siempre que la tasa y el plazo no varíen. INTERÉS COMPUESTO Es la ganancia que se saca de la cantidad de dinero que se presta, la cual se llama capital. La tasa de interés determina el porcentaje a cobrar, generalmente anual, por uso del capital o cantidad de dinero prestado. “El interés compuesto es la ganancia de una capital al que se van agregando sus réditos para que produzcan otros”.[ CITATION FES05 \l 10250 ]

M (n) =P (1+i )n Donde M = monto o valor futuro P = capital o valor actual n = número de periodos i = tasa de interés (mensual, trimestral, semestral, etc.)

2.2. ¿Cuáles son las orientaciones didácticas que emplearías para el desarrollo de los aprendizajes? A continuación, se realiza la explicación del desarrollo de una competencia a partir de situaciones problemáticas, utilizando orientaciones metodológicas, materiales, medios e instrumentos para la verificación del logro de los aprendizajes.

2.2.1. Buscando la mejor opción

Competencia Capacidades Actúa y piensa Matematiza matemáticamente en situaciones. situaciones de cantidad.

Indicadores Compara y contrasta modelos de tasas de interés simple al vincularlos con situaciones de decisión financiera.

El docente debe dar la bienvenida y repartir la situación problemática a los estudiantes.

INICIO Situación problemática 1

Un comunero del pueblo de Chiquián, de la provincia de Bolognesi, región Áncash, establece una conversación con el profesor Alejandro, a quien le manifiesta que tiene la necesidad de obtener un préstamo de S/ 3600 para mejorar su negocio de crianza de conejos. Por ello, el comunero requiere 129

orientación sobre una mejor opción, ya que dispone de tres propuestas diferentes de prestamistas y no sabe por cuál decidir. a) El primer prestamista ofreció prestar 3600 soles, con una tasa anual de 6 % pagadero en tres años. b) El segundo prestamista ofreció 3600 soles, con una tasa anual del 6 % pagadero en 8 meses c) El tercer prestamista ofreció prestar 3600 soles, con una tasa anual del 6 % pagadero en 15 días. ¿Cómo sabré a cuánto asciende cada propuesta? ¿Cuál de las propuestas le conviene? ¿Crees que son justos los intereses de las tres propuestas?

El docente debe formar equipos de trabajo heterogéneos para que se ayuden durante la sesión; seguidamente, motivar a los estudiantes por medio de temas relacionados con la situación problemática. Pueden dirigirse a la granja para que el mismo comunero les cuente sus dudas y les hable de sus éxitos en la crianza de conejos. Seguidamente, el docente debe recoger los saberes previos, con el objetivo de trabajar dentro de la zona de desarrollo próximo de los estudiantes. Cuando realizas préstamos, ¿qué pasa con el valor de tu dinero? ¿Qué idea tienes sobre interés? A continuación, el docente debe generar conflicto cognitivo en sus estudiantes a través de preguntas: ¿Qué modelo matemático controla el comportamiento del interés simple? ¿Por qué a mayor tiempo el interés aumenta? Luego debemos mencionar el propósito de la sesión, para que los estudiantes sepan lo que se pretende conseguir. Construir el significado y uso del interés simple a través de una situación problemática. Asimismo, se debe señalar a los estudiantes cómo serán evaluados, para lo cual brindarán información acerca de qué instrumentos y técnicas de evaluación se utilizarán. Instrumento de evaluación: ficha de coevaluación Técnica de evaluación: observación DESARROLLO El docente debe mencionar a los estudiantes qué estrategia general utilizarán en la resolución de problemas. Durante el desarrollo de la situación problemática, sugerimos utilizar los cuatro pasos de Pólya. 

Comprender el problema 130

El docente debe pedir a los estudiantes que lean analíticamente el problema, hasta familiarizarse con él. Para cerciorarse de que se entendió, debe hacer preguntas relacionadas con identificación de datos, condiciones e interrogantes. ¿Qué datos identificas en el problema? a) El primer prestamista ofreció 3600 soles, con una tasa anual de 6 % pagadero en tres años. b) El segundo prestamista ofreció 3600 soles, con una tasa anual del 6 % pagadero en 8 meses. c) El tercer prestamista ofreció 3600 soles, con una tasa anual del 6 % pagadero en 15 días. ¿Qué condiciones identificas en el problema? Pagaderos en 3 años. Pagaderos en 8 meses. Pagaderos en 15 días. ¿Qué interrogantes encontramos? a) ¿Cómo sabré a cuánto asciende cada propuesta? b) ¿Cuál de las propuestas le conviene? c) ¿Crees que son justos los intereses de las tres propuestas? 

Configuración del plan

El docente debe pedir a los estudiantes que propongan estrategias, tomando como punto de partida los datos, las condiciones y las interrogantes, e induciendo al descubrimiento de estrategias a través de las siguientes preguntas: ¿Alguna vez resolvieron un problema parecido? ¿Observan relación entre los datos? ¿Identifican relación entre los datos y la incógnita? Si los estudiantes aún no construyeron sus estrategias, el docente debe proporcionar, a cada grupo, un problema parecido de menor dificultad para identificarlas. Lista sistemática en cuadro de doble entrada de tiempo, capital y tasa. Construiremos un modelo matemático con tasa anual, mensual y diaria, a través de la generalización. Construiremos por separado cada modelo que represente año, meses, días, respectivamente.

 Ejecución del plan El docente debe acompañar a los estudiantes durante el proceso de construcción de sus aprendizajes. Primera propuesta: cuando el tiempo es en años. a) Para resolver este problema, utilizaremos la estrategia de lista sistemática en una tabla de doble entrada y por inducción deduciremos una fórmula (estrategia de generalización).

TIEMPO (años)

CAPITAL (S/)

TASA (anual)

INTERÉS (S/)

1

3600

6%

(1)216 = (1)(3600)(6 %)

2

3600

6%

(2)216 = (2)(3600)(6 %)

3

3600

6%

(3)216 = (3)(3600)(6 %)

131









T

C

r%

I=t . C . r%

Luego de tres años, el interés será igual a la suma de los intereses de cada año, es decir, S/. 648. O sea: 3 x 3600 x 6 % = 648 Como podemos observar, para obtener el interés se multiplicaron el tiempo, el porcentaje y el capital. De lo anterior, deducimos:

I=

I=C . r %.t o

C .r . t 100

Donde: I: interés simple; C: Capital; r%: tasa; t: tiempo (anual) El monto que pagará el comunero al prestamista luego de 3 años será: 3600 + 648 = 4248 soles De lo anterior, deducimos que el modelo matemático del monto (M) es:

M =C+ I

Donde: M: monto; C: capital; I: interés simple Segunda propuesta: cuando el tiempo es en meses.  El segundo prestamista ofreció 3600 soles, con una tasa anual del 6 % pagadero en 8 meses. b) Para resolver este problema, utilizaremos la estrategia de lista sistemática en una tabla de doble entrada y, por inducción, deduciremos una fórmula (estrategia de generalización). TIEMPO 1 año

CAPITAL 3600

TASA (anual) 6%

1 mes

3600

6%

2 meses

3600

6%

3 meses

3600

6%







3600

6%

8 3600×6 × 12 100







C

r%

⋮ 8 meses

⋮ t (meses)

INTERÉS (soles)

1×3600×6 100 1 3600×6 × 12 100 2 3600×6 × 12 100 3 3600×6 × 12 100

t .C.r . I = 1200

Luego de 8 meses, el interés será igual al producto del número de meses por 1/12 (porque el año tiene 12 meses) con el interés de un año.

132

1 3600×6 8× × =144 12 100 O sea: Como podemos observar, la tabla es una buena estrategia para construir por inducción un modelo matemático de interés simple, cuando el tiempo es en meses. En este caso, para obtener el interés, se multiplicó el tiempo en meses, con la 12. a parte del interés en un año:

I=

t .C .r . 1200

En el caso particular, el interés, luego de 8 meses, es 144 soles. El monto a pagar luego del plazo previsto será: 3600 + 144 = 3744 soles. Para responder al monto, hemos sumado el capital prestado con el interés simple que generó. 

El tercer prestamista ofreció prestar 3600 soles con una tasa anual del 6 % pagadero en 180 días.

c) Para resolver este problema, utilizaremos la estrategia de lista sistemática en una tabla de doble entrada y, por inducción, deduciremos una fórmula (estrategia de generalización).

TIEMPO

CAPITAL

TASA (anual)

INTERÉS (soles)

1 año

3600

6%

1 mes

3600

6%

1 día

360

6%

2 días

3600

6%

3 días

3600

6%









180 días

4800

6%

180 1 3600×6 × × 30 12 100









t (días)

C

r%

1×3600×6 100 1 3600×6 × 12 100 1 1 3600×6 × × 30 12 100 2 1 3600×6 × × 30 12 100 3 1 3600×6 × × 30 12 100

t .C .r . I = 36000

Luego de 180 días, el interés será igual al producto del número de días por 1/30 (debido a que el mes tiene 30 días) y el interés de un mes.

180× Ósea:

1 1 3600×6 × × = 30 12 100 108

133

Como podemos observar, la tabla es una buena estrategia para construir por inducción un modelo matemático de interés simple, cuando el tiempo es en días. En el caso particular, para obtener el interés, se multiplicó el tiempo en días con la 30. a parte del interés en un mes.

I=

t .C . r . 36000

Luego el monto a pagar será: 3708 soles. Los estudiantes socializan sus resultados, explicando los procedimientos y resultados finales. 

Examinar el proceso seguido

El docente debe pedir a los estudiantes que revisen todo el proceso y que luego respondan las preguntas finales. Ahora estamos en condiciones de responder las dudas del comunero. Montos a pagar: a) Al primer prestamista : 4248 soles. b) Al segundo prestamista : 3744 soles. c) Al tercer prestamista : 3708 soles.

Observaciones: Si el tiempo es en meses, la fórmula de interés simple se divide entre 12. Si el tiempo es en días, la fórmula de interés simple se divide entre 360. Si el tiempo es bimestral, la fórmula de interés simple se divide entre 6. Si el tiempo es trimestral, la fórmula de interés simple se divide entre 4. Si el tiempo es semestral, la fórmula de interés simple se divide entre 2. CIERRE El docente debe sistematizar los conocimientos trabajados en la sesión. Además, debe inducir a los estudiantes a realizar un proceso metacognitivo, a través de preguntas para que reflexionen sobre la construcción de los conocimientos adquiridos. FICHA DE COEVALUACIÓN Equipo:

Docente:

CRITERIOS INTEGRANTES

Trabaja en orden y con disciplina.

Aporta en el trabajo grupal.

Respeta las ideas y opiniones de sus compañeros.

Cumple con la tarea asignada.

1 2 3 4 PUNTUACIÓN

SIEMPRE: 5 puntos

CASI SIEMPRE: 4 puntos A VECES: 3 puntos NUNCA: 2 puntos

2.2.2. Financiando el techado del aula Competencia

Capacidades

Indicadores 134

NOTA

Piensa y actúa matemáticamente en situaciones de cantidad.

Compara y contrasta modelos de tasas de interés compuesto al vincularlos con situaciones de decisión financiera.

Matematiza situaciones.

INICIO El docente debe dar la bienvenida y, a continuación, entregar la situación problemática a los estudiantes. Situación problemática 2 Junio es un periodo que afecta a la región Puno, debido a la presencia de heladas. Por este motivo, los padres de familia de los estudiantes de segundo grado de secundaria del colegio Los Libertadores del distrito de Chucuito desean prestarse dinero para concluir el techado de su aula. El docente Ávalos desea aprovechar el contexto para desarrollar interés compuesto con sus estudiantes presentando la siguiente situación: Si nos prestamos 1000 soles con capitalización compuesta en 5 años y una tasa efectiva del 5 %: ¿Cómo construiré el modelo matemático que me permita saber el monto que pagaré durante los 5 años? ¿Cuál será el monto a pagar al final del tiempo pactado? ¿Qué consejos darías a los padres de familia? El docente aconseja desarrollar la clase de la siguiente forma: Se debe organizar a los estudiantes en equipos de trabajo para que se ayuden entre ellos. Seguidamente, debe motivarlos a través de una actividad relacionada con la situación problemática. La motivación es necesaria para que el estudiante se interese y se produzca la recepción de la información. Luego debe recoger los saberes previos de los estudiantes, para conocer aspectos como vivencias, inquietudes, habilidades. Estos datos son la base del desarrollo del nuevo aprendizaje y permitirán al docente trabajar dentro de la zona de desarrollo próximo de los estudiantes, a través de una didáctica pertinente. ¿Qué desean hacer los padres de familia? ¿Qué idea tienes sobre tasa de interés? ¿Por qué se paga interés de un dinero prestado? ¿Un dinero prestado a mayor tiempo ganará más o menos interés? A continuación, debemos generar conflicto cognitivo en los estudiantes, a través de un pregunta retadora. ¿Qué diferencia existe entre interés simple e interés compuesto? ¿Qué modelo matemático describe el comportamiento del interés compuesto? El docente debe mencionar el propósito de la sesión, para que los estudiantes estén informados sobre lo que se quiere lograr en ellos y la forma como serán evaluados. El propósito de esta sesión es construir un modelo matemático de interés compuesto, a partir de una situación problemática, y responder las preguntas. 135

Los estudiantes serán evaluados durante el proceso, a través de una ficha de cotejo cuyos criterios estarán pegados a un costado de la pizarra.

DESARROLLO En esta etapa, los estudiantes deben construir sus aprendizajes, a través de actividades que desarrollen la situación problemática. Para este caso, el docente sugerirá a los estudiantes utilizar los cuatro pasos de Pólya.

Comprender el problema Se debe pedir a los estudiantes que hagan una lectura analítica de la situación problemática entregada. Para verificar si están entendiendo, el docente debe interrogar sobre la identificación de datos, condiciones y preguntas. a) ¿Qué datos tenemos? Préstamo: 1000 soles Tiempo: 5 años Tasa de interés: 5 % b) ¿Qué condición se visualiza? Condición: capitalización compuesta c) ¿Qué nos pide el problema? ¿Cómo construiré el modelo matemático que me permita saber el monto que pagaré durante los 5 años? ¿Cuál es el monto a pagar al final del tiempo pactado? ¿Qué consejos darías a los padres de familia?



Configuración del plan

En esta etapa, el docente debe pedir a los estudiantes que propongan estrategias de resolución. Si no lo consiguen, se deben poner varias estrategias en la pizarra para que los estudiantes opten por algunas. Si aún no lograron identificar, debe dar a los estudiantes una situación problemática más sencilla, o, en todo caso, una situación problemática similar resuelta para que identifiquen la estrategia utilizada. El docente debe explicar en qué consiste cada una de las estrategias propuestas. El docente propone las siguientes estrategias para que los estudiantes opten por algunas: Cuadro de doble entrada, lista sistemática, ensayo error, ecuaciones, generalización, simplificar el problema, etc. De lo anterior, los estudiantes eligen: a) Estrategia de lista sistemática: colocar secuencialmente en una tabla los datos de tiempo, capital, tasa y deuda al final del periodo. b) Secuencialmente, se va agregando al capital anterior el interés para obtener el monto del primer año, segundo año, etc. c) Generalización para llegar al modelo matemático.

136



Ejecución del plan

En esta etapa, el docente debe estar atento para que todos desarrollen las estrategias seleccionadas, que todos trabajen. Si el estudiante se traba, el docente debe intervenir a través de preguntas que le permitan construir la idea. Si los estudiantes tienen dificultades con las operaciones algorítmicas, el docente debe facilitar libros de ayuda; pero si algún grupo ya desarrolló el problema, se le debe pedir que socialice sus estrategias con sus compañeros. Para resolverlo, utilizaremos la estrategia de lista sistemática en un cuadro de doble entrada. A través de una secuencia analítica, llegaremos a la respuesta.

t: tiempo (periodo)

C: Capital que se adeuda al inicio del periodo

i : tasa de intereses del periodo

Cantidad que se adeuda al final del periodo

1

1000

1000 + 50 = 1050

2

1050

3

1102,50

4

1157,63

5

1215,51

5 ×1000=50 100 5 ×1050=52 .50 100 5 ×1102.50 100 = 55,13 5 ×1157 .63 100 = 57,88 5 ×1215 .51 100 = 60,78

1050 + 52,50 = 1102,50 1102.50 + 55,13 = 1157,63 1157,63 + 57.88 = 1215,51 1215,51 + 60.78 = 1276,28

Como podemos observar, en el cuadro anterior, la estrategia didáctica utilizada fue el desarrollo secuencial por cada periodo hasta llegar a los 5 años, cuyo monto resultó 1276.28 soles. A partir del caso particular anterior, construiremos el modelo matemático de interés compuesto. TIEMPO (Años)

TASA i

1

i

2

i

MONTO POR AÑOS

Co ) Co C o i = C o ( 1+i )1 + C o ( 1+i )1 M1 = C o ( 1+i ) i + C o ( 1+i ) (Monto inicial

C o ( 1+i ) (1+ i ) = C o ( 1+i )2 C o ( 1+i )2 M =

INTERÉS

Co i

C o ( 1+i )

C o ( 1+i ) (1+ i ) i

2

3

i

C o ( 1+i ) (1+ i ) + C o ( 1+i ) (1+ i ) i C o ( 1+i ) (1+ i )(1+i)= C o ( 1+i )3 M3 =





t

i

⋮ t

M=

C o ( 1+i )3 i

C o ( 1+i )3

⋮ C o ( 1+i ) 137

i



Examinar el proceso seguido

En esta etapa, el docente debe pedir a los estudiantes que verifiquen todo el proceso, si la respuesta responde a la pregunta, si existen otras formas de resolver el mismo problema. Como podemos observar, llegamos a la misma respuesta de forma analítica o utilizando fórmula:

C ( 1+ i )t

o M= = 1000(1+5 %)5 = 1000(1+0,05)5 = 1000(1,05)5 = 1000(1,276282) = 1276,278 Para reforzar el significado de interés simple y compuesto, podemos hacer un sociodrama utilizando billetes didácticos. Durante el proceso, el docente debe actuar como facilitador del aprendizaje, permitiendo que los estudiantes construyan sus propios aprendizajes a partir de situaciones reales y cercanas a su contexto.

CIERRE El docente debe sistematizar todos los conocimientos tratados durante la sesión con la participación de los estudiantes. En el cierre, se hace una reflexión sobre la sesión, es decir, se realiza un proceso metacognitivo a través de preguntas. ¿Qué opinión te merece la construcción del modelo matemático de interés compuesto? ¿Qué dificultades tuviste durante el desarrollo de la sesión? ¿En qué otras situaciones de la vida real utilizarás interés compuesto? Observación: el proceso pedagógico debe ser registrado en una lista de cotejo.

138

Evaluación de interés simple y compuesto EVALUACIÓN FORMATIVA. Instrumento: escala descriptiva. Técnica: observación sistemática Muchas Algunas CRITERIOS Siempre Nunca veces veces Participa en el trabajo cooperativo. Hace preguntas durante el proceso. Argumenta con claridad sus resultados. Muestra dificultades durante el proceso de generalización. Se empodera del método de Pólya. Demuestra curiosidad, asombro e interés. Permite que todos los miembros de su equipo participen.

3. Herramientas para la nueva práctica

3.1. Actividades de reflexión individual a) ¿Qué materiales o recursos disponibles de la zona utilizarás para construir el significado del contenido disciplinar de interés simple e interés compuesto en tus estudiantes? b) ¿Qué estrategias didácticas utilizarás para construir en tus estudiantes el significado y operación de interés simple y compuesto? c) ¿Qué parte de la unidad te pareció interesante e innovador y que ayudará en tu práctica pedagógica? d) ¿Con qué otras áreas articularías los temas tratados en esta sección?

3.2. Actividades a distancia A continuación, se presentan actividades para poner en práctica lo desarrollado en esta unidad, las cuales serán ejecutas en el componente a distancia.

a) A partir de la situación significativa de su unidad didáctica, formula una situación problemática de acuerdo a tu contexto intercultural, que implique desarrollar conocimientos relacionados con intereses. b) Selecciona una competencia, capacidades, indicadores, estrategias, recursos y materiales (actores y/o escenarios), a partir de una situación problemática. c) Elabora un diseño de sesión de aprendizaje, tomando en cuenta la actividad anterior. d) Ejecuta la sesión elaborada y recoge las evidencias correspondientes para que sean adjuntadas al portafolio (fotos, videos, audios, etc.). e) Archiva el diseño de la sesión e instrumento de evaluación en tu portafolio.

139

3.3. Actividades metacognitivas

a) ¿Cómo puedes relacionar la información de interés simple con situaciones reales? b) ¿Cuál es la función del docente en un enfoque por competencias? c) ¿Menciona tus conclusiones sobre las actividades propuestas? d) ¿Qué estrategia utilizaste para participar activamente en el proceso? e) ¿Qué lecciones aprendidas pueden mencionar? f)

¿Cuál es el rol protagónico que asumirás en tu institución educativa?

e) ¿Tuviste dificultades en el desarrollo hasta esta parte?, ¿cómo las superaste? f)

¿Qué rol asumes en el desarrollo de los trabajos cooperativos?

3.4. Autoevaluación Indicaciones. Estimado docente, evalúa tu participación durante el taller. N.o Apellidos y nombres: 1

3

Participé activamente en las actividades indicadas durante el desarrollo de la sesión. Escuché con atención las indicaciones y respeté las normas de trabajo señaladas. Cumplí con el tiempo previsto con las actividades señaladas.

4

Respeté a mis colegas y contribuí en las actividades grupales.

5

Me interesé en el tema y aclaré mis dudas.

2

4. Glosario Interés simple. Beneficio que se obtiene de una inversión. Monto. Es la suma del capital y su interés. Capital. Es el dinero invertido. Tasa de interés. Es el porcentaje que se traduce en monto de dinero. Interés compuesto. Beneficio que se obtiene de una inversión. Usura. Interés excesivo en un préstamo. Jubileo. Periodo de cancelación de las deudas.

5. Texto complementario Los incas y su extraña economía 140

Si

No

En los siglos XV y XVI, el Imperio inca fue el más grande de América del Sur. Centrado en el Perú, se extendía a través de cimas de las montañas de los Andes y abajo de la costa. Con la incorporación de las tierras de las actuales Colombia, Chile, Bolivia, Ecuador, Argentina y Perú, todos conectados por una red de caminos extensos, que por su complejidad rivalizaba con cualquiera en el Viejo Mundo. Diversos en los productos alimenticios, textiles, oro y coca, los incas eran maestros de la construcción de la ciudad; sin embargo, no tenían dinero. De hecho, no tenían mercados en absoluto. Muchos aspectos de la vida incaica siguen siendo un misterio; en parte, porque nuestros relatos de la vida de los incas provienen de los invasores españoles que, efectivamente, acabaron con ellos.  Pero el verdadero golpe llegó casi una década antes, cuando los invasores europeos, sin saberlo, desataron una epidemia de viruela que algunos epidemiólogos creen que pudo haber matado hasta el 90 por ciento de los incas. Nuestro conocimiento de estos hechos y nuestra comprensión de la cultura de la época incaica vienen de tan solo unos pocos observadores –en su mayoría, los misioneros españoles– y de un sacerdote e historiador mestizo llamado Blas Valera, nacido en el Perú dos décadas después de la caída del Imperio incaico. 

Riqueza sin dinero Los documentos de los misioneros y de Valera describen al pueblo inca como constructores y planificadores de la tierra, capaces de agricultura de montaña muy sofisticada y constructores de ciudades espléndidas. La sociedad inca era tan rica que podía permitirse el lujo de tener a cientos de personas que se especializaban en la planificación de los usos agrícolas de las zonas recién conquistadas. Ellos construyeron granjas terrazas en las laderas de las montañas, cuyos cultivos de patatas, maíz, cacahuetes y calabaza fueron elegidos cuidadosamente para prosperar en las temperaturas promedio para diferentes altitudes. También cultivaban árboles para mantener la capa superficial del suelo fino en buenas condiciones. Arquitectos incas eran igualmente talentosos, diseñaron y levantaron pirámides enormes, regaron con obras hidráulicas sofisticadas –como las que se encuentran en Tipón– y crearon enormes templos como Pachacámac, junto con retiros de montaña como Machu Picchu. Los diseñadores utilizaron un sistema de cuerdas anudadas para hacer los cálculos necesarios para construir en pendientes.  http://www.taringa.net/posts/ciencia-educacion/15483232/Los-Incas-y-su-extrana-economia.html

6. Referencias Aliaga. C. (2015). Aplicaciones prácticas de Matemática Financiera. Lima: Editorial Universidad. Escudero, F. (2005). Matemática quinto de secundaria. Lima: Ediciones el Nocedal SAC . Farfán. O. (1988). Aritmética (Teoría, práctica y aplicación actualizada). Lima: Editorial San Marcos. Galdós, L. (2005). Dominando Las Matemáticas. Lima: Grupo La República.

Lexus. (2003). La biblia de las Matemáticas. México: Editorial Letrarte S. A. Velasquez, C. (2005). Aritmética Regla de Interés. Lima: Cuzcano.

141

142

Anexo EVALUACIÓN DE LA UNIDAD V Tabla de especificaciones La tabla de especificaciones es un instrumento de evaluación que contiene información sobre el contenido esencial de lo que se va a evaluar en esta unidad.

Funciones de la tabla de especificaciones La tabla de especificaciones sirve para: 1. Construir los ítems del instrumento de evaluación. 2. Orientar la interpretación de los resultados.

Tabla de especificaciones CÓDIGO PSPCRIS 1 PSPCRIC 2

RSPCRIS 3

RSPCRIC 4

DESCRIPTORES Plantea situaciones problemáticas de cantidad relacionadas con interés simple, considerando recursos, actores y escenarios rurales. Plantea situaciones problemáticas de cantidad relacionadas con interés compuesto, considerando recursos, actores y escenarios rurales. Resuelve situaciones problemáticas de cantidad relacionadas con interés simple, en un contexto intercultural, utilizando los cuatro pasos de Pólya y describiendo las estrategias empleadas en cada una de las fases. Resuelve situaciones problemáticas de cantidad relacionadas con interés simple, en un contexto intercultural, utilizando los cuatro pasos de Pólya y describiendo las estrategias empleadas en cada una de las fases.

TOTAL

N.° DE ÍTEMS 1 1

1

1

4

Pruebas de desempeño A diferencia de las pruebas de opción múltiple que son instrumentos de evaluación cerrados, las pruebas de desempeño son instrumentos de evaluación abiertos que permiten evaluar los procesos de planteamiento y resolución de diversas situaciones problemáticas de cantidad. Consistencia de las pruebas de desempeño Una prueba de desempeño es consistente si corresponde a la tabla de especificaciones, es decir, si cada uno de los ítems de la prueba responde a los descriptores y es un representante significativo del descriptor. PSPCRIS 1 Plantea situaciones problemáticas de cantidad que tengan las siguientes características: a) Los datos deben partir de actividades de contexto rural. b) Deben considerar recursos y actores locales. c) Deben inducir a la construcción del significado de interés simple. d) Deben permitir formular preguntas del saber, hacer y ser. 143

PSPCRIC 2 Plantea situaciones problemáticas de cantidad que permitan utilizar interés compuesto y que tengan las siguientes características: a) Los datos deben partir de situaciones de contexto rural. b) Deben considerar recursos y actores locales. c) Deben inducir a la construcción del significado de interés compuesto. d) Deben permitir formular preguntas del saber, hacer y ser. RSPCRIS 3 La señora Orfila, una pequeña comerciante del distrito de Lampa, región Puno, está buscando ampliar su capital de trabajo, por lo que decide solicitar un préstamo de S/ 10 000 a una entidad bancaria. El funcionario del banco hace una simulación por un préstamo efectivo en cuotas normales y le dice: Opción 1: El préstamo se cancela en 6 meses. El monto de la cuota es S/ 1820,61 Opción 2: El préstamo se cancela en 12 meses. El monto de la cuota es S/ 980,65 Opción 3: El préstamo se cancela en 12 meses. El monto de la cuota es s/.980,64 Ponte en el lugar de la señora Orfila y escoge una opción entre la 1, la 2 y la 3. Luego responde las siguientes preguntas y socializa tus respuestas. a) ¿Cómo sabré cuánto pagaré en total al banco? b) ¿Cuál es la diferencia entre el monto total de dinero pagado y el capital prestado? c) ¿Da una razón por la que consideras que existe esa diferencia? d) ¿Qué opción recomendarías a la señora Orfila? RSPCRIC 4 Un campesino productor de cacao en la Amazonía peruana abre una cuenta bancaria el 14 de abril con S/ 1000, percibiendo una tasa nominal mensual del 4 % con capitalización diaria. El 2 de mayo retira S/ 400, el 15 de mayo retira S/ 200 y el 3 de junio deposita S/ 100. a) ¿Cuál es el interés hasta el 2 de mayo? b) ¿Cómo sabré el monto que acumuló desde la fecha de su depósito inicial hasta el 24 de junio, fecha en que canceló la cuenta? c) ¿Qué recomendaciones darías al campesino?

144

Fuente:http://www.agraria.pe/noticias/peru-tiene50-mil-hectareas-disponibles-10028

Related Documents


More Documents from "yo_solo"